Trang chủ Đề thi & kiểm tra Khác 1400 câu trắc nghiệm Đọc hiểu Tiếng Anh có đáp án !!

1400 câu trắc nghiệm Đọc hiểu Tiếng Anh có đáp án !!

Câu 2 :
The word “graduate” in line 2 is closest in meaning to ......

A. finish studying

B. start studying

C. study

D. learn

Câu 4 :
The word “employment” in line 4 refers to the ......

A. work

B. music

C. play

D. money

Câu 5 :
Medical students have to follow a course lasting for ............

A. four to five years

B. only 4 years

C. six or seven years

D. about 5 years

Câu 8 :
Which of the following is NOT mentioned in the passage as full-time
university students?

A. have other work

B. go to lectures

C. study by themselves

D. become members of sports clubs

Câu 9 :
Read the following passage and mark the letter A, B, C or D on your answer sheet to indicate the correct answer to each of the questions
It is difficult to write rules that tell exactly when we should apologize, but it is not difficult to learn how. If we have done something to hurt someone’s feeling or if we have been impolite or rude, we should apologize. An apology indicates that we realize we have made a mistake, and we are sorry for it. It is a way of expressing our regret for something. When we apologize, we admit our wrongdoing, usually offer a reason for it, and express regret. The simplest way to apologize is to say “I’m sorry”, but often that is not enough. Let’s take a common situation. Mario is late for class and enters the classroom, interrupting the teacher in the middle of the class. What does he do? The most polite action is usually to take a seat as quietly as possible and apologize later. But if the teacher stops and waits for him to say something, he could apologize simply “I’m sorry I’m late”, ask permission to take his seat, and sit down. Naturally, more than this, a reason for the tardiness, is needed, but this is not the time or the place for it because he has already caused one interruption and doesn’t need to make it any longer or worse than it already is.When we apologize, ..............

A. we express our sadness and unluckiness.

B. we realize our wrongdoing.

C. we express our happiness.

D. we admit our wrongdoing, offer a reason for it, and express regret.

Câu 10 :
We should apologize .........

A. when we feel tired and make mistake.

B. when we have been impolite, rude or done something to hurt someone’s feeling.

C. when we are angry with somebody about something.

D. when we are not happy or lose something

Câu 11 :
Is it difficult to learn how to apologize somebody?

A. Yes, it is.

B. not difficult.

C. No, it isn’t.

D. No, hasn’t

Câu 12 :
The word “rude” in paragraph 1 could be best replaced by .................

A. incorrect behavior

B. polite

C. correct behavior

D. good behavior

Câu 13 :
Which of the following is NOT mentioned in the passage as the most polite action in Mario’s case?

A. keep quiet

B. apologize later

C. interrupt the teacher

D. take a seat

Câu 14 :
What is the simplest way to apologize?

A. We express our regret.

B. We said nothing.

C. We say “I’m sorry”.

D. We admit our wrongdoing

Câu 16 :
Read the following passage and mark the letter A, B, C, or D on your answer sheet to indicate the correct answer to each of the questions
On the fourth Thursday in November, in houses around the United States, families get together for a feast, or a large meal. Almost all families eat turkey and cranberry sauce for this meal, and have pumpkin pie for dessert. This feast is part of a very special day, the holiday of Thanksgiving. In 1620 the Pilgrims made a difficult trip across the ocean from England. They landed in what is now Massachusetts. In England the Pilgrims had not been allowed to freely practice their religion. So they went to the New World in search of religious freedom. The Pilgrims' first winter was very hard. Almost half the group died of cold, hunger and disease. But the Indians of Massachusetts taught the Pilgrims to plant corn, to hunt and to fish. When the next fall came, the Pilgrims had plenty of food. They were thankful to God and the Indians and had a feast to give thanks. They invited the Indians to join them. This was the first Thanksgiving. Thanksgiving became a national holiday many years later because of the effort of a woman named Sarah Hale. For forty years Sarah Hale wrote to each president and asked for a holiday of Thanksgiving. At last she was successful. In 1863 President Lincoln declared Thanksgiving a holiday. How much is Thanksgiving today like the Pilgrims’ Thanksgiving? In many ways they are different. For example, historians think that the Pilgrims ate deer, not turkey. The idea of Thanksgiving, though, is very much the same: Thanksgiving is a day on which we celebrate and give thanks. When did the the Pilgrims make a difficult trip to across the ocean from England?

A. in 1863

B. in 1621

C. in 1830

D. in 1620

Câu 17 :
The Pilgrims immigrated to the New World because ..............

A. They wanted to search for religious freedom.

B. They wanted to be taught how to plant corn.

C. They wanted to have more land to cultivate.

D. They wanted to make a difficult trip

Câu 18 :
According to the passage, today’s Thanksgiving ..........

A. is only celebrated in Massachusetts.

B. is a day on which the Pilgrims eat deer.

C. is different from the Pilgrims’s Thanksgiving in many ways.

D. is just like the Pilgrims’s Thanksgiving.

Câu 19 :
Which of the following is NOT true about Thanksgiving?

A. It is celebrated on the fourth Thursday on November.

B. It is a day on which Americans celebrate and give thanks.

C. Americans usually have turkey, cranberry sauce and pumpkin pie for this occasion.

D. It became a national holiday thanks to President Lincoln’s 40-year efforts.

Câu 20 :
The word “they” in paragraph 3 refer to ...........

A. families

B. the Pilgrims

C. thanks

D. the Native Americans

Câu 21 :
A housewife’s services in a family of five people are worth ............

A. $160 a mouth on average

B. nearly $ 1,000 a mouth on average.

C. more than $1,000 a mouth on average.

D. $1,600 a mouth on average

Câu 23 :
According to the researchers, husbands should ............

A. pay wages to their wives for their housework.

B. hire others to take over their wives’ household chores.

C. help their wives with the housework.

D. care for the children

Câu 24 :
The word “employers” in the passage refers to ..............

A. their sponsors

B. their husband

C. their owners

D. their bosses

Câu 25 :
What is NOT true about the passage?

A. Full-time housewives are allowed to pay social security taxes.

B. Women who go to work get more offers than housewives.

C. Unlike working women, housewives get no pay for housework.

D. Housewives’services should be rewarded.

Câu 26 :
Read the following passage and mark the letter A, B, C, or D on your answer sheet to indicate the correct answer to each of the questions.
It is a characteristic of human nature that people like to get together and have fun, and people live during America's frontier days were no exception. However, because life was hard and the necessities of dayto-day living took up their time, it was common for recreation to be combined with activities necessary for survival. One example of such a form of recreation was logrolling. Many frontier areas were heavily wooded, and in order to settle an area it was necessary to move the trees. A settler could cut down the trees alone, but help was needed to move the cut trees. After a settler had cut a bunch of trees, he would then invite is neighbours over for a logrolling. A logrolling is a community event where families got together for a combination of work and fun. The women would bring food and have a much needed and infrequent opportunity to relax and chat with friends, the children would play together exuberantly, and the men would hold lively competitions that involved rolling logs from place to place as quickly as possible. This was a day of fun for everyone involved, but as its foundation was the need to clear the land.
The main idea of the passage is that in America's frontier days ..............

A. people combined work with recreation

B. people cleared land by rolling logs

C. it was necessary for early settlers to clear the land

D. a logrolling involved the community

Câu 27 :
The expression day-to-day could best be replaced by which of the following?

A. daytime

B. every day

C. day after day

D. today

Câu 28 :
According to the passage, what did people have to do first to settle an area?

A. Develop recreation ideas

B. Build farms

C. Get rid of the trees

D. Invite neighbors over

Câu 29 :
According to the passage, which of the following is NOT true about a logrolling?

A. It involved a lot of people.

B. It could be enjoyable.

C. There could be a lot of movement.

D. It was rather quiet

Câu 30 :
This passage would probably be assigned reading in which of the following courses?

A. Forestry

B. Environmental Studies

C. Psychology

D. History

Câu 31 :
Read the following passage and mark the letter A, B, C, or D on your answer sheet to indicate the correct answer to each of the questions
People travel for a lot of reasons: Some tourists go to see battlefields or religious shrines. Others are looking for culture, or simply want to have their pictures taken in front of famous places. But most European tourists are looking for a sunny beach to lie on. Northern Europeans are willing to pay a lot of money and put up with a lot of inconveniences for the sun because they have so little of it. Residents of cities like London, Copenhagen, and Amsterdam spend a tot of their winter in the dark because the days are so short, and much of the rest of the year in the rain. This is the reason why the Mediterranean has always attracted them. Every summer, more than 25 million people travel to Mediterranean resorts and beaches for their vacation. They all come for the same reason, sun! The huge crowds mean lots of money for the economies of Mediterranean countries. Italy’s 30,000 hotels are booked solid every summer. And 13 million people camp out on French beaches, parks and roadsides. Spain’s long sandy coastline attracts more people than anywhere else. 37 million tourists visit yearly, or one tourist for every person living in Spain. But there are signs that the area is getting more tourists than it can handle. The Mediterranean is already one of the most polluted seas on earth. And with increased tourism, it’s getting worse. The French can’t figure out what to do with all the garbage left by campers around St. Tropez. And in many places, swimming is dangerous because of pollution. None of this, however, is spoiling anyone’s fun. The Mediterranean gets more popular every year with tourists. Obviously, they don’t go there for clean water and solitude. They tolerate traffic jams and seem to like crowded beaches. They don’t even mind the pollution. No matter how dirty the water is, the coastline still looks beautiful. And as long as the sun shines, it’s still better than sitting in the cold rain in Berlin, London, or Oslo.
In paragraph 2, cities like London, Copenhagen, and Amsterdam are mentioned ............

A. to prove that they have got more tourism than they handle.

B. to tell us how wealthy their people are.

C. to suggest that these cities lack places of historic interest and scenic beauty.

D. to show that they are not good cities in terms of geography and climate.

Câu 33 :
The writer seems to imply that Europeans travel mostly for the reason that ...............

A. they want to see historic remains or religious spots.

B. they would like to take pictures in front of famous sites.

C. they wish to escape from the cold, dark and rainy days back at home.

D. they are interested in different cultural traditions and social customs

Câu 34 :
The latter half of the last sentence in paragraph 3, “or one tourist for every person living in Spain” means .........

A. every Spanish is visited by a tourist every year.

B. every person living in Spain has to take care of a tourist annually.

C. every year almost as many tourists visit Spain as there are people living in that country.

D. all the 37 million people living in Spain are tourists.

Câu 35 :
The word “tolerate” in paragraph 5 is closest in meaning to ............

A. reject

B. endure

C. exclude

D. neglect

Câu 36 :
The word “solid” in paragraph 3 means most nearly the same as .................

A. having no spaces inside.

B. seeming to be hard to book.

C. having less people than normal.

D. being uncomfortable to live in.

Câu 39 :
What of the following statement is true about life in the future?

A. People will go to work as they do today.

B. Hi-tech equipment will be out of the question.

C. It will be dangerous to drive cars because they are too fast.

D. People can have balanced diets for their meal.

Câu 40 :
Which of the following is NOT true about life in the future?

A. Eating is a problem because food contains too much fat.

B. There’s no need to concentrate much when driving.

C. Contacts between people are almost instant.

D. Getting information is a matter of just a few seconds.

Câu 41 :
According to the passage, what do people use a diagnostic machine for?

A. To make food for them.

B. To find out which foods their body needs.

C. To provide them with food.

D. To sell food for humans

Câu 42 :
Which “reply” CLOSET in meaning to?

A. replay

B. request

C. answer

D. question

Câu 43 :
The word “urgent” in the last paragraph probably means .............

A. expected

B. pressing

C. unnecessary

D. hurry

Câu 44 :
What is the main idea of the passage?

A. What life is like in the future.

B. The role of the computer in future life.

C. What foods people will eat in the future.

D. Life in the future will be the same as life at present.

Câu 45 :
Read the following passage and mark the letter A, B, C, or D on your answer sheet to indicate the correct answer to each of the questions
Herman Melville, an American author best known today for his novel Moby Dick, was actually more popular during his lifetime for some of his other works. He traveled extensively and used the knowledge gained during his travels as the basis for his early novels. In 1837, at the age of eighteen, Melville signed as a cabin boy on a merchant ship that was to sail from his Massachusetts home to Liverpool, England. His experiences on this trip served as a basis for the novel Redburn (1849). In 1841 Melville set out on a whaling ship headed for the South Seas. After jumping ship in Tahiti, he wandered around the islands of Tahiti and Moorea. This South Sea island sojourn was a backdrop to the novel Omoo (1847). After three years away from home, Melville joined up with a U.S. naval frigate that was returning to the eastern United States around Cape Horn. The novel White-Jacket (1850) describes this lengthy voyage as a navy seaman. With the publication of these early adventure novels, Melville developed a strong and loyal following among readers eager for his tales of exotic places and situations. However, in 1851, with the publication of Moby Dick, Melville's popularity started to diminish. Moby Dick, on one level the saga of the hunt for the great white whale, was also a heavily symbolic allegory of the heroic struggle of man against the universe. The public was not ready for Melville's literary metamorphosis from romantic adventure to philosophical symbolism. It is ironic that the novel that served to diminish Melville's popularity during his lifetime is the one for which he is best known today. 
The main subject of the passage is ...........

A. Melville’s travels

B. Moby Dick

C. Melville’s personal background

D. the popularity of Melville’s novels.

Câu 46 :
The word “basis” in paragraph 1 is closest in meaning to ............

A. background

B. message

C. bottom

D. dissertation

Câu 47 :
According to the passage, Melville’s early novels were ...............

A. published while he was traveling

B. completely fictional

C. all about his work on whaling ships

D. based on his travel experience

Câu 48 :
The passage implies that Melville stayed in Tahiti because ...........

A. he had unofficially left his ship

B. he was on leave while his ship was in port

C. he had finished his term of duty

D. he had received permission to take a vacation in Tahiti

Câu 49 :
How did the publication of Moby Dick affect Melville’s popularity?

A. His popularity remained as strong as ever.

B. It caused his popularity to decrease.

C. His popularity increased immediately.

D. It had no effect on his popularity

Câu 50 :
According to the passage, Moby Dick is .............

A. symbolic of humanity fighting the universe

B. a single-faceted work

C. a short story about a whale

D. a 47 adventure

Câu 51 :
Herman Melville, an American author best known today for his novel Moby Dick, was actually more popular during his lifetime for some of his other works. He traveled extensively and used the knowledge gained during his travels as the basis for his early novels. In 1837, at the age of eighteen, Melville signed as a cabin boy on a merchant ship that was to sail from his Massachusetts home to Liverpool, England. His experiences on this trip served as a basis for the novel Redburn (1849). In 1841 Melville set out on a whaling ship headed for the South Seas. After jumping ship in Tahiti, he wandered around the islands of Tahiti and Moorea. This South Sea island sojourn was a backdrop to the novel Omoo (1847). After three years away from home, Melville joined up with a U.S. naval frigate that was returning to the eastern United States around Cape Horn. The novel White-Jacket (1850) describes this lengthy voyage as a navy seaman. With the publication of these early adventure novels, Melville developed a strong and loyal following among readers eager for his tales of exotic places and situations. However, in 1851, with the publication of Moby Dick, Melville's popularity started to diminish. Moby Dick, on one level the saga of the hunt for the great white whale, was also a heavily symbolic allegory of the heroic struggle of man against the universe. The public was not ready for Melville's literary metamorphosis from romantic adventure to philosophical symbolism. It is ironic that the novel that served to diminish Melville's popularity during his lifetime is the one for which he is best known today.
In what year did Melville’s book about his experiences as a cabin boy appear?

A. 1849

B. 1837

C. 1847

D. 1841

Câu 53 :
Read the following passage and mark the letter A, B, C, or D on your answer sheet to indicate the correct answer to each of the questions
The handling and delivery of mail has always been a serious business, underpinned by the trust of the public in requiring timeliness, safety, and confidentiality. After early beginnings using horseback and stagecoach, and although cars and trucks later replaced stagecoaches and wagons, the Railway Mail Service still stands as one of America’s most resourceful and exciting postal innovations. This service began in 1832, but grew slowly until the Civil War. Then from 1862, by sorting the mail on board moving trains, the Post Office Department was able to decentralize its operations as railroads began to crisscross the nation on a regular basis, and speed up mail delivery. This service lasted until 1974. During peak decades of service, railway mail clerks handled 93% of all non-local mail and by 1905 the service had over 12,000 employees. Railway Post Office trains used a system of mail cranes to exchange mail at stations without stopping. As a train approached the crane, a clerk prepared the catcher arm which would then snatch the incoming mailbag in the blink of an eye. The clerk then booted out the outgoing mailbag. Experienced clerks were considered the elite of the Postal Service’s employees, and spoke with pride of making the switch at night with nothing but the curves and feel of the track to warn them of an upcoming catch. They also worked under the greatest pressure and their jobs were considered to be exhausting and dangerous. In addition to regular demands of their jobs they could find themselves the victims of train wrecks and robberies. As successful as it was, “mail-on-the-fly” still had its share of glitches. If they hoisted the train’s catcher arm too soon, they risked hitting switch targets, telegraph poles or semaphores, which would rip the catcher arm off the train. Too late, and they would miss an exchange.
Which of the following can be inferred from the first paragraph?

A. There was a high turnover of railway mail clerks.

B. The development of the mail roads during the second half of the 19th century enabled Post Office Department to focus on timeliness.

C. The Post Office Department was more concerned about speeding up mail delivery than the safety of its clerks.

D. Mail was often lost or damaged as it was exchanged on the mail crane.

Câu 54 :
The word “elite” in the second paragraph is closest in meaning to..............

A. majority

B. superior

C. more capable

D. leade

Câu 55 :
What does the passage mainly discuss?

A. How the mail cranes exchanged the mail.

B. Improvements in mail handling and delivery.

C. How Post Office Trains handled the mail without stopping.

D. The skills of experienced clerks.

Câu 57 :
The word “glitches” in the third paragraph can be replaced by ...........

A. accidents

B. blames

C. advantages

D. problems

Câu 58 :
Which of the following is TRUE according to the passage?

A. The clerk booted out the outgoing mailbag before snatching the incoming bag.

B. Clerks couldn’t often see what they were doing.

C. The Railway Mail clerk’s job was considered elite because it was safe and exciting.

D. Despite their success, railway mail clerks only handled a small proportion of all non-local mail.

Câu 60 :
Read the following passage and mark the letter A, B, C or D on your answer sheet to indicate the correct answer to each of the following questions
Each person has different learning preferences and styles that benefit them. Some may find they have a dominant learning style. Others prefer different learning styles in different circumstances. There is no right or wrong answer to which learning style is best for you - or mix of learning styles. However, by discovering and better understanding your own learning styles, you can employ techniques that will improve the rate and quality of your learning. If you prefer lessons that employ images to teach, you are a visual learner. Visual learners retain information better when it is presented in pictures, videos, graphs and books. They frequently draw pictures or develop diagrams when trying to comprehend a subject or memorize rote information. If you are a visual learner, use pictures, images, color, diagrams and other visual media in your note taking, test preparation and studying. Whenever possible, use pictures instead of text. Try to develop diagrams to comprehend concepts and storyboards to remember important sequences and relationships. Aural (auditory) learners retain information better when it’s presented in lecture format, via speeches, audio recordings, and other forms of verb communication. While a visual learner would prefer to read a book or watch a video, auditory learners would prefer to attend a lecture or listen to a book on tap. Aural learners are also big on sound and music. They can typically sing, play an instrument and identify different sounds. If you are an aural learner, integrate auditory media, listening techniques, sound, rhyme, or even music in your learning and studying. You may also consider using background music and sounds to help you with visualization of processes and systems. For example, if you’re practicing fight procedures, you may consider playing a recording of an aircraft in the background as you study. Replacing the lyrics of a favorite song with information you are learning is a very powerful way to memorize large amounts of information for aural learning. Use this technique and you will never forget the information again. 
What does the passage mainly discuss?

A. Features and techniques of two learning styles.

B. Visual and aural learners’ problems and solutions.

C. Why and how to understand your learning styles.

D. What and how to use your learning techniques

Câu 61 :
By discovering and better understanding your own learning styles, you can improve ..............

A. Your learning quality and quantity.

B. Your learning rate.

C. Your learning styles.

D. Your learning quality and speed.

Câu 62 :
The word “They” in paragraph 2 refers to ...............

A. visual learners

B. pictures

C. videos

D. graphs

Câu 63 :
According to the passage, one benefit of diagrams is ...............

A. to understand concepts.

B. to remember sequences.

C. to understand story boards.

D. to use pictures not texts.

Câu 64 :
All of the following statements are TRUE about visual learners EXCEPT ...............

A. They employ images to teach.

B. They remember graphs well.

C. They prefer pictures to texts.

D. They use story boards for relationships.

Câu 65 :
According to the passage, the benefit of listening to music while learning and studying is that it ..............

A. is a hobby or an interest.

B. attracts your attention.

C. makes you feel relaxed.

D. helps you visualize processes and systems.

Câu 66 :
The author suggests that to remember lessons, aural learners can ................

A. forget melody

B. learn by heart lyrics

C. sing along

D. write songs with your favorite lyrics and information you’re learning.

Câu 67 :
It can be inferred from the passage that a person’s learning style ..............

A. is completely different from others’.

B. can never be best for them.

C. determines learning quality.

D. has its effective technique.

Câu 68 :
Read the following passage and mark the letter A, B, C or D on your answer sheet to indicate the correct answer to each of the following questions
Ranked as the number one beverage consumed worldwide, tea takes the lead over coffee in both popularity and production with 5 million metric tons of tea produced annually. Although much of this tea is consumed in Asian, European and African countries, the United States drinks its fair share. According to estimates by the Tea Council of the United States, tea is enjoyed by no less than half of the U.S population on any given day. Black tea or green tea – iced, spiced or instant – tea drinking has spurred a billion-dollar with major tea producer in Africa and South America and throughout Asia. Tea is made from the leaves from an evergreen plant, Camellia saneness, which grows tall and lush in tropical region. On tea plantation, the plant is kept trimmed to approximately four feet high and as new buds called flush appear; they are plucked off by hand. Even in today’s world of modern agricultural machinery, hand harvesting continues to be the preferred method. Ideally, only the top two leaves and a bud should be picked. This new growth produces the highest quality tea. After being harvested, tea leaves are laid out on long drying racks, called withering racks, for 18 to 20 hours. During this process, the tea softens and become limp. Next, dependent on the type of tea being product, the leaves may be crushed or chopped to release flavor, and then fermented under controlled condition of heat and humidity. For green tea, the whole leaves are often steamed to retain their green color, and the fermentation process is skipped. Producing black teas requires fermentation during which the tea leaves begin darken. After fermentation, black tea is dried in vats to produce its rich brown or black color. No one knows when or how tea became popular, but legend has it that tea as a beverage was discovered in 2737 B.C. by Emperor Shen Nung of China when leaves from camellia dropped into his drinking water as it was boiling over a fire. As the story goes, Emperor Shen Nung drank the resulting liquid and proclaimed that the drink to be most nourishing and refreshing. Though this account cannot be documented, it is thought that tea drinking probably originated in China and spread to other parts of Asia, then Europe, and ultimately to America colonies around 1650. With about half the caffeine content of coffee, tea is often chosen by those who want to reduce, but not necessarily eliminate their caffeine intake. Some people find that tea is less acidic than coffee and therefore easier on the stomach. Others have become interested in tea drinking since the National Cancer Institute publishes its finding on the antioxidant properties of tea. But whether tea is enjoyed for it perceived health benefit, its flavor, or as a social drink, teacups continue to be filled daily with the world’ most popular beverage.
According to the passage, what is implied about tea harvesting?

A. It is totaling done with the assistance of modern agricultural machinery.

B. It is longer done in China.

C. The method has remained nearly the same for a long time.

D. The method involves trimming the uppermost branches of the plant.

Câu 69 :
What does the word “they” in paragraph 2 refer to?

A. tea pickers

B. new buds

C. evergreen plants

D. tropical regions

Câu 70 :
Which of the following is NOT true about the tea production process?

A. Black tea develops its dark color during fermentation and final drying.

B. Green tea requires a long fermentation process.

C. Green tea is often steamed to keep its color.

D. Black tea goes through two phases during production.

Câu 71 :
According to the passage, what is TRUE about origin of tea drinking?

A. It began during the Shen Nung Dynasty.

B. It may begin some time around 1950.

C. It is unknown when tea first became popular.

D. It was originally produced from Camilla plants in Europe.

Câu 73 :
According to the passage, why someone would choose to drink tea instead of coffee?

A. Because it’s easier to digest than coffee.

B. Because it has higher nutritional content than coffee.

C. Because it helps prevent heart attacks.

D. Because it has more caffeine than coffee does.

Câu 74 :
What best describes the topic of the passage?

A. Tea consumption and production.

B. The two most popular types of tea.

C. The benefits of tea consumption worldwide.

D. How tea is produced and brewed.

Câu 75 :
Read the following passage and mark the letter A, B, C or D on your answer sheet to indicate the correct answer to each of the following questions
The economic expansion prompted by the Second World War triggered a spectacular population boom in the West. Of course, the region was no stranger to population booms. Throughout much of its history, western settlement had been characterized by spurts, rather than by a pattern of gradual and steady population growth, beginning with the gold and silver rushes of the 1850's and 1860's. The decade after the First World War - the 1920's - witnessed another major surge of people pouring into the West, particularly into urban areas. But the economic depression of the 1930's brought this expansion to a halt; some of the more sparsely settled parts of the region actually lost population as migrants sought work in more heavily industrialized areas. By 1941 when the United States entered the Second World War and began to mobilize, new job opportunities were created in the western part of the nation. If the expansion of industries, such as shipbuilding and aircraft manufacturing, was most striking on the pacific coast, it also affected interior cities like Denver, Phoenix, and Salt Lake City. Equally dramatic were the effects of the establishment of aluminum plants in Oregon and Washington and the burgeoning steel industry in Utah and California. The flow of people into these areas provided an enormous impetus to the expansion of the service industries - banks, health care services and schools. Although strained to the limit by the influx of newcomers, western communities welcomed the vast reservoir of new job opportunities. At the same time, the unprecedented expansion of government installations in the West, such as military bases, created thousands of new civilian openings. As land had served as a magnet for western migrants in the late nineteenth century, so wartime mobilization set in motion another major expansion of population. Indeed, it could be said that the entire western United States became a giant boomtown during the Second World War. This was especially true of California. Of the more than eight million people who moved into the West in the decade after 1940, almost one-half went to the Pacific coast. In fact, between 1940 and 1950, California's population surged by more than three million people.
What is the main point of the passage?

A. California dominated the economic growth of the West during the Second World War.

B. Industrial growth during the 1940's attracted large numbers of people to the West.

C. The military drew people away from civilian jobs during the 1940's.

D. The West experienced gradual and steady economic growth from 1900 to 1940.

Câu 76 :
The word "triggered" in paragraph 1 is closest in meaning to ...........

A. was connected to

B. generated

C. interfered with

D. illuminated

Câu 77 :
Why does the author mention “the gold and silver rushes of the 1850's and 1860's” in the first paragraph?

A. As causes of gradual population growth.

B. As contrasts to late patterns of population.

C. As illustrations of a market economy.

D. As examples of western population booms.

Câu 78 :
According to the passage, the depression of the 1930's caused which of the following?

A. A lack of population growth in the West.

B. The building of new suburbs.

C. A creation of more job opportunities.

D. A growth in immigration from abroad.

Câu 79 :
The word “it” in paragraph 2 refers to ................

A. expansion

B. Denver

C. manufacturing

D. the Pacific coast

Câu 80 :
The passage suggests that industrialization in the West led to all of the following EXCEPT .............

A. An increase in school construction.

B. Improved access to doctors.

C. An increase in the number of banks.

D. A reduction in the price of land.

Câu 81 :
It can be inferred from the passage that the principal cause of California’s population surge between 1940 and 1950 was ..........

A. the increased availability of land.

B. people’s desire to live in a warm, coastal climate.

C. the industrial mobilization necessitated by the Second World War.

D. overcrowding in urban areas in other regions of the United States

Câu 82 :
Read the following passage and mark the letter A, B, C, or D on your answer sheet to indicate the correct answer to each of the questions
The Development of Refrigeration Cold storage, or refrigeration, is keeping food at temperatures between 32 and 45 degrees F in order to delay the growth of microorganisms - bacteria, molds, and yeast - that cause food to spoil. Refrigeration produces few changes in food, so meats, fish, eggs, milk, fruits, and vegetables keep their original flavor, color, and nutrition. Before artificial refrigeration was invented, people stored perishable food with ice or snow to lengthen its storage time. Preserving food by keeping it in an ice-filled pit is a 4,000-year-old art. Cold storage areas were built in basements, cellars, or caves, lined with wood or straw, and packed with ice. The ice was transported from mountains, or harvested from local lakes or rivers, and delivered in large blocks to homes and businesses. Artificial refrigeration is the process of removing heat from a substance, container, or enclosed area, to lower its temperature. The heat is moved from the inside of the container to the outside. A refrigerator uses the evaporation of a volatile liquid, or refrigerant, to absorb heat. In most types of refrigerators, the refrigerant is compressed, pumped through a pipe, and allowed to vaporize. As the liquid turns to vapor, it loses heat and gets colder because the molecules of vapor use energy to leave the liquid. The molecules left behind have less energy and so the liquid becomes colder. Thus, the air inside the refrigerator is chilled. Scientists and inventors from around the world developed artificial refrigeration during the eighteenth and nineteenth centuries. William Cullen demonstrated artificial refrigeration in Scotland in 1748, when he let ethyl ether boil into a partial vacuum. In 1805, American inventor Oliver Evans designed the first refrigeration machine that used vapor instead of liquid. In 1842, physician John Gorrie used Evans's design to create an air-cooling apparatus to treat yellow-fever patients in a Florida hospital. Gorrie later left his medical practice and experimented with ice making, and in 1851 he was granted the first U.S. patent for mechanical refrigeration. In the same year, an Australian printer, James Harrison, built an ether refrigerator after noticing that when he cleaned his type with ether it became very cold as the ether evaporated. Five years later, Harrison introduced vaporcompression refrigeration to the brewing and meatpacking industries. Brewing was the first industry in the United States to use mechanical refrigeration extensively, and in the 1870s, commercial refrigeration was primarily directed at breweries. German-born Adolphus Busch was the first to use artificial refrigeration at his brewery in St. Louis. Before refrigeration, brewers stored their beer in caves, and production was constrained by the amount of available cave space. Brewing was strictly a local business since beer was highly perishable and shipping it any distance would result in spoilage. Busch solved the storage problem with the commercial vapor- compression refrigerator. He solved the shipping problem with the newly invented refrigerated railcar, which was insulated with ice bunkers in each end. Air came in on the top, passed through the bunkers, and circulated through the car by gravity. In solving Busch's spoilage and storage problems, refrigeration also revolutionized an entire industry. By 1891, nearly every brewery was equipped with mechanical refrigerating machines. The refrigerators of today rely on the same basic principle of cooling caused by the rapid evaporation and expansion of gases. Until 1929, refrigerators used toxic gases - ammonia, methyl chloride, and sulfur dioxide - as refrigerants. After those gases accidentally killed several people, chlorofluorocarbons (CFCs) became the standard refrigerant. However, they were found to be harmful to the earth's ozone layer, so refrigerators now use a refrigerant called HFC 134a, which is less harmful to the ozone.
What is the main reason that people developed methods of refrigeration?

A. They wanted to improve the flavor and nutritional value of food.

B. They needed to slow the natural processes that cause food to spoil.

C. They needed to use for the ice that formed on lakes and rivers.

D. They wanted to expand the production of certain industries.

Câu 83 :
The word “perishable” in paragraph 1 is closest in meaning to ............

A. capable of spoiling

B. uncooked

C. of animal origin

D. highly nutritious

Câu 84 :
What can be inferred from paragraph 1 about cold storage before the invention of artificial refrigeration?

A. It kept food cold for only about a week.

B. It was dependent on a source of ice or snow.

C. It required a container made of mental or wood.

D. It was not a safe method of preserving meat.

Câu 85 :
Artificial refrigeration involves all of the following processes EXCEPT .................

A. the pumping of water vapor through a pipe.

B. the rapid expansion of certain gases.

C. the evaporation of a volatile liquid.

D. the transfer of heat from one place to another.

Câu 86 :
According to the passage, who was the first person to use artificial refrigeration for a practical purpose?

A. William Cullen

B. Oliver Evans

C. John Gorrie

D. Adolphus Busch

Câu 87 :
The word “it” in paragraph 3 refers to ........

A. printer

B. refrigerator

C. type

D. ether

Câu 88 :
The word “constrained” in paragraph 4 is closest in meaning to .

A. restricted

B. spoiled

C. improved

D. alternated

Câu 90 :
Read the following passage and mark the letter A, B, C, or D on your answer sheet to indicate the correct answer to each of the questions
Today, roller skating is easy and fun. But a long time ago, it wasn't easy at all. Before 1750, the idea of skating didn't exist. That changed because of a man named Joseph Merlin. Merlin's work was making musical instruments. In his spare time he liked to play the violin. Joseph Merlin was a man of ideas and dreams. People called him a dreamer. One day Merlin received an invitation to attend a fancy dress ball. He was very pleased and a little excited. As the day of the party came near, Merlin began to think how to make a grand entrance at the party. He had an idea. He thought he would get a lot of attention if he could skate into the room. Merlin tried different ways to make himself roll. Finally, he decided to put two wheels under each shoe. These were the first roller skates. Merlin was very proud of his invention as he dreamed of arriving at the party on wheels while playing the violin. On the night of the party Merlin rolled into the room playing his violin. Everyone was astonished to see him. There was just one problem. Merlin had no way to stop his roller skates. He rolled on and on. Suddenly, he ran into a huge mirror that was hanging on the wall. Down fell the mirror, breaking to pieces. Nobody forgot Merlin's grand entrance for a long time! 
The passage is mainly about ..................

A. how people enjoyed themselves in the 18th century

B. a strange man

C. how roller skating began

D. an unusual party

Câu 91 :
The word "astonished" in paragraph 4 can be best replaced by ........

A. tired

B. surprised

C. embarrassed

D. polite

Câu 92 :
Merlin put wheels under his shoes in order to ..................

A. impress the party guests

B. arrive at the party sooner

C. show his skill in walking on wheels

D. test his invention

Câu 93 :
The word "ball" in paragraph 2 probably means ...............

A. game

B. party

C. round object

D. match

Câu 94 :
People thought Merlin was a dreamer because he ..............

A. was a gifted musician

B. invented the roller skates

C. often gave others surprises

D. was full of imagination

Câu 95 :
What is the main point the writer is trying to make in the last paragraph?

A. Merlin got himself into trouble.

B. Merlin succeeded beyond expectation.

C. The roller skates needed further improvement.

D. The party guests took Merlin for a fool.

Câu 96 :
The word "These" in paragraph 3 refers to ..........

A. wheels

B. roller skates

C. different ways

D. shoes

Câu 98 :
The word "rough"in the passage is closest in meaning to ...........

A. forceful

B. beautiful

C. careful

D. easy

Câu 99 :
The writer's father retired early because ..................

A. he walked on the cliffs every morning.

B. he was unable to get anywhere on time.

C. he had to drive his kid to school everyday.

D. he lost rack of time.

Câu 100 :
According to the passage, all of the following are true EXCEPT ................

A. the writer didn't know how to sail.

B. house to school was 8 miles.

C. the waves came crashing into the writer's front garden in bad weather.

D. the write moved to a new house when he was 8.

Câu 102 :
The writer learnt to sail ................. 

A. When he/ she was eight 

B. when his family moved to a new house 

C. before going to school 

D. before learning to ride a bike

Câu 103 :
When taking the university entrance exams, ..............

A. the weather was terrible

B. the writer's family moved to a new house by the sea

C. the writer had to live in a friend's house

D. the writer's father drove him/ her to university

Câu 104 :
The phrase "All in all "in the passage is closest in meaning to .............

A. In all

B. In the whole

C. On general

D. On the whole

Câu 106 :
The word “interfere” in paragraph 2 most nearly means ..........

A. prescribe

B. aid

C. help

D. hinder

Câu 107 :
Why are beta clockers not prescribed regularly?

A. Students are expected to do poorly.

B. They cause test anxiety.

C. The drugs are only 25 years old.

D. There are side effects.

Câu 108 :
According to the passage, .................. 

A. all people can take beta blockers. 

B. beta blockers are widely prescribed. 

C. beta blockers work only to improve test scores if the test-taker truly knows the material. 

D. beta blockers work only on test anxiety.

Câu 109 :
The expression “readministration” in this passage refers to ................

A. giving the test again to both groups after beta blockers have been administered to one group.

B. giving the test again to people without administering beta blockers.

C. giving the beta blockers without retesting.

D. giving the test to both groups of test-takers and then giving them beta blockers.

Câu 110 :
What possible use for beta blockers was NOT discussed in this passage?

A. Pain relief

B. Anxiety test

C. Heart conditions

D. Minor stress

Câu 111 :
Beta blockers work on some physical and emotional symptoms because they ...........

A. interfere with the side effects of adrenalin

B. primarily change human thought processes

C. produce side effects worse than the symptoms

D. fool a person into a healthier stance

Câu 112 :
Faugel’s research showed that beta blockers given to his sample ...............

A. increased scores the same as the national average.

B. decreased scores.

C. increased scores less than the national average.

D. increased scores much more than the national average.

Câu 114 :
What does the word “filters out” in paragraph 1 probably mean?

A. prevents

B. separates

C. keeps

D. stops

Câu 115 :
What is the most important purpose of the ozone layer?

A. Shielding the sun

B. Protecting the earth

C. Destroying chemicals

D. Providing fluorocarbons

Câu 116 :
What does the word “depletion” in paragraph 2 probably mean?

A. deletion

B. deployment

C. departure

D. destruction

Câu 117 :
What is the ozone layer made of?

A. Oxygen

B. Shields

C. Ultraviolet light

D. Fluorocarbons

Câu 118 :
The speaker’s main topic is ................... 

A. air-conditioning systems 

B. fluorocarbons and the ozone layer 

C. ultraviolet light 

D. the use of spray cans

Câu 119 :
What will the speaker probably discuss next?

A. The make-up of the ozone layer.

B. The sun as a cause of ozone layer depletion.

C. How to make air conditioners with fluorocarbons.

D. Harmful effects of ultraviolet light.

Câu 120 :
Read the following passage and mark the letter A, B, C, or D on your answer sheet to indicate the correct answer to each of the questions
A lot of advice is available for college leavers heading for their first job. In this article we consider the move to a second job. We are not concerned with those looking for a second temporary position while hunting for a permanent job. Nor are we concerned with those leaving an unsatisfactory job within the first few weeks. Instead, we will be dealing with those of you taking a real step on the career ladder, choosing a job to fit in with your ambitions now that you have learnt your way around, acquired some skills and have some idea of where you want to go. What sort of job should you look for? Much depends on your longterm aim. You need to ask yourself whether you want to specialize in a particular field, work your way up to higher levels of responsibility or out of your current employment into a broader field. Whatever you decide, you should choose your second job very carefully. You should be aiming to stay in it for two or three years. This job will be studied very carefully when you send your letter of application for your next job. It should show evidence of serious career planning. Most important, it should extend you, develop you and give you increasing responsibility. Incidentally, if you are interested in traveling, now is the time to pack up and go. You can do temporary work for a while when you return, pick up where you left off and get the second job then. Future potential employers will be relieved to see that you have got it out of your system, and are not likely to go off again. Juliette Davidson spend her first year after leaving St. Aldate’s College working for three lawyers. It was the perfect first job in that “ OK ... they were very supportive people. I was gently introduced to the work, learnt my way around an office and improve my word processing skills. However, there was no scope for advancement. One day, I gave my notice, bought an air ticket and traveled for a year.” Juliette now works as a Personal Assistant to Brenda Cleverdon, the Chief Executive of business in the Community. “In two and a half years I have become more able and my job has really grown”, she says. “ Right from the beginning my boss was very keen to develop me. My job title is the same as it was when I started but the duties have changed. From mainly typing and telephone work, I have progressed to doing most of the correspondence and budgets. I also have to deal with a variety of queries, coming from chairmen of large companies to people wanting to know how to start their own business. Brenda involves me in all her work but also gives me specific projects to do and events to organize.” 
Who is intended to benefit from the advice given in the article?

A. students who have just finished their studies

B. people who are unhappy with their current job

C. those who are interested in establishing a career

D. people who change jobs regularly

Câu 121 :
According to the writer, why is the choice of your second job important?

A. It will affect your future job prospects.

B. It will last longer than your first job.

C. It will be difficult to change if you don’t like it.

D. It should give you the opportunity to study.

Câu 122 :
“It” in the passage refers to ................. 

A. first job 

B. second job

C. application 

D. career

Câu 123 :
If you have a desire to travel, when does the writer suggest that you do it?

A. straight after you have left college

B. when you are unable to find a permanent job

C. after you have done some temporary work

D. between the first and second job

Câu 124 :
What does the phrase “you have got it out of your system” in passage mean?

A. You have planned your career sensibly.

B. You are an experienced traveler.

C. You have satisfied your wish to travel.

D. You have learned to look after yourself.

Câu 125 :
How did Juliette Davidson benefit from the experience of her first job?

A. It was good introduction to working in an office.

B. She met a variety of interesting people.

C. It enabled her to earn enough money to travel.

D. She learnt how to use a word processor.

Câu 126 :
In what way is Juliette’s current job better her first job?

A. She has a more impressive job title.

B. She now know how to start her own business.

C. She has been able to extend her skills.

D. She is more involve in the community

Câu 127 :
Read the following passage and mark the letter A, B, C, or D on your answer sheet to indicate the correct answer to each of the questions
It is commonly believed that school is where people go to get an education. Nevertheless, it has been said that today children interrupt their education to go to school. The difference between schooling and education implied by this remark is important. Education is much more open-ended and all-inclusive than schooling. Education knows no limits. It can take place anywhere, whether in the shower or on the job, whether in the kitchen or on the tractor. It includes both the formal learning that takes place in school and the whole universe of informal learning. The agent (doer) of education can vary from respected grandparents to the people arguing about politics on the radio , from a child to a famous scientist. Whereas schooling has a certain predictability, education quite often produces surprises. A chance conversation with a stranger may lead a person to discover how little is known of other religions. People receive education from infancy on. Education, then, is a very broad, inclusive term; it is a lifelong process, a process that starts long before the start of school, and one that should be a necessary part of one’s entire life. Schooling, on the other hand, is a specific, formalized process, whose general pattern varies little from one setting to the next. Throughout a country, children arrive at school at about the same time, take the assigned seats, are taught by an adult, use similar textbooks, do homework, take exams, and so on. The pieces of reality that are to be learned, whether they are the alphabet or an understanding of the workings of governments, have been limited by the subjects being taught. For example, high school students know that they are not likely to find out in their classes the truth about political problems in their society or what the newest filmmarkers are experimenting with. There are clear and undoubted conditions surrounding the formalized process of schooling. 
In the passage, the expression “children interrupt their education to go to school” mostly implies that ..........

A. schooling prevents people discovering things

B. schooling takes place everywhere

C. all of life is an education

D. education is totally ruined by schooling

Câu 128 :
What does the writer mean by saying “education quite often produces surprises”?

A. Educators often produce surprises.

B. Informal learning often brings about unexpected results.

C. Success of informal learning is predictable.

D. It’s surprising that we know little about other religions.

Câu 129 :
Which of the following would the writer support?

A. Without formal education, people won’t be able to read and write.

B. Going to school is only part of how people become educated.

C. Schooling is of no use because students do similar things every day.

D. Our education system needs to be changed as soon as possible.

Câu 130 :
According to the passage, the doers of education are .............

A. only respected grandparents

B. mostly famous scientists

C. mainly politicians

D. almost all people

Câu 131 :
Which of the following is TRUE according to passage?

A. Education and schooling are quite different experience.

B. The best schools teach a variety of subjects.

C. Students benefit from schools, which require long hours and homework.

D. The more years students go to school, the better their education is.

Câu 132 :
The word “they” in the last paragraph refers to ............

A. workings of governments

B. newest filmmarkers

C. political problems

D. high school students

Câu 133 :
The word “all-inclusive” in the passage mostly means ............

A. including everything or everyone

B. going in many directions

C. involving many school subjects

D. allowing no exceptions

Câu 134 :
This passage is mainly aimed at ................. 

A. telling the difference between the meaning of two related words “schooling” and “education” 

B. telling a story about execellent teachers 

C. listing and discussing several educational problems 

D. giving examples of different schools

Câu 135 :
Read the following passage and mark the letter A, B, C, or D on your answer sheet to indicate the correct answer to each of the questions
In the 1960s, The Beatles were probably the most famous pop group in the whole world. Since then, there havebeen a great many groups that have achieved enormous fame, so it is perhaps difficult now to imagine how sensational The Beatles were at that time. They were four boys from the north of England and none of them had any training in music. They started by performing and recording songs by black Americans and they had some success with these songs. Then they started writing their own songs and that was when they became really popular. The Beatles changed pop music. They were the first pop group to achieve great success from songs they had written themselves. After that it became common for groups and singers to write their own songs. The Beatles did not have a long career. Their first hit record was in 1963 and they split up in 1970. They stopped doing live performances in 1966 because it had become too dangerous for them - their fans were so excited that they surrounded them and tried to take their clothes as souvenirs! However, today some of their songs remain as famous as they were when they first came out. Throughout the world, many people can sing part of a Beatles song if you ask them. 
The passage is mainly about ...............

A. why the Beatles split up after 7 years

B. the Beatles' fame and success

C. many people's ability to sing a Beatles song

D. how the Beatles became more successful than other groups

Câu 136 :
The four boys of the Beatles ................. 

A. came from the same family 

B. were at the same age 

C. came from a town in the north of England 

D. received good training in music

Câu 137 :
The word “sensational” is closest in meaning to .............

A. notorious

B. shocking

C. bad

D. popular

Câu 138 :
The first songs of the Beatles were ............

A. written by black American

B. broadcast on the radio

C. paid a lot of money

D. written by themselves

Câu 139 :
What is NOT TRUE about the Beatles?

A. The members had no training in music.

B. They became famous when they wrote their own songs.

C. They had a long stable career.

D. They were afraid of being hurt by fans.

Câu 140 :
The Beatles stopped their live performances because .............

A. they had earned enough money

B. they did not want to work with each other

C. they spent more time writing their own songs

D. they were afraid of being hurt by fan

Câu 141 :
The word “they” in line 10 refers to ..........

A. the first

B. the singers

C. the songs

D. the performances

Câu 142 :
Read the following passage and mark the letter A, B, C, or D on your answer sheet to indicate the correct answer to each of the questions
In the history of technology, computers and calculators were innovative developments. They are essentially different from all other machines because they have a memory. This memory stores instructions and information. In a calculator, the instructions are the various functions of arithmetic, which are permanently remembered by the machine and cannot be altered or added to. The information consists of the numbers keyed in. An electronic pocket calculator can perform almost instant arithmetic. A calculator requires an input unit to feed in numbers, a processing unit to make the calculation, a memory unit, and an output unit to display the result. The calculator is powered by a small battery or by a panel of solar cells. Inside is a microchip that contains the memory and processing units and also controls the input unit, which is the keyboard, and the output unit, which is the display. The input unit has keys for numbers and operations. Beneath the key is a printed circuit board containing a set of contacts for each key. Pressing a key closes the contacts and sends a signal along a pair of lines in the circuit board to the processing unit, in which the binary code for that key is stored in the memory. The processing unit also sends the code to the display. Each key is connected by a different pair of lines to the processing unit, which repeatedly checks the lines to find out when a pair is linked by a key. The memory unit stores the arithmetic instructions for the processing unit and holds the temporary results that occur during calculation. Storage cells in the memory unit hold the binary codes for the keys that have been pressed. The number codes, together with the operation code for the plus key, are held in temporary cells until the processing unit requires them. When the equals key is pressed, it sends a signal to the processing unit. This takes the operation code-for example, addition-and the two numbers being held in the memory unit and performs the operation on the two numbers. A full adder does the addition, and the result goes to the decoder in the calculator's microchip. This code is then sent to the liquid crystal display unit, which shows the result, or output, of the calculation. 
The word "innovative" in line 1 could best be replaced by ...............

A. recent

B. important

C. revolutionary

D. complicated

Câu 143 :
What can be inferred about machines that are not calculators or computers?

A. They can not store information in a memory.

B. They are less expensive than computers.

C. They have simple memory and processing units.

D. They are older than computers.

Câu 144 :
In what part of the calculator are the processing and memory units?

A. the battery

B. the solar cells

C. the output unit

D. the microchip

Câu 145 :
According to the passage, one function of the memory unit is ...............

A. to control the keyboard

B. to store temporary results during calculation

C. to send codes to the display unit

D. to alter basic arithmetic instructions

Câu 146 :
The word "This" in paragraph 5 refers to .................

A. the plus key

B. the processing unit

C. the memory unit

D. the equals key

Câu 147 :
The word "contacts" in paragraph 3 is closest in meaning to ..............

A. commands

B. codes

C. locations

D. connections

Câu 148 :
Which of the following could NOT be said about calculators?

A. The calculator's "thinking" takes place in the processing and memory units.

B. Calculators require a lot of instructions to operate quickly.

C. Calculators and computers are similar.

D. Pressing a key activates a calculator.

Câu 149 :
What is the main purpose of the passage?

A. To discuss innovative developments in technology

B. To compare computers and calculators with other machines

C. To summarize the history of technology

D. To explain how a calculator works

Câu 150 :
Read the following passage and mark the letter A, B, C, or D on your answer sheet to indicate the correct answer to each of the questions
Facebook is a for-profit online social media and social networking service. The Facebook website was launched on February 4th, 2004 by Mark Zuckerberg, along with fellow Harvard College students and roommates. Since 2006, anyone age 13 and older has been allowed to become a registered user of Facebook though variations exist in minimum age requirement, depending on applicable local laws. Over 25 million people in the Uk use Facebook. That’s 45% of population! And on average, each user spends over six hours a month on Facebook. Though not the highest this is a considerable number. Is Facebook is a dangerous obsession or just harmless fun? Seventeenyear-old Bethan has written on her blog about what it was like to stop using Facebook. I think I am a Facebook addict. I log on to Facebook everyday to chat to my friends real friends and loads of online friends. Sometimes I have ten conversations going at the same time. I upload photos and update my Facebook profile all the time. But recently I’ve started to feel worried if I am offline for more than a few hours. And then last week I forgot to meet a real friend because I was online! I’ve realised I could have a problem. So I’ve decided to give it up for a while. I found it really hard. Facebook and my friends demanded to know why I had left. I spent the first few evenings wondering what everyone was chatting on Facebook. I even phoned a couple of friends to find out. The fourth night I wasn’t quite so bad. I actually concentrated on my homework better and I had more time to watch my TV programmes. And I spoke to my friends during the day at school. At the end of the first week, I reactivated my account, I think Facebook is fun and it’s useful for posting messages to friends and sharing photos. But I’ll try not to spend so much time on it in the future. 
Which of the following is NOT true about Facebook users in UK?

A. 45% of the country’s population used Facebook.

B. More than 25 millions of Brits use Facebook.

C. The amount of time British users spent on Facebook is highest.

D. Averagely, 6 hours per month are spent by British users.

Câu 151 :
Facebook is a for-profit online social media and social networking service. The Facebook website was launched on February 4th, 2004 by Mark Zuckerberg, along with fellow Harvard College students and roommates. Since 2006, anyone age 13 and older has been allowed to become a registered user of Facebook though variations exist in minimum age requirement, depending on applicable local laws. Over 25 million people in the Uk use Facebook. That’s 45% of population! And on average, each user spends over six hours a month on Facebook. Though not the highest this is a considerable number. Is Facebook is a dangerous obsession or just harmless fun? Seventeenyear-old Bethan has written on her blog about what it was like to stop using Facebook. I think I am a Facebook addict. I log on to Facebook everyday to chat to my friends real friends and loads of online friends. Sometimes I have ten conversations going at the same time. I upload photos and update my Facebook profile all the time. But recently I’ve started to feel worried if I am offline for more than a few hours. And then last week I forgot to meet a real friend because I was online! I’ve realised I could have a problem. So I’ve decided to give it up for a while. I found it really hard. Facebook and my friends demanded to know why I had left. I spent the first few evenings wondering what everyone was chatting on Facebook. I even phoned a couple of friends to find out. The fourth night I wasn’t quite so bad. I actually concentrated on my homework better and I had more time to watch my TV programmes. And I spoke to my friends during the day at school. At the end of the first week, I reactivated my account, I think Facebook is fun and it’s useful for posting messages to friends and sharing photos. But I’ll try not to spend so much time on it in the future.
The highlight word “it” in the passage refers to ................

A. worrying

B. facebook

C. a problem

D. meeting her real friend

Câu 152 :
Facebook ............... 

A. was non-profitable. 

B. was launched solely by Mark Zuckerberg. 

C. follows each country’s regulation. 

D. can be used by people of all ages.

Câu 153 :
Which of the following is NOT the thing Bethan does as a Facebook addict?

A. be curious about Facebook activities when logging out.

B. have numerous online conversation at the same time.

C. use Facebook everyday

D. long to be offline nearly all the time.

Câu 154 :
The word “applicable” in paragraph 1 is closest in meaning to ..........

A. different

B. unsuitable

C. relevant

D. opposite

Câu 155 :
What does Bethan conclude about Facebook?

A. The best use of Facebook is to share photos and messages.

B. Facebook is not as good as TV.

C. Users should spend more time on Facebook.

D. Facebook is great as long as not too much time is spent on.

Câu 156 :
What happened to make Bethan decide to quit Facebook for a while?

A. She forgot an offline meeting

B. She started to feel nervous

C. She had too many offline friends

D. She uploaded too many photos

Câu 157 :
When she decided to give up Facebook ..............

A. her friends didn’t care much

B. she found it too difficult to continue

C. she made progress after some days

D. she couldn’t focus on her homework

Câu 158 :
Read the following passage and mark the letter A, B, C, or D on your answer sheet to indicate the correct answer to each of the questions
It’s a sound you will probably never hear, a sickened tree sending out a distress signal. But a group of scientists has heard the cries, and they think some insects also hear the trees and are drawn to them like vulture to a dying animal. Researchers with the US Department of Agriculture’s Forest Service fastened sensors to the bark of droughtstricken trees clearly heard distress calls. According to one of the scientists, most parched trees transmit their plight in the 50-hertz to 50-kilohertz range. (The unaided human ear can detect no more than 20 kilohertz). Red oak, maple, white pine, and birch all make slightly different sounds in the form of vibrations at the surface of the wood. The scientists think that the vibrations are created when the water columns inside tubes that run along the length of the tree break, a result of too little water following through them. These fractured columns send out distinctive vibration pattems. Because some insects communicate at ultrasonic frequencies, they may pick up the trees' vibration and attack the weakened trees. Researchers are now running tests with potted trees that have been deprived of water to see if the sound is what attracts the insects. “Water-stressed trees also smell differently from other trees, and they experience thermal changes, so insects could be responding to something other than sound”, one scientist said. 
All the following are mentioned as possible factors in drawing insects to weakened trees EXCEPT .............

A. thermal changes

B. sounds

C. changes in color

D. smells

Câu 159 :
It can be inferred from the passage that research concerning the distress signals of trees ...............

A. has been unproductive up to now

B. is no longer sponsored by the government

C. was conducted many years ago

D. is continuing

Câu 160 :
Which of the following could be considered a cause of the distress signals of trees?

A. attacks by insects

B. experiments by scientists

C. torn roots

D. lack of water

Câu 161 :
Which of the following is the main topic of the passage?

A. The effect of insects on trees.

B. The vibrations produced by insects.

C. The mission of the U.S Forest Service.

D. The sounds made by trees.

Câu 163 :
It can be inferred from the passage that the sounds produced by the trees ...............

A. are the same no matter what type of tree produces them

B. cannot be heard by the unaided human ear

C. fall into the 1-20 kilohertz

D. serve as a form of communication among trees

Câu 164 :
The word “parched” in paragraph 1 is closest in meaning to ............

A. dehydrated

B. recovered

C. damaged

D. burned

Câu 165 :
Read the following passage and mark the letter A, B, C or D on your answer sheet to indicate the correct answer to each of the questions
Choosing a career may be one of the hardest jobs you ever have, and it must be done with care. View a career as an opportunity to do something you love, not simply as a way to earn a living. Investing the time and effort to thoroughly explore your options can mean the difference between finding a stimulating and rewarding career and move from job to unsatisfying job in an attempt to find the right one. Work influences virtually every aspect of your life, from your choice of friends to where you live. Here are just a few of the factors to consider. Deciding what matters most to you is essential to making the right decision. You may want to begin by assessing your likes, dislikes, strengths, and weaknesses. Think about the classes, hobbies, and surroundings that you find most appealing. Ask yourself questions, such as “Would you like to travel? Do you want to work with children? Are you more suited to solitary or cooperative work?” There are no right or wrong answers; only you know what is important to you. Determine which job features you require, which ones you would prefer, and which ones you cannot accept. Then rank them in order of importance to you. The setting of the job is one factor to take into account. You may not want to sit at a desk all day. If not, there are diversity occupation – building inspector, supervisor, real estate agent – that involve a great deal of time away from the office. Geographical location may be a concern, and employment in some fields is concentrated in certain regions. Advertising job can generally be found only in large cities. On the other hand, many industries such as hospitality, law education, and retail sales are found in all regions of the country. If a high salary is important to you, do not judge a career by its starting wages. Many jobs, such as insurance sales, offers relatively low starting salaries; however, pay substantially increases along with your experience, additional training, promotions and commission. Don’t rule out any occupation without learning more about it. Some industries evoke positive or negative associations. The traveling life of a flight attendant appears glamorous, while that of a plumber does not. Remember that many jobs are not what they appear to be at first, and may have merits or demerits that are less obvious. Flight attendants must work long, grueling hours without sleeps, whereas plumbers can be as highly paid as some doctors. Another point to consider is that as you mature, you will likely develop new interests and skills that may point the way to new opportunities. The choice you make today need not be your final one. 
The author states that “There are no right or wrong answers” in order to ............

A. indicate that the answers are not really important.

B. show that answering the questions is a long and difficult process.

C. emphasize that each person’s answers will be different.

D. indicate that each person’s answers may change over time.

Câu 166 :
The word “them” in paragraph 2 refers to .............

A. answers

B. questions

C. features

D. jobs

Câu 167 :
The word “assessing” in paragraph 2 could best be replaced by ...........

A. measuring

B. disposing

C. discovering

D. considering

Câu 169 :
In paragraph 5, the author suggests that ..........

A. you may want to change careers at some time in the future.

B. you will be at your job for a lifetime, so choose carefully.

C. as you get older, your career will probably less fulfilling.

D. you will probably jobless at some time in the future.

Câu 170 :
Why does the author mention “long, grueling hours without sleeps” in paragraph 4?

A. To contrast the reality of a flight attendant’s job with most people’s perception.

B. To emphasize the difficulty of working as a plumber.

C. To discourage readers from choosing a career as a flight attendant.

D. To show that people must work hard for the career they have chosen.

Câu 171 :
According to the passage, which of the following is true?

A. If you want an easy and glamorous lifestyle, you should consider becoming flight attendant.

B. Your initial view of certain careers may not be accurate.

C. To make lots of money, you should rule out all factory jobs.

D. To make a lot of money, you should not take a job with a low starting salary.

Câu 175 :
What percent of the total rainfall on the earth does the Amazon rain forests receive?

A. exactly 30 to 40

B. about 30 to 40

C. about 20 to 30

D. less than 30

Câu 176 :
According to the passage, what must the nations of the world do to find a solution?

A. work together

B. save their lives

C. work alone

D. save rain forests only

Câu 177 :
Read the following passage and mark the letter A, B, C, or D on your answer sheet to indicate the correct answer to each of the questions
THE PEOPLE BEHIND THE MUSIC Think for a moment about the last music album you bought. Most likely, you’ll think of the singer or band that made you want to buy the album. You might even know the name of the guitar player or the drummer. Those talented performers, however, are only some of the people in the music industry work behind the scenes, but the roles they play in the musical progress are very important. Songwriters: Songs begin with the songwriter. Some songwriters work alone, but many work in teams that combine the talents of a lyricist, who writes the words to songs, and an instrumentalist, often a piano player or guitarist, who writes the music. Many of today’s pop stars work with songwriters. For example, some of Lady Gaga’s biggest hits were written by Nadir Khayat, also known “RedOne”. Some songwriting teams have become very famous, such as Mike Stock, Matt Aitken, and Pete Waterman, who were responsible for many big 1980s pop hits. Arrangers: After a song has been written, music arrangers make it more appealing by deciding which instruments will be used, what tempo, or speed, the song will have, and whether the song should have a lower or higher pitch. A good arrangement can bring a song to life and make it a classic. Studio Musicians: Not every singer or instrumentalist can be a star, and many work in the background as studio musicians. These artists are not a part of any one musical group. Instead, they are hired for recording sessions that eventually become the albums you buy, as well as soundtracks for television shows, movies, and radio ads. Recording Engineers: Recording engineers also play a major role in creating the final sound that you hear. First, these engineers set up the recording studio, the room where the performers play, placing musicians and microphones in exactly the right places to get the best sound. Next, they use electronic equipment, such multi-track recorders, to capture the music. Finally, long after the musicians have gone home, recording engineers use a mixing board to balance the melodies and rhythms of each musician, and sometimes to incorporate special sound effects or additional tracks. Many people make a living with music. You may not recognize all of their names, but all of them work together to create the songs you love to listen to. 
Which piece of equipment is used at the end of the recording process?

A. a multi-track recorder

B. a guitar or piano

C. a mixing board

D. a microphone

Câu 178 :
Which sentence is NOT true about studio musicians?

A. They work in the recording studio.

B. They earn money for each session that they do.

C. They are not as famous as the artists they play for.

D. They usually support or play for the same artist.

Câu 179 :
Which of the following do music arrangers probably NOT do?

A. decide the price of the CD

B. decide how fast or slow a song will be

C. decide which instrument to use

D. decide which pitch to use

Câu 180 :
The word “classic” in paragraph 3 is closest in meaning to ............

A. a traditional song

B. a famous song

C. a lively song

D. a modern song

Câu 181 :
Which statement would the author probably agree with?

A. Studio musicians would be more successful as members of one musical group.

B. It is important to buy albums made by performing artists who are not yet famous.

C. Having a famous person sing a song will usually make it successful.

D. People are often unaware of the amount of work that goes into creating music

Câu 182 :
Why does the author mention Lady Gaga?

A. to give an example of a star who works with a songwriter.

B. to explain why she does not write her own songs.

C. to compare her with other talented songwriters.

D. to persuade readers to buy her music.

Câu 183 :
The article is mainly about.......... 

A. people who play a background role in creating music 

B. pop stars who write the biggest hits 

C. songwriting teams who combine their talents 

D. instrumentalists who work as hired musicians

Câu 184 :
Read the following passage and mark the letter A, B, C, or D on your answer sheet to indicate the correct answer to each of the questions
The Rise of Robots As kids, our grandparents frequently scared us by talking about how difficult life was when they were growing up. They mentioned walking miles to school in the snow, or doing hours of manual labour for little pay. Life has changed greatly since then, and it seems to get easier year by year. In fact, with the help of robots we soon might have to do much at all. But is this a good thing? By 2030, it’s estimated that robotics will be a $10 billion business worldwide. Companies are already starting to integrate them into the workforce. The electronics manufacturer Foxconn is drawing up plans to launch a factory within the next 10 years that’s completely staffed by robot workers. Meanwhile, an American company Briggo has invented a robot that serves gourmet- quality coffee to customers. With the push of a button it will crush coffee beans, measure exact quantities of water, and even wave a steam wand to ensure customers get the perfect cream on top. Unlike human baristas, it can serve multiple drinks at once and work all day and night without a break. Robots are also invading our homes. The Rooma is a mini-robot that vacuums rooms automatically according to a schedule. The Robomow is a device that will cut the grass for you while you sit in the shade. Then there’s the Nanda clocky, an alarm clock that makes sure that even the deepest sleepers get up on time. The clock is attached to a pair of wheels, and it will randomly move around the room. Once you finally catch it, you’re probably too awake to hit the snooze button. Although these early home robots are somewhat basic, they will likely become more capable as times goes on. Although robots certainly help us to eliminate tedious tasks, many people are concerned about a future filled with robots. Some fear that humanity will start to decline if machines do everything for us. Others have even warned about the robot rebellion, in which robots become so smart that they may decide to turn on their masters. These ideas may seem a bit far-fetched, but there are certainly lots of questions that need to be answered before everyone opens up to the idea of a robotic future. 
How is Briggo’s invention superior to human workers?

A. It is more knowledgeable about coffee-making.

B. It is better at conversation.

C. It never has to stop.

D. It can operate machinery

Câu 185 :
What would happen if you pressed the snooze button?

A. You would have to wake up immediately

B. The alarm clock would turn off forever

C. The alarm would stop, but go off again soon

D. Nothing would happen at all

Câu 186 :
Which of the following is NOT true about the concerns over roboties?

A. It takes some time for people to accept robots.

B. Robots may do some harm to humans.

C. The idea of robotics may seem far-fetched.

D. Robots may be dominant at the workplace.

Câu 187 :
The word “tedious” in the last paragraph mostly means

A. boring

B. difficult

C. intelligent

D. expensive

Câu 188 :
What is the last paragraph mainly about?

A. Some reasons why people don’t accept robots yet.

B. A very amusing science-fiction story about robots.

C. How people in local communities are supporting robots.

D. A few of the lastest robots on the market today.

Câu 189 :
According to the first paragraph, how is the life changed since our parents time?

A. The education system has got much worse

B. Things aren’t as hard as they once were

C. Children have to walk longer distance to school

D. It hasn’t changed much at all

Câu 190 :
All of the following are true about robots EXCEPT that ............

A. they can work all day and night without break

B. they have greater capabilities

C. they can draw up plans to launch a story

D. they can do boring tasks for human

Câu 191 :
We can infer from the passage that ..............

A. present domestic robots can work without any programs

B. the communication will decline with the use of robots

C. a robot rebellion can happen daily

D. robotics will have been a major business by 2030

Câu 192 :
Read the following passage and mark the letter A, B, C, or D on your answer sheet to indicate the correct answer to each of the questions
As a result of pollution, Lake Erie, on the borders of the USA and Canada, is now without any living things. Pollutions in water are not simply a matter of “poisons” killing large numbers of fish overnight. Very often the effects of pollution are not noticed for many months or years because the first organisms to be affected are either plants or plankton. But these organisms are the food of fish and birds and other creatures. When this food disappears, the fish and birds die in this way a whole food chain can be wiped out, and it is not until dead fish and water birds are seen at the river’s edge or on the sea shore that people realize what is happening. Where do the substances which pollute the water come from? There are two main sources – sewage and industrial waste. As more detergent is used in the home, so more of it is finally put into our rivers, lakes and seas. Detergents harm water birds dissolving the natural substance which keep their feather waterproof. Sewage itself, if it is not properly treated, makes the water dirty and prevents all forms of life in rivers and the sea from receiving the oxygen they need. Industrial waste is even more harmful since there are many high poisonous things in it, such as copper and lead. So, if we want to stop this pollution, the answer is simple, sewage and industrial waste must be made clean before flowing into the water. It may already be too late to save some rivers and lakes, but others can still be saved if the correct action is taken at once. 
According to the passage, the way to stop water pollution is ...........

A. to make the waste material harmless.

B. to realize the serious situation clearly.

C. to make special room in the sea for our rubbish.

D. to put oxygen into the river.

Câu 193 :
Which of the following is harmful according to the passage?

A. industrial waste

B. water for cleaning

C. Chemicals

D. All of the above

Câu 194 :
Pollution in water is noticed ............. 

A. when the first organisms is affected. 

B. as soon as the balance of nature is destroyed. 

C. when poisons are poured into water. 

D. when a good many fish and birds die.

Câu 196 :
What is the meaning of “waterproof” in the second paragraph?

A. not allowing water to go through

B. covered with water

C. full of water

D. cleaned by water

Câu 197 :
Read the following passage and mark the letter A, B, C, or D on your answer sheet to indicate the correct answer to each of the questions
How to protect children Web fans from unsuitable material online while encouraging them to use the Internet has long been discussed in the US. For some parents, the Internet can seem like a jungle, filled with danger for their children. But jungles contain wonders as well as hazard and with good guides, some education, and a few precautions, the wilds of the Internet can be safely navigated. “Kids have to be online. If we tell our kids they can’t be allowed to surf the Internet, we’re cutting them off from their future,” said an expert. Most kids have started to use search engines. Many of them are great for finding tons of interesting Internet sites, and they can also locate places where you might not want your kids to go. There are search engines designed just for kids. A certain software contains only sites that have been selected as safe. The most popular way would be to use what is known as a “content screener”. But this can’t be wholly reliable, and the best thing parents can do is to talk to their kids and let them know what is OK or not OK to see or do on the Internet. Another way is that mum or dad is nearby when the child is surfing the Internet. A few other tips as follows: Don’t put the PC in a child’s room but keep it in an area where mum or dad can keep an eye on things. That also makes the Internet more of a family activity. Ask your child what he or she has been doing and about any friends they make online. Tell your child not to give online strangers personal information, especially like address and phone number. And tell your children never to talk to anyone they meet on line over the phone, send them anything, accept anything from them or agree to meet with them unless you go along. 
Which of the following is right according to the passage?

A. Children’s not having access to Internet may have effect on their progress.

B. Searching engines can help children to select materials fit for them.

C. Using a content screener is most reliable for keeping children having access to Internet.

D. Surfing the Internet is the best method of educating children.

Câu 198 :
The passage is mainly about the subject of .............

A. nternet in America.

B. appreciating Internet.

C. opposing children’s on-line.

D. American children going on-line

Câu 199 :
According to the passage, we can infer that .........

A. Internet is a jungle full of danger

B. a child who is on-line is in danger

C. Internet contains a lot of harmful sites

D. soft wares fit for children want programming 

Câu 200 :
The best way to protect children from improper material is .............

A. to buy some search engines for the children

B. to talk to the children and persuade them to tell right from wrong

C. to be nearby when they are surfing the Internet

D. to install a content screener on the computer

Câu 201 :
How to protect children Web fans from unsuitable material online while encouraging them to use the Internet has long been discussed in the US. For some parents, the Internet can seem like a jungle, filled with danger for their children. But jungles contain wonders as well as hazard and with good guides, some education, and a few precautions, the wilds of the Internet can be safely navigated. “Kids have to be online. If we tell our kids they can’t be allowed to surf the Internet, we’re cutting them off from their future,” said an expert. Most kids have started to use search engines. Many of them are great for finding tons of interesting Internet sites, and they can also locate places where you might not want your kids to go. There are search engines designed just for kids. A certain software contains only sites that have been selected as safe. The most popular way would be to use what is known as a “content screener”. But this can’t be wholly reliable, and the best thing parents can do is to talk to their kids and let them know what is OK or not OK to see or do on the Internet. Another way is that mum or dad is nearby when the child is surfing the Internet. A few other tips as follows: Don’t put the PC in a child’s room but keep it in an area where mum or dad can keep an eye on things. That also makes the Internet more of a family activity. Ask your child what he or she has been doing and about any friends they make online. Tell your child not to give online strangers personal information, especially like address and phone number. And tell your children never to talk to anyone they meet on line over the phone, send them anything, accept anything from them or agree to meet with them unless you go along.
The word “hazard” in the passage means ................

A. peril

B. loss

C. luck

D. instruction

Câu 202 :
Mark the letter a, B, C, or D on your answer sheet to indicate the sentence that is closest meaning to each of the following questions or indicate the correct answer to each of them
Tsunami is a Japanese word which means harbor wave and is used as the scientific term for seismic sea wave generated by an undersea earthquake or possibly an undersea landslide or volcanic eruption. When the ocean floor is tilted or offset during an earthquake, a set of waves is created similar to the concentric waves generated by an object dropped into the wave. Most tsunamis originate along the Ring of Fire, a zone of volcanoes and seismic activity, 32.500 km long that encircles the Pacific Ocean. Since 1819, about 40 tsunami have struck the Hawaiian Islands. A tsunami can have wave lengths, or widths, of 100 to 200 km, and may travel hundreds of kilometres across the deep ocean, reaching speeds of about 725 to 800 kilometres an hour. Upon entering shallow coastal waters, the wave, which may have been only about half a metre high out at sea, suddenly grows rapidly. When the wave reaches the shore, it may be 15 metres high or more. Tsunamis have tremendous energy because of the great volume of water affected. They are capable of obliterating coastal settlements. Tsunami should not be confused with storm surges, which are domes of water that rise underneath hurricanes or cyclones and cause extensive coastal flooding when the storms reach land. Storm surges are particularly devastating if they occur at high tide. A cyclone and accompanying storm surge killed an estimated 500,000 people in Bangladesh in 1970. The tsunami which truck south and southeast Asia in late 2004 killed over 200 thousand people. 
What does the word concentric mean?

A. Wavy

B. Having many centres

C. Having a common centre

D. A ring

Câu 203 :
What will happen when an object is dropped into the water?

A. Volcanic eruption may be a consequence

B. Some concentric waves will be generated

C. There will be seismic activity

D. Earthquake may happen

Câu 204 :
What is the zone of volcanoes and seismic activity in the world called?

A. The concentric wave

B. The tsunami

C. The Pacific Ocean

D. The Ring of Fire

Câu 205 :
What is the greatest speed of tsunami traveling across the deep ocean?

A. 200 kilometres an hour

B. 700 kilometres an hour

C. 800 kilometres an hour

D. 150,000 kilometres an hour

Câu 206 :
How high is the wave of the tsunami when it reaches the shore?

A. 100 metres

B. 200 metres

C. half a metre

D. fifteen metres

Câu 207 :
How are tsunami capable of obliterating coastal settlements?

A. They have tremendous energy due to the great volume of water affected.

B. They are a metre high or more.

C. They travel hundreds of kilometers.

D. They can strike the shore fifteen metres high.

Câu 208 :
What killed an estimated 500,000 people in Bangladesh?

A. A tsunami.

B. A cyclone and accompanying storm surge.

C. A high tide.

D. flooding

Câu 209 :
Which of the following is NOT true?

A. Tsunami only occurs in Asia.

B. A cyclone along with storm surge happened in Asia in 1970.

C. Storm surges are domes of water rising underneath hurricanes or cyclones.

D. Storm surges causes extensive coastal flooding.

Câu 210 :
Mark the letter a, B, C, or D on your answer sheet to indicate the sentence that is closest meaning to each of the following questions or indicate the correct answer to each of them
It is estimated that over 99 percent of all species that ever existed have become extinct. What causes extinction? When a species is no longer adapted to a changed environment, it may perish. The exact causes of a species’ death vary from situation to situation. Rapid ecological change may render an environment hostile to a species. For example, temperatures may change and a species may not be able to adapt. Food resources may be affected by environmental changes, which will then cause problems for a species requiring these resources. Other species may become better adapted to an environment, resulting in competition and, ultimately, in the death of a species. The fossil record reveals that extinction has occurred throughout the history of Earth. Recent analyses have also revealed that on some occasions many species become extinct at the same time- a mass extinction. One of the best- known examples of mass extinction occurred 65 million years ago with the demise of dinosaurs and many other forms of life. Perhaps the largest mass extinction was the one that occurred 225 million years ago, when appropriately 95 percent of all species died. Mass extinction can be caused by a relatively rapid change in the environment and can be worsened by the close interrelationship of many species. If, for example, something were to happen to destroy much of the plankton in the oceans, then the oxygen content of Earth would drop, affection even organisms not living in the ocean. Such a change would probably lead to a mass extinction. One interesting, and controversial, finding is that extinctions during the past 250 million years have tended to be more intense every 26 million years. This periodic extinction might be due to intersection of the Earth’s orbit with a cloud of comets, but this theory is purely speculative. Some researchers have also speculated that extinction may often be random. That is, certain species may be eliminated and others may survive for no particular reason. A species’ survival may have nothing to do with its ability to adapt. If so, some of evolutionary history may reflect a sequence of essentially random events.
The underlined word “ultimately” is closest in meaning to .............

A. eventually

B. unfortunately

C. dramatically

D. exceptionally

Câu 211 :
What does the author say in paragraph 1 regarding most species in Earth’s history?

A. They are no longer in existence.

B. They have caused rapid change in the environment.

C. They have remained basically unchanged from their original forms.

D. They have been able to adapt to ecological changes.

Câu 212 :
Which of the following is NOT mentioned in paragraph 1 as resulting from repid ecological change?

A. Introduction of new species

B. Competition among species

C. Availability of food resources

D. Temperature changes

Câu 213 :
The underlined word “demise” is closest in meaning to .............

A. help

B. change

C. death

D. recovery

Câu 214 :
Why is “plankton” mentioned in the second paragraph?

A. To demonstrate the interdependence of different species.

B. To illustrate a comparison between organisms that live on the land and those that live in the ocean.

C. To emphasize the importance of food resources in preventing mass extinction.

D. To point out that certain species could never become extinct

Câu 215 :
According to paragraph 2, evidence from fossil suggests that ...............

A. extinction of species has occurred from time to time throughout Earth’s history.

B. dinosaurs became extinct much earlier than scientists originally believed.

C. extinctions on Earth have generally been massive.

D. there has been only one mass extinction in Earth’s history

Câu 216 :
According to the passage, it is believed that the largest extinction of a species occurred ...........

A. 250 million years ago

B. 225 million years ago

C. 65 million years ago

D. 26 million years ago

Câu 217 :
Read the following passage and mark the letter A, B, C, or D on your answer sheet to indicate the correct answer to each of the questions
Successful students often do the followings while studying. First they have an overview before reading. Next, they look for important information and pay greater attention to it (which often needs jumping forward or backward to process information). They also relate important points to one another. Also, they activate and use their prior knowledge. When they realize that their understanding is not good, they do not wait to change strategies. Last, they can monitor understanding and take action to correct or “ fix-up” mistakes in comprehension. Conversely, students with low academic achievement often demonstrate ineffective study skills. They tend to assume a passive role in learning and rely on others(e.g, teachers, parents) to monitor their studying. For example, low-achieving students often do not monitor their understanding of content, they may not be aware of the purpose of studying, and their show little evidence of looking back, or employing “fix-up” strategies to fix understanding problems. Students who struggle with learning new information seem to be unaware that they must extent beyond simply reading the content to understand and remember it. Children with learning disabilities do not plan and judge the quality off their studying. Their studying may be disorganized. Students with learning problems face challenges with personal organization as well. They often have difficulty keeping track of materials and assignments, following directions, and completing work on time. Unlike good student who employ a variety of study skills in a flexible yet purposeful manner, low-achieving students use a restricted range of skills. They can not explain why good study strategies are important for learning, and they tend to use the same, often ineffective, study approach for all learning tasks, ignoring task content, structure of difficulty. 
What is the topic of the passage?

A. Successful learners and their learning strategies.

B. Successful and low-academic achieving students.

C. Effective and ineffective ways of learning.

D. Study skills for high school students.

Câu 218 :
The word “prior” is CLOSET meaning to ...............

A. forward

B. earlier

C. important

D. good

Câu 219 :
The word “Conversely” is OPPOSITE meaning to ...............

A. Actually

B. Alternatively

C. Consequently

D. Similarity

Câu 220 :
According to the passage, what can be learnt about passive students?

A. They are slow in their studying.

B. They depend on other people to organize their learning.

C. They monitor their understanding.

D. The know the purpose of studying.

Câu 221 :
Which of the followings is NOT an evidence of monitoring studying?

A. Looking at their backs.

B. Monitoring their understanding of content.

C. Being aware of the purpose of studying.

D. Fixing up mistakes in the understanding.

Câu 222 :
According to the passage, to learn new information, low-achieving students do NOT .............

A. read it.

B. just understand it.

C. simply remember it.

D. relate it to what they have known.

Câu 223 :
In compared with low-achieving students, successful students use ...........

A. inflexible study ways.

B. various study skills.

C. restricted strategies.

D. aimless studying techniques.

Câu 224 :
The underlined pronoun “they”refers to ...............

A. study strategies.

B. low-achieving students.

C. study skills.

D. good students.

Câu 225 :
According to the passage, what does “slip away” mean?

A. pick up

B. break down

C. fall behind

D. pass quickly

Câu 227 :
According to the passage, why do many people never seem to have enough time to accomplish things?

A. They fail to deal with trivial matters.

B. They get tied down by one difficult problem.

C. They do not prioritize tasks.

D. They do not seek the advice of time management experts.

Câu 228 :
In paragraph 2, the word “those” refers to ...............

A. priorities

B. trivial matters

C. daily lists

D. people

Câu 229 :
The passage states that one solution to time management problem is to ..............

A. accomplish time – consuming matters first.

B. consult a time management.

C. spend only a short time on each task.

D. keep daily lists of priorities and check them regularly.

Câu 230 :
In the paragraph 2, the word “accomplishments” can be best replaced by ..............

A. decisions

B. priorities

C. assessments

D. achievements

Câu 231 :
In the paragraph following the passage most probably discusses ..............

A. another solution to time management problems.

B. mental and physical health problems.

C. different types of lists.

D. ways to achieve a sense of fulfillment.

Câu 232 :
Read the following passage and mark the letter A, B, C, or D on your answer sheet to indicate the correct answer to each ofthe questions
American movies create myths about college life in the United States. These stories are entertaining, but they are not true. You have to look beyond Hollywood movies to understand what college is really like. Thanks to the movies, many people believe that college students party and socialize more than they study. Movies almost never show students working hard in class or in the library. Instead, movies show them eating, talking, hanging out, or dancing to loud music at wild parties. While it is true that American students have the freedom to participate in activities, they also have academic responsibilities. In order to succeed, they have to attend classes and study hard. Another movie myth is that athletics is the only important extracurricular activity. In fact, there is a wide variety of nonacademic activities on campus such as special clubs, service organizations, art, and theater programs. This variety allows students to choose what interests them. Even more important, after graduation, students' résumés look better to employers if they list a few extracurricular activities. Most students in the movies can easily afford higher education. If only this were true! While it is true that some American college students are wealthy, most are from families with moderate incomes. Up to 80% of them get some type of financial aid. Students from middle and lower-income families often work part-time throughout their college years. There is one thing that many college students have in common, but it is not something you will see in the movies. They have parents who think higher education is a priority, a necessary and important part of their children's lives. Movies about college life usually have characters that are extreme in some way: super athletic, super intelligent, super wealthy, super glamorous, etc. Movies use these stereotypes, along with other myths of romance and adventure because audiences like going to movies that include these elements. Of course, real college students are not like movie characters at all. So the next time you want a taste of the college experience, do not go to the movies. Look at some college websites or brochures instead. Take a walk around your local college campus. Visit a few classes. True, you may not be able to see the same people or exciting action you will see in the movies, but you can be sure that there are plenty of academic adventures going on all around you. 
The story about college life in Amerian movies are not ...............

A. true

B. interesting

C. boring

D. exciting

Câu 233 :
The phrase “academic responsibilities” in the second paragraph is closest in meaning to .............

A. learning duties

B. training skills

C. caring professions

D. teaching methods

Câu 234 :
Which of the following is NOT true?

A. Learning is only part of students’ college life.

B. There is a wide choice of extracurricular activities for college students.

C. Extracurricular activities are of no importance to employers.

D. Not all extracurricular activities are students' academic responsibilities.

Câu 235 :
The word “they” in the third paragraph refers to ..............

A. employers

B. activities

C. colleges

D. résumés

Câu 236 :
The word “moderate” in the fourth paragraph is closest in meaning to .............

A. not high

B. unlimited

C. not steady

D. sensible

Câu 237 :
Many American students have to work part-time throughout their college years because ....................

A. they are not allowed to work full-time.

B. they want to gain experience.

C. their parents force them to.

D. they can earn money for their expenses.

Câu 238 :
Which of the following could best serve as the title of the passage?

A. Going to College: The Only Way to Succeed in Life.

B. Hollywood Movies: The Best About College Life.

C. Extracurricular Activities and Job Opportunities.

D. American College Life and the Movies.

Câu 239 :
Read the following passage and mark the letter A, B, C, or D on your answer sheet to indicate the correct answer to each of the questions
The need for a surgical operation, especially an emergency operation, almost always comes as a severe shock to the patient and his family. Despite modern advances, most people still have an irrational fear of hospitals and anaesthetics. Patients do riot often believe they really need surgery- cutting into a part of the body as opposed to treatment with drugs. In the early year of the 20 century, there was little specialization in surgery. A good surgeon was capable of performing almost every operation that had been advised up to that time. Today the situation is different. Operations are now being carried out that were not even dreamed of fifty years ago. The heart can be safely opened and its valves repaired. Cloyed blood vessels can be clean out, and broken ones mended and replaced. A lung, the whole stomach, or even part of the brain can be removed and still permit the patient to live a comfortable and satisfactory life. However, not every surgeon wants to, or is qualified to carry out every type of modern operation. The scope of surgery has increase remarkable in the past decades. Its safety has increased, too. Deaths from most operations are about 20% of what they were in 1910 and surgery has been extended in many directions, for example, to certain types of birth defects in new born babies, and at the other end of the scale, to life-saving operation for the octogenarian .The hospital stay after surgery has been shortened to as little as a week for most major operations. Most patients are out of bed on the day after an operation and may be back at work in two or three weeks. One of the most revolutionary areas of modern surgery is that of organ transplants. Until a few decades ago, no person, except an identical twins, was able to accept into his body the tissues of another person without reacting against them and eventually killing them. Recently, however, it has been discovered that with the use of X-rays and special drugs, it is possible to graft tissues from one person to another which will survive for periods of a year or more. Kidneys have been successfully transplanted between non-identical twins. Heart and lung transplants have also been reasonably successful. “Spare parts” surgery, the simple routine replacement of all worn-out organs by new ones, is still a dream of the future but surgery is ready for such miracles. In the meantime, you can be happy if the doctors say to you: “Yes, l think it is possible to operate on you for this condition”. 
Most people are afraid of being operated on ............

A. in spite of improvements in modem surgery.

B. because they think modern drugs are dangerous.

C. because they do not believe they need anesthetics.

D. unless it is an emergency operation.

Câu 240 :
Surgeons in the early 20 century compared with modern ones ............

A. had less to learn about surgery.

B. needed more knowledge.

C. were more trusted by their patients.

D. could perform every operation known today.

Câu 241 :
A patient can still live a comfortable life even after the removal of ..........

A. his brain.

B. a major organ such as the stomach or one lung.

C. his lungs.

D. part of the stomach or the whole liver.

Câu 242 :
The word “clogged” is most likely to correspond to ............

A. clean

B. blocked

C. covered

D. unwashed

Câu 243 :
Today, compared with 1910 ..............

A. 20% fewer of all operation patients die.

B. 20% of all operation patients recover.

C. operation deaths have increased by 20%.

D. five times fewer patients die after being operated on.

Câu 244 :
What is the similar meaning to the word “octogenarian”?

A. eighteen-year-old.

B. person in his eighties.

C. person having eighth operation.

D. eye specialist.

Câu 245 :
The main difficulty with organ transplants is .............

A. it is difficult to find organs of the same size.

B. only identical twins can give permission for their organs to be exchanged.

C. the body's tendency to reject alien tissues.

D. the patient is not allowed to use drugs after them.

Câu 246 :
You can be happy if your surgeon can operate because it means .................

A. he thinks your condition may be curable.

B. he is a good doctor.

C. he knows you will survive.

D. you are getting better already.

Câu 247 :
Read the following passage and mark the letter A, B, C, or D on your answer sheet to indicate the correct answer to each of the questions
Productivity Hacks for Great Success No matter who we are or where we are from, we only have 24 hours each day to get things done. Some people seem to make the most of their time, using it to achieve their goals and dreams. Others feel that life is passing them by and they aren’t accomplishing anything. If you are in the latter group, it is not too late to turn things around. By implementing a few simple hacks, you can start getting better results very quickly. If you want to become more productive, it pays to learn about the Pareto principle (which is also known as the 80-20 rule). It states that for most tasks, 80 percent of the results we get come from 20 percent of the work we do. For example. 80 percent of the company sales usually come from 20 percent of customers. Therefore, the trick is to focus your efforts on the key 20 percent of actions that truly matter to greatly improve your results. In some cases, people may be busy from morning to night but still seem to get nothing done. In this situation, the problem might be that clear goals haven’t been set. To fix this, set some time aside to list the things you want to achieve in life. Then, select the top three or four that you want to focus on for the next year. Look at your goals every night before bedtime and ask yourself what are the next actions to accomplish each day. Then, promise yourself that you will finish these tasks no matter how busy you get. If possible try completing the tasks first thing in the morning so you are assured of success. Another common mistake regarding productivity is not considering our energy levels. If we try to work around the clock, we will eventually get tired and quit. Everyone needs time to sleep, eat, socialize with friends, and unwind from the stress of the day. To make sure you don’t burn out, be sure to schedule some time for breaks and fun activities into your schedule every day. Finally, no matter how difficult your goals may seem, remember the old saying that “a journey of a thousand miles begins with a single step”. 
What is implied in the last sentence of the passage?

A. It is best to take a big first step.

B. Start moving toward your dreams little by little.

C. Think twice before taking any steps.

D. People get their best ideas when traveling.

Câu 248 :
In order to achieve our goals, we should ...............

A. select the most important goals for each day and take the priority to achieve them within the day

B. eat a lot to burn enough energy for our daily activities.

C. set the goals just before bedtime in order to remember them better.

D. set as many goals as possible so that we can achieve some of them or at least 20 percent.

Câu 249 :
Which of the following statements does the author support most?

A. How difficult our goals may seem, we should take steps to travel a thousand miles to achieve them.

B. Reasonable goals and good techniques of time management may help you get great achievements.

C. We try to work around the clock and set some time aside to list the things we want to achieve in life.

D. We need time to sleep, eat, socialize with friends, and achieve our goals for 20 percent of the day’s time.

Câu 250 :
The word “this” in paragraph 3 refers to ...............

A. a lack of time

B. setting no aims

C. clear goals

D. failing in business

Câu 251 :
Productivity Hacks for Great Success No matter who we are or where we are from, we only have 24 hours each day to get things done. Some people seem to make the most of their time, using it to achieve their goals and dreams. Others feel that life is passing them by and they aren’t accomplishing anything. If you are in the latter group, it is not too late to turn things around. By implementing a few simple hacks, you can start getting better results very quickly. If you want to become more productive, it pays to learn about the Pareto principle (which is also known as the 80-20 rule). It states that for most tasks, 80 percent of the results we get come from 20 percent of the work we do. For example. 80 percent of the company sales usually come from 20 percent of customers. Therefore, the trick is to focus your efforts on the key 20 percent of actions that truly matter to greatly improve your results. In some cases, people may be busy from morning to night but still seem to get nothing done. In this situation, the problem might be that clear goals haven’t been set. To fix this, set some time aside to list the things you want to achieve in life. Then, select the top three or four that you want to focus on for the next year. Look at your goals every night before bedtime and ask yourself what are the next actions to accomplish each day. Then, promise yourself that you will finish these tasks no matter how busy you get. If possible try completing the tasks first thing in the morning so you are assured of success. Another common mistake regarding productivity is not considering our energy levels. If we try to work around the clock, we will eventually get tired and quit. Everyone needs time to sleep, eat, socialize with friends, and unwind from the stress of the day. To make sure you don’t burn out, be sure to schedule some time for breaks and fun activities into your schedule every day. Finally, no matter how difficult your goals may seem, remember the old saying that “a journey of a thousand miles begins with a single step”.
The phrase “hacks for” in the title is closest in meaning to ..........

A. leads to

B. cut off

C. secretly achieve

D. kicks without control

Câu 252 :
Which of the following sentences is true?

A. No one ever achieves all of their goals.

B. Everyone has the same time in a day.

C. No one has any spare time these days.

D. Everyone manages time the same way.

Câu 253 :
What is the Pareto principle?

A. If you work hard 80 percent of the time, you can relax for 20 percent.

B. You need to do lots of work to increase sales by 20 percent.

C. A few important things produce most of the results.

D. Companies don’t need to hire more than 80 people.

Câu 254 :
Read the following passage and mark the letter A, B, C, or D on your answer sheet to indicate the correct answer to each of the questions
Carbon dating can be used to estimate the age of any organic natural material; it has been used successfully in archeology to determine the age of ancient artifacts or fossils as well as in a variety of other fields. The principle underlying the use of carbon dating is that carbon is a part of all living things on Earth. Since a radioactive substance such as carbon-14 has a known half-life, the amount of carbon-14 remaining in an object can be used to date that object. Carbon-14 has a half-life of 5,570 years, which means that after that number of years, half of the carbon- 14 atoms have decayed into nitrogen-14. It is the ratio of carbon-14 in that substance that indicates the age of the substance. If, for example, in a particular sample the amount of carbon- 14 is roughly equivalent to the amount of nitrogen-14, this indicates that around half of the carbon- 14 has decayed into nitrogen-14, and the sample is approximately 5,570 years old. Carbon dating cannot be used effectively in dating objects that are older than 80,000 years. When objects are that old, much of the carbon-14 has already decayed into nitrogen-14, and the molecule amount that is left doesn’t provide a reliable measurement of age. In the case of older objects, other age-dating methods are available, methods which use radioactive atoms with longer half-lives than carbon has. 
This passage is mainly about ............

A. archeology and the study of ancient artifacts.

B. one method of dating old objects.

C. various uses for carbon.

D. the differences between carbon-14 and nitrogen-14.

Câu 255 :
The word “it” in paragraph 1 refers to ..............

A. carbon dating

B. the age

C. any organic natural material

D. archeology

Câu 256 :
Read the following passage and mark the letter A, B, C, or D on your answer sheet to indicate the correct answer to each of the questions
William Sydney Porter (1862-1910), who wrote under the pseudonym of O. Henry, was born in North Carolina. His only formal education was to attend his Aunt Lina’s school until the age of fifteen, where he developed his lifelong love of books. By 1881 he was a licensed pharmacist. However, within a year, on the recommendation of a medical colleague of his Father’s, Porter moved to La Salle County in Texas for two years herding sheep. During this time, Webster’s Unabridged Dictionary was his constant companion, and Porter gained a knowledge of ranch life that he later incorporated into many of his short stories. He then moved to Austin for three years, and during this time the first recorded use of his pseudonym appeared, allegedly derived from his habit of calling “Oh, Henry” to a family cat. In 1887, Porter married Athol Estes. He worked as a draftsman, then as a bank teller for the First National Bank. In 1894 Porter founded his own humor weekly, the “Rolling Stone”, a venture that failed within a year, and later wrote a column for the Houston Daily Post. In the meantime, the First National Bank was examined, and the subsequent indictment of 1886 stated that Porter had embezzled funds. Porter then fled to New Orleans, and later to Honduras, leaving his wife and child in Austin. He returned in 1897 because of his wife’s continued ill-health, however she died six months later. Then, in 1898 Porter was found guilty and sentenced to five years imprisonment in Ohio. At the age of thirty five, he entered prison as a defeated man; he had lost his job, his home, his wife, and finally his freedom. He emerged from prison three years later, reborn as O. Henry, the pseudonym he now used to hide his true identity. He wrote at least twelve stories in jail, and after re-gaining his freedom, went to New York City, where he published more than 300 stories and gained fame as America’s favorite short Story writer. Porter married again in 1907, but after months of poor health, he died in New York City at the age of forty-eight in 1910. O. Henry’s stories have been translated all over the world.
According to the passage, Porter’s Father was ___________.

A. the person who gave him a life-long love of books

B. a medical doctor

C. a licensed pharmacist

D. responsible for his move to La Salle County in Texas

Câu 257 :
Which of the following is NOT true about carbon-14?

A. It and nitrogen always exist in equal amounts in any substance.

B. Its half-life is more than 5,000 years.

C. It can decay into nitrogen-14.

D. It is radioactive.

Câu 258 :
The word “imprisonment” in paragraph 2 is closet in meaning to _________.

A. captivity

B. escape

C. insult

D. punishment

Câu 259 :
Why did the author write the passage?

A. to outline the career of a famous American

B. because of his fame as America’s favorite short story writer

C. because it is a tragic story of a gifted writer

D. to outline the influences on O. Henry’s writing

Câu 260 :
What is the passage primarily about?

A. The life and career of William Sydney Porter.

B. The way to adopt a nickname.

C. O.Henry’s influence on American literature.

D. The adventures of O.Henry.

Câu 261 :
The author implies which of the following is true?

A. Porter’s wife might have lived longer if he had not left her in Austin when he fled.

B. Porter was in poor health throughout his life.

C. O. Henry is as popular in many other countries as he is in America.

D. Porter would probably have written less stories if he had not been in prison for three years.

Câu 262 :
The word “pseudonym” in the passage refers to ___________.

A. William Sydney Porter

B. O. Henry

C. Athol Estes

D. the Aunt Lina

Câu 263 :
Which of the following is true, according to the passage?

A. Porter left school at 15 to become a pharmacist.

B. Porter wrote a column for the Houston Daily Post called “Rolling Stone”.

C. The first recorded use of his pseudonym was in Austin.

D. Both of Porter’s wives died before he died.

Câu 264 :
The word “vary” in paragraph 6 can be best replaced by ___________.

A. decrease

B. change

C. differ

D. increase

Câu 265 :
Read the following passage and mark the letter A, B, C, or D on your answer sheet to indicate the correct answer to each of the questions
Contrary to popular belief, one does not have to be a trained programmer to work online. Of course, there are plenty of jobs available for people with high-tech computer skills, but the growth of new media has opened up a wide range of Internet career opportunities requiring only a minimal level of technical expertise. Probably one of the most well-known online job opportunities is the job of Webmaster. However, it is hard to define one basic job description for this position. The qualifications and responsibilities depend on what tasks a particular organization needs a Webmaster to perform. To specify the job description of a Webmaster, one needs to identify the hardware and software the website the Webmaster will manage is running on. Different types of hardware and software require different skill sets to manage them. Another key factor is whether the website will be running internally or externally (renting shared space on the company servers). Finally, the responsibilities of a webmaster also depend on whether he or she will be working independently, or whether the firm will provide people to help. All of these factors need to be considered before one can create an accurate webmaster job description. Webmaster is one type of Internet career requiring in-depth knowledge of the latest computer applications. However, there are also online jobs available for which traditional skills remain in high demand. Content jobs require excellent writing skills and a good sense of the web as a “new media”. The term “new media” is difficult to define because it compasses a constantly growing set of new technologies and skills. Specifically, it includes websites, email, internet technology, CD-ROM, DVD, streaming audio and video, interactive multimedia presentations, ebooks, digital music, computer illustration, video games, virtual reality, and computer artistry. Additionally, many of today’s Internet careers are becoming paid-bythe-job professions. With many companies having to downsize in tough economic items, the outsourcing and contracting of freelance workers online has become common business practice. The Internet provides an infinite pool of buyers from around the world with whom freelancers can contract their services. An added benefit to such online jobs is that freelancers are able to work on projects with companies outside their own country. How much can a person make in these kinds of careers? As with many questions related to today’s evolving technology, there is no simple answer. There are many companies willing to pay people with Internet skills salaries well over $70,000 a year. Generally, webmasters start at about $30,000 per year, but salaries can vary greatly. Freelance writers working online have been known to make between $40,000 to $70,000 a year. 
Which of the followings is true about the job of the freelance writers?

A. They may work with others in the company.

B. It is considered a “content” job.

C. They manage hardware and software.

D. They require in-depth knowledge of applications.

Câu 266 :
It can be inferred from the passage that ___________.

A. only skilled workers make good money

B. online workers can work full-time online

C. it is easy to become a webmaster

D. workers with limited computer skills can not work online

Câu 267 :
According to the passage, which of the following is TRUE of webmasters?

A. They never work independently.

B. The duties they perform depend on the organization they work for.

C. They require a minimal level of expertise.

D. They do not support software products.

Câu 268 :
What is the purpose of the passage?

A. To inform people about the tasks and the roles of a webmaster.

B. To inform people about employment related to the Internet.

C. To inform people about the computer industry.

D. To explain why webmasters make a lot of money.

Câu 269 :
According to the passage, all of the followings are TRUE except ___________.

A. Webmasters must have knowledge of the latest computer applications.

B. Online workers can not free themselves from the office.

C. “New media” is not easy to define.

D. There are online jobs available for workers with minimal computer skills.

Câu 270 :
Which of the followings is not mentioned as part of the “new media”?

A. Internet technology

B. writing skills

C. websites

D. video games

Câu 271 :
The word “identify” in paragraph 2 is closest in meaning to___________.

A. name

B. discover

C. encounter

D. estimate

Câu 272 :
Read the following passage and mark the letter A, B, C, or D on your answer sheet to indicate the correct answer to each of the questions
Centuries ago, man discovered that removing moisture from food helped to preserve it, and that the easiest way to do this was to expose the food to sun and wind. In this way the North American Indians produced pemmican (dried meat ground into powder and made into cakes), the Scandinavians made stockfish and the Arabs dried dates and apricots. All foods contain water - cabbage and other leaf vegetables contain as much as 93% water, potatoes and other root vegetables 80%, lean meat 75% and fish anything from 80% to 60% depending on how fatty it is. If this water is removed, the activity of the bacteria which cause food to go bad is checked. Fruit is sun-dried in Asia Minor, Greece, Spain and other Mediterranean countries, and also in California, South Africa and Australia. The methods used vary, but in general the fruit is spread out on trays in drying yards in the hot sun. In order to prevent darkening, pears, peaches and apricots are exposed to the fumes of burning sulphur before drying. Plums for making prunes, and certain varieties of grapes for making raisins and currants, are dipped in an alkaline solution in order to crack the skins of the fruit slightly and remove their wax coating, so increasing the rate of drying. Nowadays most foods are dried mechanically; the conventional method of such dehydration is to put food in chambers through which hot air is blown at temperatures of about 110oC at entry to about 45oC at exit. This is the usual method for drying such things as vegetables, minced meat, and fish. Liquids such as milk, coffee, tea, soups and eggs may be dried by pouring them over a heated horizontal steel cylinder or by spraying them into a chamber through which a current of hot air passes. In the first case, the dried material is scraped off the roller as a thin film which is then broken up into small, though still relatively coarse flakes. In the second process it falls to the bottom of the chamber as a fine powder. Where recognizable pieces of meat and vegetables are required, as in soup, the ingredients are dried separately and then mixed. Dried foods take up less room and weigh less than the same food packed in cans or frozen, and they do not need to be stored in special conditions. For these reasons they are invaluable to climbers, explorers and soldiers in battle, who have little storage space. They are also popular with housewives because it takes so little time to cook them. 
What is the main idea of the passage?

A. Advantages of dried foods.

B. Water: the main component of food.

C. Mechanization of drying foods.

D. Different methods of drying foods.

Câu 273 :
The phrase “do this” in the first paragraph mostly means ______.

A. expose foods to sun and wind

B. remove moisture from foods

C. produce pemmican

D. moisten foods

Câu 274 :
The word “checked” in the second paragraph is closest in meaning to ______.

A. reduced considerably

B. put a tick

C. examined carefully

D. motivated to develop

Câu 275 :
In the process of drying certain kinds of fruits, sulphur fumes help ______.

A. remove their wax coating

B. kill off bacteria

C. maintain their color

D. crack their skin

Câu 276 :
What does the word “which” in the fourth paragraph refer to?

A. Vegetables

B. Foods

C. Things

D. Chambers

Câu 277 :
Nowadays the common method for drying vegetables and minced meat is ______.

A. spreading them out on trays in drying yards

B. dipping them in an alkaline solution

C. putting them in chambers and blowing hot air through

D. pouring them over a heated horizontal steel cylinder

Câu 278 :
Which of the following is NOT mentioned in the passage?

A. Liquids are not dried in the same way as fruits and vegetables.

B. Dried foods have several advantages over canned or frozen foods.

C. Fruit is usually dried by being laid out on trays in the sun.

D. People in India began to use drying methods centuries ago.

Câu 279 :
The final product of the process of drying liquids that uses the first method will be ______.

A. small flakes

B. fine powder

C. dried soup

D. recognizable pieces

Câu 281 :
Which of the following helps to create the Earth’s magnetic fields?

A. Its blue waters

B. Its nitrogen atmosphere

C. Its molten metal core

D. The moon

Câu 282 :
What two factors help protect the Earth from radiation?

A. Magnetic field and atmosphere

B. Blue waters and white clouds

C. Rapid spin and molten nickel-iron core

D. The Sun and the Moon

Câu 283 :
The word consisting most nearly means ________.

A. hardening

B. withholding

C. containing

D. shortening

Câu 284 :
The main idea of this passage is that ________.

A. Earth is predominantly water.

B. There are life-supporting characteristics on Earth

C. Earth is the only planet with a moon

D. Earth has no common characteristics with other planets

Câu 285 :
The word distinguishing as it is used in this selection means ________.

A. elevating in nature

B. characteristics like all other planets

C. devastating in nature

D. characteristics that set it apart from other planets

Câu 286 :
It’s probable that the next paragraph would discuss ________.

A. people on planets

B. the solar system as a whole

C. rings around Saturn

D. the Earth’s natural satellite – the moon

Câu 287 :
Read the following passage and mark the letter A, B, C, or D on your answer sheet to indicate the correct answer to each of the questions
Which of the follow
The White House, the official home of the United States president, was not built in time for George Washington to live in it. It was begun in 1792 and was ready for its first inhabitants. President and Mrs.John Adams, who moved in on November 1, 1800. When the Adamses moved in, the White House was not yet complete, and the Adamses suffered many inconveniences; for example, the main staircase was incomplete, which hindered movement from floor to floor, and the future laundry yard was merely a pool of mud, so wet laundry was hung in the unfinished East Room to dry. Thomas Jefferson, the third president, improved the comfort of the White House in many respects and added new architectural features such as the terraces on the east and west ends. When the British forces burned the White House on August 24, 1814, President Madison was forced to leave.All the remained after the fire was the exterior walls, the interior was completely destroyed. It was not until December of 1817 that the following president, James Monroe, was able to move into a rebuilt residence. Since then, the White House has continued to be modified but has been continuously occupied by each succeeding U.S president. ing would be the most appropriate title for this text?

A. George Washington’s life in the White House.

B. The Early History of the White House.

C. The burning of the White House.

D. Presidential Policies of Early U.S.Presidents.

Câu 288 :
Why did George Washington not live in the White House?

A. It had been burned by the British.

B. He did not like the architectural features.

C. He did not want to suffer the inconveniences that the Adamses had suffered.

D. Construction had not yet been completed.

Câu 289 :
The word “inhabitants” in line 2 is closest meaning to: .........

A. modifications

B. moves

C. celebrations

D. residents

Câu 290 :
It can be inferred from the passage that John Adams was:

A. the first president of the United States.

B. the second president of the United States.

C. the third president of the United States.

D. the fourth president of the United States.

Câu 291 :
What of the White House was not yet complete when the Adamses moved in?

A. main staircase

B. laundry yard

C. pool

D. A and B

Câu 292 :
The word “forces” in line 9 could best be replaced by:

A. military

B. effort

C. power

D. energy

Câu 293 :
According to the passage, when James Monroe came to the White House, it had been .......

A. repressed

B. reconstructed

C. relocated

D. reserved

Câu 294 :
The paragraph following the passage most likely discusses:

A. the details of the destruction of the White House by the British

B. James Monroe’s policies as presidents.

C. modifications by presidents who followed.

D. other presidents who were unable to occupy the White House.

Câu 295 :
Read the following passage and mark the letter A, B, C, or D on your answer sheet to indicate the correct answer to each of the questions
Family life in the United States is changing. Thirty or forty years ago, the wife was called a "housewife". She cleaned, cooked, and cared for the children. The husband earned the money for the family. He was usually out working all day. He came home tired in the evening, so he did not do much housework. And he did not see the children very much, except on weekends. These days, however, more and more women work outside the home. They cannot stay with the children all day. They, too, come home tired in the evening. They do not want to spend the evening cooking dinner and cleaning up. They do not have time to clean the house and do the laundry. So who is going to do the housework now? Who is going to take care of the children? Many families solve the problem of housework by sharing it. In these families, the husband and wife agree to do different jobs around the house, or they take turns doing each job. For example, the husband always cooks dinner and the wife always does the laundry. Or the wife cooks dinner on some nights and the husband cooks dinner on other nights. Then there is a question of the children. In the past, many families got help with child care from grandparents. Now families usually do not live near their relatives. The grandparents often are too far away to help in a regular way. More often, parents have to pay for child care help. The help may be a babysister or a day-care center. The problem with this kind of help is the high cost. It is possible only for couples with jobs that pay well. Parents may get another kind of help from the companies they work for. Many companies now let people with children work part-time. That way, parents can spend more time with their children. Some husbands may even stop working for a while to stay with the children. For these men there is a new word: They are called "househusband". In the United States more ans more men are becoming househusband every year. These changes in the home mean changes in the family. Fathers can learn to understand their children better, and the children can get to know their fathers better. Husbands and wives may also find changes in their marriage. They, too, may have a better understanding of each other. 
These changes in the Amercan home may ______________

A. not happen

B. not change the children at all.

C. help families.

D. cause problems for a marriage.

Câu 296 :
It can be inferred from paragraph 4 that__________.

A. couples with low-paid jobs can’t afford the cost of a babysitter or a day-care center.

B. in the past, grandparents did not help the couples with child care.

C. all couples with jobs can pay for help from a babysitter or a day-care center.

D. grandparents can help care the children in a regular way.

Câu 297 :
Sixty years ago, most women ____________.

A. had no children.

B. were housewives.

C. went out to work.

D. did not do much housework.

Câu 298 :
Nowadays there are ____________. 

A. more and more women staying with the children all day. 

B. more work outside the home before. 

C. more housewives than before. 

D. more women going out to work than before.

Câu 299 :
This article is about ________.

A. how more American women are working. 

B. how family life in America is changing. 

C. American men as househusbands.

D. housewives in America.

Câu 300 :
The word “they” in paragraph 5 refers to _______________.

A. children who spend more time with fathers than mothers.

B. husbands who stop working to stay with the children.

C. parents who work part-time.

D. fathers who spend more time with their children.

Câu 301 :
Family life in the United States is changing. Thirty or forty years ago, the wife was called a "housewife". She cleaned, cooked, and cared for the children. The husband earned the money for the family. He was usually out working all day. He came home tired in the evening, so he did not do much housework. And he did not see the children very much, except on weekends. These days, however, more and more women work outside the home. They cannot stay with the children all day. They, too, come home tired in the evening. They do not want to spend the evening cooking dinner and cleaning up. They do not have time to clean the house and do the laundry. So who is going to do the housework now? Who is going to take care of the children? Many families solve the problem of housework by sharing it. In these families, the husband and wife agree to do different jobs around the house, or they take turns doing each job. For example, the husband always cooks dinner and the wife always does the laundry. Or the wife cooks dinner on some nights and the husband cooks dinner on other nights. Then there is a question of the children. In the past, many families got help with child care from grandparents. Now families usually do not live near their relatives. The grandparents often are too far away to help in a regular way. More often, parents have to pay for child care help. The help may be a babysister or a day-care center. The problem with this kind of help is the high cost. It is possible only for couples with jobs that pay well. Parents may get another kind of help from the companies they work for. Many companies now let people with children work part-time. That way, parents can spend more time with their children. Some husbands may even stop working for a while to stay with the children. For these men there is a new word: They are called "househusband". In the United States more ans more men are becoming househusband every year. These changes in the home mean changes in the family. Fathers can learn to understand their children better, and the children can get to know their fathers better. Husbands and wives may also find changes in their marriage. They, too, may have a better understanding of each other.
The word “laundry” in paragraph 2 is closest in meaning to ___________.

A. washing and ironing.

B. tidying up.

C. cooking and washing up.

D. shopping.

Câu 302 :
Read the following passage and mark the letter A, B, C, or D on your answer sheet to indicate the correct answer to each of the questions
Martin Luther King, Jf., is well- known for his work in civil rights and for his many famous speeches, among which is his moving “I have a dream” speech. But fewer people know much about King’s childhood. M.L, as he was called, was born in 1929 in Atlanta, Georgia, at the home of his maternal grandfather. M.L.’s grandfather purchased their home on Auburn Avenue in 1909, twenty years before M.L was born. His grandfather allowed the house to be used as a meeting place for a number of organizations dedicated to the education and social advancement of blacks. M.L. grew up in the atmosphere, with his home being used as a community gathering place, and was no doubt influenced by it. M.L.’s childhood was not especially eventfully. His father was a minister and his mother was a musician. He was the second of three children, and he attended all black schools in a black neighborhood. The neighborhood was not poor, however. Auburn Avenue was an area of banks, insurance companies, builders, jewelers, tailors, doctors, lawyers, and other businesses and services. Even in the face of Atlanta’s segregation, the district thrived. Dr. King never forgot the community spirit he had known as a child, nor did he forget the racial prejudice that was a huge barrier keeping black Atlantans from mingling with whites.
What is the passage mainly about?

A. The prejudice that existed in Atlanta.

B. M.L.’s grandfather

C. Martin Luther King’s childhood.

D. The neighborhood King grew up in.

Câu 303 :
When was M.L. born? 

A. in 1909 

B. in 1929

C. in 1949 

D. 20 years after his parents had met.

Câu 304 :
What is Martin Luthur King well- known for?

A. His publications.

B. His neighborhood.

C. His childhood.

D. His work in civil rights

Câu 305 :
According to the author, M.L. _______.

A. had a difficult childhood.

B. was a good musician as a child.

C. loved to listen to his grandfather speak.

D. grew up in a relatively rich area of Atlanta.

Câu 306 :
Which of the following statements is NOT true?

A. Auburn was a commercial areas.

B. M.L.’s grandfather built their home on Auburn Avenue in 1909.

C. M. L. grew up in a rich, black neighborhood.

D. M.L.’s childhood was uneventful.

Câu 308 :
M.L. was _______ by the atmosphere in which he grew up.

A. not affected at all

B. doubted

C. certainly influenced

D. prejudiced

Câu 309 :
Read the following passage and mark the letter A, B, C, or D on your answer sheet to indicate the correct answer to each of the questions
For centuries, time was measured by the position of the sun with the use of sundials. Noon was recognized when the sun was the highest in the sky, and cities would set their clock by this apparent solar time, even though some cities would often be on a slightly different time. Daylight Saving Time (DST), sometimes called summer time, was instituted to make better use of daylight. Thus, clocks are set forward one hour in the spring to move an hour of daylight from the morning to the evening and then set back one hour in the fall to return to normal daylight. Benjamin Franklin first conceived the idea of daylight saving during his tenure as an American delegate in Paris in 1984 and wrote about it extensively in his essay, "An Economical Project." It is said that Franklin awoke early one morning and was surprised to see the sunlight at such an hour. Always the economist, Franklin believed the practice of moving the time could save on the use of candlelight, as candles were expensive at the time. In England, builder William Willett (1857–1915) became a strong supporter for Daylight Saving Time upon noticing blinds of many houses were closed on an early sunny morning. Willet believed everyone, including himself, would appreciate longer hours of light in the evenings. In 1909, Sir Robert Pearce introduced a bill in the House of Commons to make it obligatory to adjust the clocks. A bill was drafted and introduced into Parliament several times but met with great opposition, mostly from farmers. Eventually, in 1925, it was decided that summer time should begin on the day following the third Saturday in April and close after the first Saturday in October. The U.S. Congress passed the Standard Time Act of 1918 to establish standard time and preserve and set Daylight Saving Time across the continent. This act also devised five time zones throughout the United States: Eastern, Central, Mountain, Pacific, and Alaska. The first time zone was set on "the mean astronomical time of the seventy- fifth degree of longitude west from Greenwich" (England). In 1919, this act was repealed. President Roosevelt established year-round Daylight Saving Time (also called War Time) from 1942–1945. However, after this period, each state adopted its own DST, which proved to be disconcerting to television and radio broadcasting and transportation. In 1966, President Lyndon Johnson created the Department of Transportation and signed the Uniform Time Act. As a result, the Department of Transportation was given the responsibility for the time laws. During the oil embargo and energy crisis of the 1970s, President Richard Nixon extended DST through the Daylight Saving Time Energy Act of 1973 to conserve energy further. This law was modified in 1986, and Daylight Saving Time was reset to begin on the first Sunday in April (to spring ahead) and end on the last Sunday in October (to fall back). 
The word “it” in paragraph 2 refers to ............

A. Franklin’s idea of daylight saving

B. Franklin's first conception

C. Franklin’s first official tenure

D. Franklin’s delegation

Câu 310 :
The word “obligatory” in paragraph 3 is closest in meaning to .........

A. imperative

B. approved

C. deficient

D. peculiar

Câu 311 :
Who opposed the bill that was introduced in the House of Commons in the early 1900s?

A. Sir Robert Pearce

B. television and radio broadcasting companies

C. farmers

D. the U.S. Congress

Câu 312 :
The word “devised” in paragraph 4 is closest in meaning to ........

A. divided

B. invented

C. ordered

D. adapted

Câu 313 :
Which of the following statements is true of the U.S. Department of Transportation?

A. It was created by President Richard Nixon.

B. It set standards for DST throughout the world.

C. It constructed the Uniform Time Act.

D. It oversees all time laws in the United States.

Câu 314 :
The Daylight Saving Time Energy Act of 1973 was responsible for ..

A. extending Daylight Saving Time in the interest of energy conservation

B. preserving and setting Daylight Saving Time across the continent

C. instituting five time zones in the United States

D. conserving energy by giving the Department of Transportation authority over time laws

Câu 315 :
Which of the following would be the best title for this passage?

A. Lyndon Johnson and the Uniform Time Act

B. The History and Rationale of Daylight Saving Time

C. The U.S. Department of Transportation and Daylight Saving Time

D. Daylight Saving Time in the United States

Câu 316 :
Read the following passage and mark the letter A, B, C, or D on your answer sheet to indicate the correct answer to each of the questions
Animation traditionally is done by hand-drawing or painting successive frame of an object, each slightly different than the proceeding frame. In computer animation, although the computer may be the one to draw the different frames, in most cases the artist will draw the beginning and ending frames and the computer will produce the drawings between the first and the last drawing. This is generally referred to as computer-assisted animation, because the computer is more of a helper than an originator. In full computer animation, complex mathematical formulas are used to produce the final sequences of pictures. These formulas operate on extensive databases of numbers that defines the objects in the pictures as they exist in mathematical space. The database consists of endpoints, and color and intensity information. Highly trained professionals are needed to produce such effects because animation that obtains high degrees of realism involves computer techniques from three-dimensional transformation, shading, and curvatures. High-tech computer animation for film involves very expensive computer systems along with special color terminals or frame buffers. The frame buffer is nothing more than a giant image memory for viewing a single frame. It temporarily holds the image for display on the screen. A camera can be used to film directly from the computer’s display screen, but for the highest quality images possible, expensive film recorders are used. The computer computers the positions and colors for the figures in the picture, and sends this information to the recorder, which captures it on film. Sometimes, however, the images are stored on a large magnetic disk before being sent to the recorder. Once this process is completed, it is replaced for the next frame. When the entire sequence has been recorded on the film, the film must be developed before the animation can be viewed. If the entire sequence does not seem right, the motions must be corrected, recomputed, redisplayed, and rerecorded. This approach can be very expensive and time – consuming. Often, computer-animation companies first do motion tests with simple computer-generated line drawings before selling their computers to the task of calculating the high-resolution, realistic-looking images. 
Which of the following statement is supported by the passage?

A. Computers have reduced the costs of animation.

B. In the future, traditional artists will no longer be needed.

C. Artists are unable to produce drawings as high in quality as computer drawings.

D. Animation involves a wide range of technical and artistic skills

Câu 317 :
The word “they” in the second paragraph refers to ______.

A. formulas

B. objects

C. numbers

D. database

Câu 318 :
According to the passage, the frame buffers mentioned in the third paragraph are used to ........

A. add color to the images

B. expose several frames at the same time

C. store individual images

D. create new frames

Câu 319 :
According to the passage, the positions and colors of the figures in high-tech animation are determined by ..........

A. drawing several versions

B. enlarging one frame at a lime

C. analyzing the sequence from different angles

D. using computer calculations

Câu 320 :
The word “captures” in the fourth paragraph is closest in meaning to ______.

A. separates

B. registers

C. describes

D. numbers

Câu 322 :
According to the passage, how do computer-animation companies often test motion?

A. They experiment with computer-generated line drawings.

B. They hand-draw successive frames.

C. They calculate high-resolutions images.

D. They develop extensive mathematical formulas.

Câu 324 :
Read the following passage and mark the letter A, B, C, or D on your answer sheet to indicate the correct answer to each of the questions
They call Jamaica the "Island in the sun" and that is my memory of it. Of sunshine, warmth and abundant fruit that was growing everywhere, and of love. There were two sisters ahead of me in the family, and though of course I didn't know it, there was an exciting talk of emigration, possibly to Canada but more usually to England, the land of opportunity. I guess that plans were already being made when I was born, for a year or so later my Dad left for London. Two years after that my mum went as well and my sisters and I were left in the care of my grandmother. Emigrating to better yourself was a dream for most Jamaicans, a dream many were determined to fulfill. Families were close and grandmothers were an important part of the family. So, when the mass emigrations began, it seemed perfectly right and natural for them to take over the running of families left behind. Grandmothers are often strict, but usually also spoil you. She ran the family like a military operation: each of us, no matter how young, had our tasks. Every morning, before we went to school, we all had to take a bucket appropriate to our size and run a relay from the communal tap to the barrels until they are full. My sisters had to sweep the yard before they went to school. My grandmother would give orders to the eldest and these were passed down- as I got older I found this particularly annoying! But I can tell you, no one avoided their duties. My Dad came over from England to see how we were getting on . He talked to us about the new country, about snow, about the huge city, and we all wanted to know more, to see what it was like. I didn't know it at that time., but he had come to prepare us for the move to England. Six months later my grandmother told me that I was going to join my parents and that she, too, was emigrating. London was strange and disappointing. There was no gold on the pavements, as the stories in Jaimaica had indicated. The roads were busy, the buildings were grey and dull, with many tall, high-rise blocks. It was totally unlike Jamaica, the houses all small and packed close together. In my grandmother's house I had a big bedroom, here I had to share. Then came the biggest shock: snow. While flakes came out of the sky and Dad smiled, pointed and said: "That's snow!" I rushed outside, looked up and opened my mouth to let the flakes drop in. The snow settled on my tongue and it was so cold that I cried. My toes lost all feeling. As my shoes and socks got wet and frozen, there came an excruciating pain and I cried with the intensity of it. I didn't know what was happening to me. 
The writer says that when he was very young ________.

A. he was upset because his parents left

B. he was very keen to go to England

C. his parents had decided to leave

D. his parents changed their plans

Câu 325 :
According to the writer, many people from Jamaica at that time ________.

A. wanted to be free from responsibility

B. wanted to improve their standard of living

C. had ambitions that were unrealistic

D. dislike the country they came from

Câu 326 :
What does "this" in the third paragraph refer to .............

A. being told what to do by his sisters

B. having to sweep the yard before school

C. having to do duties he found difficult

D. being given orders by his grandmother

Câu 327 :
What happened when the writer's father came?

A. His father didn't tell him why he had come

B. He didn't know how to react to his father

C. His father told him things that were untrue

D. He felt eager about what his father told him

Câu 328 :
When the writer first went to London, he was disappointed because ________.

A. it was smaller than he expected

B. he had been given a false impression of it

C. he had to spend a lot of time on his own

D. his new surroundings frightened him

Câu 329 :
The word "excruciating" in the last paragraph means ________.

A. painful

B. rather painful

C. extremely painful

D. painless

Câu 330 :
Which of the following would be the best title for this passage?

A. From Sun to Snow

B. A strange childhood

C. Hard times

D. Too many changes

Câu 331 :
Read the following passage and mark the letter A, B, C, or D on your answer sheet to indicate the correct answer to each of the questions
We get great pleasure from reading. The more advanced a man is, the greater delight he will find in reading. The ordinary man may think that subjects like philosophy or science are very difficult and that if philosophers and scientists read these subjects, it is not for pleasure. But this is not true. The mathematician finds the same pleasure in his mathematics as the school boy in an adventure story. For both, it is a play of the imagination, a mental recreation and exercise. The pleasure derived from this activity is common to all kinds of reading. But different types of books give us different types of pleasure. First in order of popularity is novel-reading. Novels contain pictures of imaginary people in imaginary situations, and give us an opportunity of escaping into a new world very much like our world and yet different from it. Here we seem to live a new life, and the experience of this new life gives us a thrill of pleasure. Next in order of popularity are travel books, biographies and memoirs. These tell us tales of places we have not seen and of great men in whom we are interested. Some of these books are as wonderful as novels, and they have an added value that they are true. Such books give us knowledge, and we also find immense pleasure in knowing details of lands we have not seen and of great men we have only heard of. Reading is one of the greatest enjoyments of life. To book-lovers, nothing is more fascinating than a favorite book. And, the ordinary educated man who is interested and absorbed in his daily occupation wants to occasionally escape from his drudgery into the wonderland of books for recreation and refreshment. 
What does the passage mainly discuss?

A. Different types of books

B. Different kinds of reading

C. Reading as an exercise for the brain

D. Reading as a pleasurable activity

Câu 332 :
According to paragraph 1, which of the following is NOT true?

A. Ordinary people may think that philosophy and science are difficult.

B. Reading about mathematics is mentally entertaining for a mathematician.

C. Philosophers and scientists do not read for pleasure.

D. A more advanced person takes greater pleasure in reading

Câu 333 :
The word “derived” in paragraph 2 is closest in meaning to ______

A. differed

B. established

C. bought

D. obtaine

Câu 334 :
The word “it” in paragraph 2 refers to ______.

A. a new life

B. our world

C. an opportunity

D. a thrill of pleasure

Câu 335 :
The word “immense” in paragraph 3 is closest in meaning to ______.

A. great

B. limited

C. personal

D. controlled

Câu 336 :
According to paragraph 4, which of the following is the most fascinating to booklovers?

A. A daily occupation

B. An ordinary educated man

C. The wonderland

D. A favorite book

Câu 337 :
According to the passage, travel books, biographies and memoirs ______.

A. are wonderful novels

B. tell stories of well-known places

C. are less popular than novels

D. are more valuable than novels

Câu 338 :
Read the following passage and mark the letter A, B, C, or D on your answer sheet to indicate the correct answer to each of the questions
The system of higher education had its origin in Europe in the Middle Ages, when the first universities were established. In modern times, the nature of higher education around the world, to some extent, has been determined by the models of influential countries such as France and Germany. Both France and Germany have systems of higher education that are basically administered by state agencies. Entrance requirements for students are also similar in both countries. In France, an examination called the baccalauréat is given at the end of secondary education. Higher education in France is free and open to all students who have passed this baccalauréat. Success in this examination allows students to continue their higher education for another three or four years until they have attained the first university degree called a licence in France. Basic differences, however, distinguish these two countries’ systems. French educational districts, called académies, are under the direction of a rector, an appointee of the national government who is also in charge of universities in each district. The uniformity in curriculum throughout the country leaves each university with little to distinguish itself. Hence, many students prefer to go to Paris, where there are better accommodations and more cultural amenities for them. Another difference is the existence in France of prestigious higher educational institutions known as grandes écoles, which provide advanced professional and technical training. Most of these schools are not affiliated with the universities, although they too recruit their students by giving competitive examinations to candidates. The grandes écoles provide rigorous training in all branches of applied science and technology, and their diplomas have a somewhat higher standing than the ordinary licence. In Germany, the regional universities have autonomy in determining their curriculum under the direction of rectors elected from within. Students in Germany change universities according to their interests and the strengths of each university. In fact, it is a custom for students to attend two, three, or even four different universities in the course of their undergraduate studies, and the majority of professors at a particular university may have taught in four or five others. This high degree of mobility means that schemes of study and examination are marked by a freedom and individuality unknown in France. France and Germany have greatly influenced higher education systems around the world. The French, either through colonial influence or the work of missionaries, introduced many aspects of their system in other countries. The German were the first to stress the importance of universities as research facilities, and they also created a sense of them as emblems of a national mind. 
What does the passage mainly discuss?

A. The nature of education around the world in modern times.

B. Systems of higher education in France and Germany.

C. The origin of higher education system in Europe

D. The influence of France and Germany on educational systems of other countries

Câu 339 :
The word “uniformity” in paragraph 3 is closest in meaning to ______.

A. proximity

B. discrepancy

C. similarity

D. uniqueness

Câu 340 :
The word “their” in paragraph 3 refers to ______.

A. schools

B. universities

C. examinations

D. branches

Câu 341 :
Which of the following about grandes écoles in France is NOT stated in paragraph 3?

A. Most of them have no connection with universities.

B. They have a reputation for advanced professional and technical training.

C. Their degrees are better recognized than those provided by universities.

D. They offer better accommodations and facilities than universities.

Câu 342 :
According to the passage, a regional university rector in Germany is elected by _____.

A. the staff of the university

B. the national government officials

C. the regional government officials

D. the staff of other universities

Câu 343 :
According to paragraph 4, what makes it possible for students in Germany to attend different universities during their undergraduate studies?

A. The university staff have become far more mobile and occupied.

B. The university’s training programs offer greater flexibility and freedom of choice.

C. University tuition fees are kept at an affordable level for all students.

D. Entry requirements to universities in Germany are made less demanding.

Câu 344 :
The word “emblems” in the final paragraph is closest in meaning to ______.

A. representatives

B. directions

C. structures

D. delegates

Câu 345 :
Which of the following can be inferred from the passage?

A. Studying in France and Germany is a good choice for people of all ages and nationalities.

B. It normally takes longer to complete a university course in France than in Germany.

C. Universities in Germany can govern themselves more effectively than those in France.

D. The level of decentralization of higher education is greater in Germany than in France.

Câu 346 :
Read the following passage and mark the letter A, B, C, or D on your answer sheet to indicate the correct answer to each of the questions
Different cultures follow their own special customs when a child's baby teeth fall out. In Korea, for example, they have the custom of throwing lost teeth up on the roof of a house. According to tradition, a magpie will come and take the tooth. Later, the magpie will return with a new tooth for the child. In other Asian countries, such as Japan and Vietnam, children follow a similar tradition of throwing their lost teeth onto the roofs of houses. Birds aren't the only animals thought to take lost teeth. In Mexico and Spain, tradition says a mouse takes a lost tooth and leaves some money. However, in Mongolia, dogs are responsible for taking teeth away. Dogs are highly respected in Mongolian culture and are considered guardian angels of the people. Tradition says that the new tooth will grow good and strong if the baby tooth is fed to a guardian angel. Accordingly, parents in Mongolia will put their child's lost tooth in a piece of meat and feed it to a dog. The idea of giving lost teeth to an angel or fairy is also a tradition in the West. Many children in Western countries count on the Tooth Fairy to leave money or presents in exchange for a tooth. The exact origins of the Tooth Fairy are a mystery, although the story probably began in England or Ireland centuries ago. According to tradition, a child puts a lost tooth under his or her pillow before going to bed. In the wee hours, while the child is sleeping, the Tooth Fairy takes the tooth and leaves something else under the pillow. In France, the Tooth Fairy leaves a small gift. In the United States, however, the Tooth Fairy usually leaves money. These days, the rate is $1 to $5 per tooth, adding up to a lot of money from the Tooth Fairy! 
What is the passage mainly about?

A. Presents for young children's lost teeth

B. Animals eating children's lost teeth

C. Customs concerning children's new teeth

D. Traditions concerning children's lost teeth

Câu 347 :
The word "their" in paragraph 1 refers to ______.

A. houses'

B. children's

C. roofs'

D. countries

Câu 348 :
According to the passage, where is a child's lost tooth thought to be taken away by a mouse?

A. In Mongolia

B. In Mexico and Spain

C. In Japan and Vietnam

D. In Korea

Câu 349 :
According to paragraph 2, parents in Mongolia feed their child's lost tooth to a dog because ______.

A. they believe that this will make their child's new tooth good and strong

B. they hope that their child will get some gifts for his or her tooth

C. they know that dogs are very responsible animals

D. they think dogs like eating children's teeth

Câu 350 :
The word "origins" in paragraph 3 is closest in meaning to ______.

A. countries

B. stories

C. beginnings

D. families

Câu 351 :
Different cultures follow their own special customs when a child's baby teeth fall out. In Korea, for example, they have the custom of throwing lost teeth up on the roof of a house. According to tradition, a magpie will come and take the tooth. Later, the magpie will return with a new tooth for the child. In other Asian countries, such as Japan and Vietnam, children follow a similar tradition of throwing their lost teeth onto the roofs of houses. Birds aren't the only animals thought to take lost teeth. In Mexico and Spain, tradition says a mouse takes a lost tooth and leaves some money. However, in Mongolia, dogs are responsible for taking teeth away. Dogs are highly respected in Mongolian culture and are considered guardian angels of the people. Tradition says that the new tooth will grow good and strong if the baby tooth is fed to a guardian angel. Accordingly, parents in Mongolia will put their child's lost tooth in a piece of meat and feed it to a dog. The idea of giving lost teeth to an angel or fairy is also a tradition in the West. Many children in Western countries count on the Tooth Fairy to leave money or presents in exchange for a tooth. The exact origins of the Tooth Fairy are a mystery, although the story probably began in England or Ireland centuries ago. According to tradition, a child puts a lost tooth under his or her pillow before going to bed. In the wee hours, while the child is sleeping, the Tooth Fairy takes the tooth and leaves something else under the pillow. In France, the Tooth Fairy leaves a small gift. In the United States, however, the Tooth Fairy usually leaves money. These days, the rate is $1 to $5 per tooth, adding up to a lot of money from the Tooth Fairy!
According to the passage, which of the following is NOT true about the tradition of tooth giving in the West?

A. Children give money to the Tooth Fairy.

B. Children hope to get money or gifts from the Tooth Fairy.

C. Lost teeth are traditionally given to an angel or fairy.

D. Children put their lost teeth under their pillows.

Câu 352 :
The phrase "the wee hours" in paragraph 3 probably refers to the period of time _____.

A. early in the evening

B. late in the morning

C. long before bedtime

D. soon after midnight

Câu 353 :
Read the following passage and mark the letter A, B, C, or D on your answer sheet to indicate the correct answer to each of the questions
Earth is richly supplied with different types of living organisms which co-exist in their environments, forming complex, interrelated communities. Living organisms depend on one another for nutrients, shelter, and other benefits. The extinction of one species can set off a chain reaction that affects many other species, particularly if the loss occurs near the bottom of the food chain. For example, the extinction of a particular insect or plant might seem inconsequential. However, there may be fish or small animals that depend on that resource for foodstuffs. The loss can threaten the survival of these creatures and larger predators that prey upon them. Extinction can have a ripple effect that spreads throughout nature. In addition to its biological consequences, extinction poses a moral dilemma for humans, the only species capable of saving the others. The presence of humans on the planet has affected all other life forms, particularly plants and animals. Human lifestyles have proven to be incompatible with the survival of some other species. Purposeful efforts have been made to eliminate animals that prey on people, livestock, crops, or pose any threat to human livelihoods. Some wild animals have been decimated by human desire for meat, hides, fur, or other body parts with commercial value. Likewise, demand for land, water, and other natural resources has left many wild plants and animals with little to no suitable habitat. Humans have also affected nature by introducing non-native species to local areas and producing pollutants having a negative impact on the environment. The combination of these human-related effects and natural obstacles such as disease or low birthrates has proven to be too much for some species to overcome. They have no chance of survival without human help. As a result, societies have difficult choices to make about the amount of effort and money they are willing to spend to save imperiled species. Will people accept limits on their property rights, recreational activities, and means of livelihood to save a plant or an animal? Should saving such popular species as whales and dolphins take priority over saving obscure, annoying, or fearful species? Is it the responsibility of humans to save every kind of life form from disappearing, or is extinction an inevitable part of nature, in which the strong survive and the weak perish? These are some difficult questions that people face as they ponder the fate of other species living on this planet. 
What does the passage mainly discuss?

A. Humans and endangered species

B. The importance of living organisms

C. Measures to protect endangered species

D. Causes of animal extinction

Câu 354 :
The word "inconsequential" in paragraph 1 is closest in meaning to ______.

A. unimportant

B. unavoidable

C. unexpected

D. unrecognizable

Câu 355 :
Which of the following can result from the loss of one species in a food chain?

A. The connections among the creatures in the food chain become closer.

B. Larger predators will look for other types of prey.

C. There might be a lack of food resources for some other species.

D. Animals will shift to another food chain in the community

Câu 356 :
The word "They" in paragraph 2 refers to ______.

A. human-related effects

B. some species

C. low birthrates

D. natural obstacles

Câu 357 :
In paragraph 2, non-native species are mentioned as ______.

A. a kind of harmless animals

B. an achievement of human beings

C. a harmful factor to the environment

D. a kind of useful plants

Câu 358 :
The word "perish" in paragraph 3 is closest in meaning to ______.

A. develop

B. complete

C. remain

D. disappear

Câu 359 :
According to the passage, which of the following statements is NOT true?

A. Humans have difficult choices to make about saving endangered species.

B. The existence of humans is at the expense of some other species.

C. No other species can threaten the survival of humans on Earth.

D. Some animals and plants cannot survive without human help.

Câu 360 :
Which of the following can be inferred from the passage?

A. Animal and plant species which pose threats to humans will die out soon.

B. Saving popular animal and plant species should be given a high priority.

C. Humans will make changes in their lifestyles to save other species.

D. The dilemma humans face between maintaining their lives and saving other species remains.

Câu 361 :
The text is mainly about________________ 

A. the importance of teachers.

B. the advantages and disadvantages of being a teacher in the USA. 

C. the difference of teaching career. 

D. the basic requirements of being a teacher in the USA.

Câu 362 :
According to the text, future post graduates should start working as teachers _______

A. after gaining their master's degree.

B. before studying for their master's degree

C. during the time they are studying for their master's degree.

D. before studying for their bachelor's degree.

Câu 363 :
The word “vital” in paragraph 2 is closest in meaning to______

A. very useless

B. very easy

C. very important

D. very interesting

Câu 364 :
According to the text, teaching requires a combination of many things EXCEPT ________.

A. qualifications

B. personal qualities

C. experience

D. appearance

Câu 365 :
According to the text, the most important quality of a teacher is _______.

A. being patient

B. being to work hard

C. being prepared

D. being a role model

Câu 366 :
The word “they” in paragraph 3 refers to ___________.

A. postgraduates

B. novices

C. schools

D. teachers

Câu 367 :
According to the text, all of the following sentences are true EXCEPT _____.

A. Those who want to become university teachers need master's degrees.

B. A teachers needs to be aware of the fact that learning can sometimes be hard work.

C. A great number of schools in the USA want to hire novices with little or no classroom experience.

D. In the USA, before one takes over a class, a training program is typically necessary to be completed.

Câu 368 :
Read the following passage and mark the letter A, B, C, or D on your answer sheet to indicate the correct answer to each of the questions
Before the mid-nineteenth century, people in the United States ate most foods only in season. Drying, smoking and salting could preserve meat for a short time, but the availability of fresh meat, like that of fresh milk, was very limited; there was no way to prevent spoilage. But in 1810, a French inventor named Nicolas Appert developed the cooking-and-sealing process of canning. And in the 1850’s an American named Gail Borden developed a means of condensing and preserving milk. Canned goods and condensed milk became more common during the 1860’s, but supplies remained low because cans had to be made by hand. By 1880, however, inventors had fashioned stamping and soldering machines that massproduced cans from tinplate. Suddenly all kinds of food could be preserved and bought at all times of the year. Other trends and inventions had also helped make it possible for Americans to vary their daily diets. Growing urban population created demand that encouraged fruit and vegetable farmers to raise more produce. Railroad refrigerator cars enabled growers and meat packers to ship perishables great distances and to preserve them for longer periods. Thus, by the 1890’s, northern city dwellers could enjoy southern and western strawberries, grapes, and tomatoes, previously available for a month at most, for up to six months of the year. In addition, increased use of iceboxes enabled families to store perishables. As easy means of producing ice commercially had been invented in the 1870’s, and by 1900 the nation had more than two thousand commercial ice plants, most of which made home deliveries. The icebox became afixture in most homes and remained so until the mechanized refrigerator replaced it in the 1920’s and 1930’s. Almost everyone now had a more diversified diet. Some people continued to eat mainly foods that were heavily in starches or carbohydrates, and not everyone could afford meat. Nevertheless, many families could take advantage of previously unavailable fruits, vegetables, and dairy products to achieve more varied fare. 
What does the passage mainly discuss?

A. Causes of food spoilage.

B. Commercial production of ice.

C. Population movements in the nineteenth century.

D. Inventions that led to changes in the American diet.

Câu 369 :
The phrase “in season” in line 1 refers to ________ .

A. a particular time of year

B. a kind of weather

C. an official schedule

D. a method of flavoring

Câu 370 :
During the 1860’s, canned food products were _________ .

A. unavailable in rural areas

B. available in limited quantities

C. shipped in refrigerator cars

D. a staple part of the American diet

Câu 371 :
The word ”them” in line 12 refers to _________ .

A. refrigerator cars

B. growers

C. perishables

D. distances

Câu 372 :
The word ”fixture” in line 16 is closest in meaning to _________ .

A. commonplace object

B. substance

C. luxury item

D. mechanical device

Câu 373 :
The author implies that in the 1920’s and 1930’s home deliveries of ice _________ .

A. increased in cost

B. occurred only in the summer

C. decreased in number

D. were on an irregular schedule

Câu 374 :
The word “Nevertheless” in line 19 is closest meaning to _________ .

A. occasionally

B. however

C. therefore

D. Because

Câu 375 :
Which of the following types of food preservation was NOT mentioned in the passage?

A. Drying

B. Chemical additives

C. Canning

D. Cold storage

Câu 376 :
Read the following passage and mark the letter A, B, C, or D on your answer sheet to indicate the correct answer to each of the questions
On March 15, Dunes View Middle School held a contest for school bands. Student bands tried out for the opportunity to perform at the school picnic, which will be held at the end of June. The winner of the contest was the band called Four Square. “We’re very proud that we won the contest and are excited to perform at the picnic,” says Peter Zandt, who plays the guitar in the band. “And since we hope to perform someday at other local places, like restaurants and parks, this will be a great first step.” The contest was the creation of music teacher Mr. Lopez and drama teacher Ms. Cho. The two thought of the idea while discussing recent years’ school picnics. “The picnic is one of the biggest events of the year, but it has become a bit formulaic ,” said Ms. Cho. “The activities are the same every year. We thought that a performance by a student band would make the school picnic more interesting and fun.” Mr. Lopez, Ms. Cho, and three other teachers judged the contest, which took place in the gym. Eight student bands signed up to audition. The bands varied in their musical forms: there were several rock bands, a folk band, and even a jazz band. “I’m disappointed that my band didn’t win, but I think the judges made the right choice,” says student Marisol Varga, a member of the folk trio called The Bell Girls. “Four Square is really good.” To see if the bands could present a wide range of musical skills, the teachers asked them each to prepare two songs: one song with original words, and another in which students played instrumental music only. The judges finally chose the band Four Square as the winner of the contest. Four Square is a rock band with an unusual twist: it includes a violin player! The members of Four Square write their own songs and practice three times a week after school. Students and teachers agreed that the band competition was a big success. All are looking forward to the school picnic in June. 
Which would be the most appropriate headline for the article?

A. Famous Band Visits School

B. Teachers Give Music Lessons

C. Students Have Fun at School Picnic

D. School Holds Student Band Contest

Câu 377 :
What is probably true about school picnics in recent years?

A. They were held in March.

B. They were not very exciting.

C. They were very hard to organize

D. They included musical performances

Câu 378 :
The word formulaic is closest in meaning to___________.

A. expensive

B. uncertain

C. long

D. dull

Câu 379 :
What type of band is NOT mentioned in the article?

A. Pop

B. Jazz

C. Folk

D. Rock

Câu 380 :
According to the article, what is unusual about the band Four Square?

A. It is named after a popular children’s game.

B. Its members practice several times a week.

C. It is a rock band with a violinist.

D. Its members all play guitar.

Câu 381 :
Why did the teachers ask each band to play two songs?

A. To decide which songs to play at the picnic

B. To see if the bands could play a variety of songs

C. To make sure the contest did not go on too long

D. To make the concert more enjoyable for students

Câu 382 :
According to the article, what does the band Four Square hope to do in the future?

A. Perform at the school picnic every year

B. Perform in many different places

C. Buy new instruments

D. Record an album

Câu 383 :
Who will play music at the picnic this year?

A. Marisol Varga

B. Mr. Lopez

C. Peter Zandt

D. Ms. Cho

Câu 384 :
Read the following passage and mark the letter A, B, C, or D on your answer sheet to indicate the correct answer to each of the questions
Dodder is an unusual and unwanted plant that attacks other plants. Except for its flowers, the plant looks like spaghetti noodles. Its almost leafless, thread–like stems hang down atop other plants that dodder needs to stay alive. Dodder does not produce its own food. Instead, it steals it from other plants. It feeds by sucking juices from the plant it is wrapped around, often making its host very weak or even killing it. Dodder can find other plants by their smell. When a dodder seedling starts growing, it follows the scent of plants it prefers, like tomato plants, potato plants, or other farm crops. Unlike most plants that usually grow in the direction of light or warmth, a dodder plant will grow in the direction of, for example, tomato odor––if a tomato happens to be growing nearby. However, a young dodder plant must find a host plant quickly. If it cannot catch a whiff of a potential host within a few days, it will dry up and disappear—even if there is plenty of water around. Once it finds a host, the young dodder plant will attach itself to it and start growing faster. At that point, the dodder plant will drop its root. Dodder is thus a difficult weed to manage and a real headache for farmers. When it does get out of hand, dodder can greatly reduce a farmer’s harvest or even destroy crops completely. Before sowing their produce, farmers in warm parts of the world often check to make sure no unwanted dodder seeds have intermingled with their crop seeds. This is a good way to stop dodder plants from sneaking their way into a crop field. 
What is the main topic of the passage?

A. A new variety of farm crop

B. Plants that are harmful to humans

C. The special abilities of a dangerous plant

D. Recent improvements in farming methods

Câu 385 :
Why does the author mention spaghetti?

A. To analyze the content of some food

B. To describe the shape of a dodder plant

C. To explain where the dodder plant came from

D. To argue that dodder plants can be used to make food

Câu 386 :
The word it refers to___________. 

A. host 

B. food 

C. plant 

D. dodder

Câu 387 :
What causes dodder to grow in a particular direction?

A. Odor

B. Light

C. Water

D. Temperature

Câu 388 :
The expression catch a whiff is closest in meaning to________.

A. eat the seeds

B. notice the smell

C. find the location

D. determine the size

Câu 389 :
What will happen if a dodder plant starts growing where there are no other plants around?

A. It will soon die.

B. It will grow deeper roots.

C. It will attract other plants.

D. It will cover the entire area of soil.

Câu 390 :
The expression get out of hand is closest in meaning to_________.

A. dry up and die

B. change its color

C. become hard to find

D. grow uncontrollably

Câu 391 :
Read the following passage and mark the letter A, B, C, or D on your answer sheet to indicate the correct answer to each of the questions
It is commonly believed in the United States that school is where people go to get an education. Nevertheless, it has been said that today children interrupt their education to go to school. The distinction between schooling and education implied by this remark is important. Education is much more open-ended and all-inclusive than schooling. Education knows no bounds. It can take place anywhere, whether in the shower or on the job, whether in a kitchen or on a tractor. It includes both the formal learning that takes place in schools and the whole universe of informal learning. The agents of education can range from a revered grandparent to the people debating politics on the radio, from a child to a distinguished scientist. Whereas schooling has a certain predictability, education quite often produces surprises. A chance conversation with a stranger may lead a person to discover how little is known of other religions. People are engaged in education from infancy on. Education, then, is a very broad, inclusive term. It is a lifelong process, a process that starts long before the start of school, and one that should be an integral part of one's entire life. Schooling, on the other hand, is a specific, formalized process, whose general pattern varies little from one setting to the next. Throughout a country, children arrive at school at approximately the same time, take assigned seats, are taught by an adult, use similar textbooks, do homework, take exams, and so on. The slices of reality that are to be learned, whether they are the alphabet or an understanding of the workings of government, have usually been limited by the boundaries of the subject being taught. For example, high school students know that they are not likely to find out in their classes the truth about political problems in their communities or what the newest filmmakers are experimenting with. There are definite conditions surrounding the formalized process of schooling. 
What does the author probably mean by using the expression “children interrupt their education to go to school” (lines 2-3) ?

A. Going to several different schools is educationally beneficial.

B. School vacations interrupt the continuity of the school year.

C. Summer school makes the school year too long.

D. All of life is an education.

Câu 392 :
The word “bounds” in line 5 is closest in meaning to ......

A. rules

B. experience

C. limits

D. exceptions

Câu 393 :
The word “ integral” in line 13 is closest in meaning to ..........

A. equitable

B. profitable

C. pleasant

D. essential

Câu 394 :
The word “they” in line 17 refers to ...........

A. slices of reality

B. similar textbooks

C. boundaries

D. seats

Câu 395 :
The phrase “For example,” line 19, introduces a sentence that gives examples of ........

A. similar textbooks

B. the results of schooling

C. the workings of a government

D. the boundaries of classroom subjects

Câu 396 :
The passage supports which of the following conclusions?

A. Without formal education, people would remain ignorant.

B. Education systems need to be radically reformed.

C. Going to school is only part of how people become educated.

D. Education involves many years of professional training.

Câu 397 :
The passage is organized by .....

A. listing and discussing several educational problems 

B. contrasting the meanings of two related words 

C. narrating a story about excellent teachers 

D. giving examples of different kinds of schools

Câu 398 :
Read the following passage and mark the letter A, B, C, or D on your answer sheet to indicate the correct answer to each of the questions
Orchids are unique in having the most highly developed of all blossoms, in which the usual male and female reproductive organs are fused in a single structure called the column. The column is designed so that a single pollination will fertilize hundreds of thousands, and in some cases millions, of seeds, so microscopic and light they are easily carried by the breeze. Surrounding the column are three sepals and three petals, sometimes easily recognizable as such, often distorted into gorgeous, weird, but always functional shapes. The most noticeable of the petals is called the labellum, or lip. It is often dramatically marked as an unmistakable landing strip to attract the specific insect the orchid has chosen as its pollinator. To lure their pollinators from afar, orchids use appropriately intriguing shapes, colors, and scents. At least 50 different aromatic compounds have been analyzed in the orchid family, each blended to attract one, or at most a few, species of insects or birds. Some orchids even change their scents to interest different insects at different times. Once the right insect has been attracted, some orchids present all sorts of one-way obstacle courses to make sure it does not leave until pollen has been accurately placed or removed. By such ingenious adaptations to specific pollinators, orchids have avoided the hazards of rampant crossbreeding in the wild, assuring the survival of species as discrete identities. At the same time they have made themselves irresistible to collectors.
What does the passage mainly discuss?

A. Birds

B. Insects

C. Flowers

D. Perfume

Câu 399 :
The orchid is unique because of ........

A. the habitat in which it lives

B. the structure of its blossom

C. the variety of products that can be made from it

D. the length of its life

Câu 400 :
The word "fused" in line 2 is closest in meaning to .......

A. combined

B. hidden

C. fertilized

D. produced

Câu 401 :
Orchids are unique in having the most highly developed of all blossoms, in which the usual male and female reproductive organs are fused in a single structure called the column. The column is designed so that a single pollination will fertilize hundreds of thousands, and in some cases millions, of seeds, so microscopic and light they are easily carried by the breeze. Surrounding the column are three sepals and three petals, sometimes easily recognizable as such, often distorted into gorgeous, weird, but always functional shapes. The most noticeable of the petals is called the labellum, or lip. It is often dramatically marked as an unmistakable landing strip to attract the specific insect the orchid has chosen as its pollinator. To lure their pollinators from afar, orchids use appropriately intriguing shapes, colors, and scents. At least 50 different aromatic compounds have been analyzed in the orchid family, each blended to attract one, or at most a few, species of insects or birds. Some orchids even change their scents to interest different insects at different times. Once the right insect has been attracted, some orchids present all sorts of one-way obstacle courses to make sure it does not leave until pollen has been accurately placed or removed. By such ingenious adaptations to specific pollinators, orchids have avoided the hazards of rampant crossbreeding in the wild, assuring the survival of species as discrete identities.
At the same time they have made themselves irresistible to collectors. How many orchid seeds are typically pollinated at one time?

A. 200

B. 2,000

C. 20,000

D. 200,000

Câu 402 :
Which of the following is a kind of petal?

A. The column

B. The sepal

C. The stem

D. The labellum

Câu 403 :
The “labellum” (line 7) is most comparable to .........

A. a microscope

B. an obstacle course

C. an airport runway

D. a racetrack

Câu 404 :
The word "their" in line 12 refers to ......

A. orchids

B. birds

C. insects

D. species

Câu 405 :
The word "discrete" in line 16 is closest in meaning to ......

A. complicated

B. separate

C. inoffensive

D. functional

Câu 406 :
Read the following passage and mark the letter A, B, C, or D on your answer sheet to indicate the correct answer to each of the questions
The American type of football was developed in the 19th century from soccer and rugby football. Played by professionals, amateurs, college and high school students, or young children, football in American is one of the most popular sports besides basketball and baseball. It attracts millions of fans each fall and people are very supportive of their favourite teams. The football playing field of today is rectangular in shape and measures 100 yards long and 53.5 yards wide. White lines are painted on the playing field to mark off the distances to the end zone. The games is divided into four quarters, each fifteen minutes long. The first two quarters are known as the first half. There is a rest period between two halves which usually last about fifteen minutes. Each team has eleven players. Each team has offensive players who play when the team has possession of the ball and defensive players who play when the other team has the possession of the ball. Because of the body contact players have during the game, helmets are worn to protect their head and face area, whereas pads are worn to protect the shoulders, arms, and legs. Also, there are officials carrying whistles and flags to make certain that the rules of the game are followed during the game. The football is made of leather and is brown in colour. It is shaped much like an oval and has white rings near each end of the football. These rings help the players see the ball when it is thrown or someone is running with it. The eight stitches on the top of the football help the players to grip the ball when throwing or passing. The most famous game of the year is Super Bowl that is played in January or February. It is televised around the world and is watched by millions of people each year. 
What do officials do during the game of football?

A. build up excitement among the fans

B. supervise the game

C. take up tickets

D. spectate the game

Câu 407 :
As mentioned in the text, who are the most active when their team has the ball?

A. offensive players

B. defensive players

C. the officials

D. the fans

Câu 408 :
Playing American football is the most similar to playing .........

A. rugby football

B. basketball

C. baseball

D. volleyball

Câu 409 :
Which is the most popular sport in the US?

A. Rugby football

B. Soccer

C. American football

D. It's not mentioned

Câu 410 :
What do pads help the players to protect?

A. their legs and arms

B. their heads

C. the whole body

D. their faces

Câu 411 :
When is the most famous football game held annually?

A. in the fall

B. January

C. February

D. January or February

Câu 412 :
Why are there white rings on each end of the football?

A. to mark off the distances to the end zone

B. to help players run

C. to help players score

D. to help players see the ball

Câu 413 :
The word "grip" in the passage means to ..........

A. avoid something

B. take something away

C. old something tightly

D. detect something

Câu 414 :
Read the following passage and mark the letter A, B, C, or D on your answer sheet to indicate the correct answer to each of the questions
One day in 1924, five men who were camping in the Cascade Mountains of Washington saw a group of huge apelike creatures coming out of the woods. They hurried back to their cabin and locked themselves inside. While they were in, the creatures attacked them by throwing rocks against the walls of the cabin. After several hours, these strange hairy giants went back into the woods. After this incident the men returned to the town and told the people of their adventure. However, only a few people accepted their story. These were the people who remembered hearing tales about footprints of an animal that walked like a human being. The five men, however, were not the first people to have seen these creatures called Bigfoot. Long before their experience, local Native Americans were certain that a race of apelike animals had been living in the neighboring mountain for centuries. They called these creatures Sasquatch. In 1958, workmen, who were building a road through the jungles of Northern California often found huge footprints in the earth around their camp. Then in 1967, Roger Patterson, a man who was interested in finding Bigfoot went into the Northern California jungles with a friend. While riding, they were suddenly thrown off from their horses. Patterson saw a tall apelike animal standing not far away. He managed to shoot seven rolls of film of the hairy creature before the animal disappeared in the hushes. when Patterson's film was shown to the public, not many people believed his story. In another incident, Richard Brown, a music teacher and also an experienced hunter spotted a similar creature. He saw the animal clearly through the telescopic lens of his rifle. He said the creature looked more like a human than an animal. Later many other people also found deep footprints in the same area. In spite of regular reports of sightings and footprints, most experts still do not believe that Bigfoot really exists. 
The word neighboring would BEST be replaced with ______.

A. remote

B. nearby

C. far-off

D. far-away

Câu 415 :
Did the town people believe the story of the five men about their meeting with Bigfoot?

A. No, not everyone believed their story.

B. All the people believed what they said.

C. Some said the five men were making up their own story.

D. Only those who had heard the same tale the second time believed them.

Câu 416 :
Which of the following pairs is INCORRECT?

A. spotted - saw

B. creatures - animals

C. woods -- jungles

D. huge -- hairy

Câu 417 :
Who were the first people to have seen these apelike creatures before the five campers?

A. Richard Brown, a music teacher and a hunter.

B. Roger Patterson and his friend.

C. The local Native Americans.

D. The workers who built the road in the jungles of Northern California.

Câu 418 :
The BEST title for this passage would be ......

A. The adventures of Bigfoot.

B. The experts and the existence of Bigfoot.

C. The creature called Bigfoot.

D. The adventures of the five campers.

Câu 419 :
What did the five campers do when they saw a group of apelike creatures?

A. They threw rocks against the walls of their cabin to frighten the creatures away.

B. They attacked the creatures by throwing rocks at them.

C. They ran into the woods and hid there for several hours.

D. They quickly ran back into their cabin and locked the cabin door.

Câu 420 :
Who called the apelike creatures 'Sasquatch'?

A. Richard Brown

B. The local Native Americans

C. The five campers

D. Roger Patterson

Câu 421 :
Read the following passage and mark the letter A, B, C, or D on your answer sheet to indicate the correct answer to each of the questions
Telecommuting is a form of computer communication between employees’ homes and offices. For employees whose jobs involve sitting at a terminal or word processor entering data or typing reports, the location of the computer is of no consequence. If the machine can communicate over telephone lines, when the work is completed, employees can dial the office computer and transmit the material to their employers. A recent survey in USA Today estimates that there are approximately 8,7 million telecommuters. But although the numbers are rising annually, the trend does not appear to be as significant as predicted when Business Week published “The Portable Executive” as its cover story a few years ago. Why hasn’t telecommuting become more popular ? Clearly, change simply takes time. But in addition, there has been active resistance on the part of many managers. These executives claim that supervising the telecommuters in a large work force scattered across the country would be too difficult, or, at least, systems for managing them are not yet developed, thereby complicating the manager’s responsibilities. It is also true that employees who are given the option of telecommuting are reluctant to accept the opportunity. Most people feel that they need regular interaction with a group, and many are concerned that they will not have the same consideration for advancement if they are not more visible in the office setting. Some people feel that even when a space in their homes is set aside as a work area, they never really get away from the office. 
How many Americans are involved in telecommuting?

A. More than predicted in Business Week.

B. More than 8 million.

C. Fewer than estimated in USA Today.

D. Fewer than last year.

Câu 422 :
The phrase “of no consequence” means ___________.

A. of no use

B. of no good

C. unimportant

D. irrelevant

Câu 423 :
The author mentions all of the following as concerns of telecommuting, EXCEPT ___________ .

A. the opportunities for advancement

B. the different system of supervision

C. the lack of interaction with a group

D. The work place is in the home

Câu 424 :
The word “ them” which is bold and italic refers to ___________.

A. systems

B. telecommuters

C. executives

D. responsibilities

Câu 425 :
The reason why telecommuting has not become popular is that the employees ___________.

A. needn't regular interaction with their families

B. are worried about the promotion if they are not seen at the office

C. like that a work area in their home is away from the office

D. are ignorant of telecommuting

Câu 426 :
It can be inferred from the passage that the author is ___________.

A. a telecommuter

B. the manager

C. a statistician

D. a reporter

Câu 427 :
When Business Week published “ The Portable Executive “, it implied that___________.

A. systems for managing telecommuters were not effective

B. there was resistance on the part of many managers about telecommuting

C. the trend for telecommuting was optimistic

D. most telecommuters were satisfied with their work

Câu 428 :
Read the following passage and mark the letter A, B, C, or D on your answer sheet to indicate the correct answer to each of the questions
Marianne Moore (1887-1972) once said that her writing could be called poetry only because there was no other name for it. Indeed her poems appear to be extremely compressed essays that happen to be printed in jagged lines on the page. Her subjects were varied: animals, laborers, artists, and the craft of poetry. From her general reading came quotations that she found striking or insightful. She included these in her poems, scrupulously enclosed in quotation marks, and sometimes identified in footnotes. Of this practice, she wrote, "Why many quotation marks?" I am asked ... When a thing has been so well that it could not be said better, why paraphrase it? Hence, my writing is, if not a cabinet of fossils, a kind of collection of flies in amber." Close observation and concentration on detail and the methods of her poetry. Marianne Moore grew up in Kirkwood, Missouri, near St. Lois. After graduation from Bryn Mawr College in 1909, she taught commercial subjects at the Indian School in Carlisle, Pennsylvania. Later she became a librarian in New York City. During the 1920’s she was editor of The Dial, an important literary magazine of the period. She lived quietly all her life, mostly in Brooklyn, New York. She spent a lot of time at the Bronx Zoo, fascinated by animals. Her admiration of the Brooklyn Dodgers-before the team moved to Los Angeles-was widely known. Her first book of poems was published in London in 1921 by a group of friends associated with the Imagist movement. From that time on her poetry has been read with interest by succeeding generations of poets and readers. In 1952 she was awarded the Pulitzer Prize for her Collected Poems. She wrote that she did not write poetry for money or fame. To earn a living is needful, but it can be done in routine ways. One writes because one has a burning desire to objectify what it is indispensable to one's happiness to express. 
What is the passage mainly about?

A. The influence of the imagists on Marianne Moore.

B. Essayists and poets of the 1920's.

C. The use of quotations in poetry.

D. Marianne Moore's life and work.

Câu 429 :
Which of the following can be inferred about Moore's poems?

A. They are better known in Europe than the United States.

B. They do not use traditional verse forms.

C. They were all published in The Dial.

D. They tend to be abstract.

Câu 430 :
According to the passage Moore wrote about all of the following EXCEPT ________

A. artists

B. animals

C. fossils

D. workers

Câu 431 :
Where did Marianne Moore grow up?

A. In Carlisle, Pennsylvania.

B. In Kirkwood.

C. In New York City

D. In Los Angeles.

Câu 432 :
The author mentions all of the following as jobs held by Moore EXCEPT _______

A. commercial artist

B. teacher

C. magazine editor

D. librarian

Câu 433 :
Where did Moore spend most of her adult life?

A. In Kirkwood.

B. In Brooklyn.

C. In Los Angeles.

D. In Carlisle.

Câu 434 :
The word "succeeding" is closest to ______.

A. inheriting

B. prospering

C. diverse

D. later

Câu 435 :
The word "it" refers to ______. 

A. writing poetry 

B. becoming famous 

C. earning a living 

D. attracting readers

Câu 436 :
Read the following passage and mark the letter A, B, C, or D on your answer sheet to indicate the correct answer to each of the questions
Mickey Mantle was one of the greatest baseball players of all time. He played for the New York Yankees in their years of glory. From the time Mantle began to play professionally in 1951 to his last year in 1968, baseball was the most popular game in the United States. For many people, Mantle symbolized the hope, prosperity, and confidence of America at that time. Mantle was a fast and powerful player, a “switch-hitter” who could bat both right-handed and lefthanded. He won game after game, one World Series championship after another, for his team. He was a wonderful athlete, but this alone cannot explain America’s fascination with him. Perhaps it was because he was a handsome, red-haired country boy, the son of a poor miner from Oklahoma. His career, from the lead mines of the West to the heights of success and fame, was a fairy-tale version of the American dream. Or perhaps it was because America always loves a “natural”: a person who wins without seeming to try, whose talent appears to come from an inner grace. That was Mickey Mantle. But like many celebrities, Mickey Mantle had a private life that was full of problems. He played without complaint despite constant pain from injuries. He lived to fulfill his father’s dreams and drank to forget his father’s early death. It was a terrible addiction that finally destroyed his body. It gave him cirrhosis of the liver and accelerated the advance of liver cancer. Even when Mickey Mantle had turned away from his old life and warned young people not to follow his example, the destructive process could not be stopped. Despite a liver transplant operation that had all those who loved and admired him hoping for a recovery, Mickey Mantle died of cancer at the age of 63.
What is the main idea of the passage?

A. Mickey Mantle’s success and private life full of problems

B. Mickey Mantle as the greatest baseball player of all time

C. Mickey Mantle and the history of baseball

D. Mickey Mantle and his career as a baseball player

Câu 437 :
It can be inferred from paragraph 1 that Mantle ______.

A. introduced baseball into the US

B. earned a lot of money from baseball

C. had to try hard to be a professional player

D. played for New York Yankees all his life

Câu 438 :
The word “this” in paragraph 2 refers to ______.

A. Mantle’s being fascinated by many people

B. Mantle’s being a wonderful athlete

C. Mantle’s being a “switch-hitter”

D. Mantle’s being a fast and powerful player

Câu 439 :
It can be inferred from the passage that for most Americans ______.

A. success in Mantle’s career was difficult to believe

B. Mantle had a lot of difficulty achieving fame and success

C. success in Mantle’s career was unnatural

D. Mantle had to be trained hard to become a good player

Câu 440 :
The author uses the word “But” in paragraph 4 to ______.

A. explain how Mantle got into trouble

B. give an example of the trouble in Mantle’s private life

C. give an argument in favor of Mantle’s success and fame

D. change the topic of the passage

Câu 441 :
The word “accelerated” in paragraph 5 is closest in meaning to ______.

A. delayed

B. worsened

C. bettered

D. quickened

Câu 442 :
Which of the following is mentioned as the main cause of the destruction of Mantle’s body?

A. His way of life

B. His loneliness

C. His own dream

D. His liver transplant operation

Câu 443 :
Which of the following is NOT mentioned as an activity of relaxation in the countryside of Britain?

A. Going swimming

B. Picking fruit

C. Going for a walk

D. Riding a bicycle

Câu 444 :
Read the following passage and mark the letter A, B, C, or D on your answer sheet to indicate the correct answer to each of the questions
The countryside of Britain is well known for its beauty and many contrasts: its bare mountains and moorland, its lakes, rivers and woods, and its long, often wild coastline. Many of the most beautiful areas are national parks and are protected from development. When British people think of the countryside they think of farmland, as well as open spaces. They imagine cows or sheep in green fields enclosed by hedges or stone walls, and fields of wheat and barley. Most farmland is privately owned but is crossed by a network of public footpaths. Many people associate the countryside with peace and relaxation. They spend their free time walking or cycling there, or go to the country for a picnic or a pub lunch. In summer people go to fruit farms and pick strawberries and other fruit. Only a few people who live in the country work on farms. Many commute to work in towns. Many others dream of living in the country, where they believe they would have a better and healthier lifestyle. The countryside faces many threats. Some are associated with modern farming practices, and the use of chemicals harmful to plants and wildlife. Land is also needed for new houses. The green belt, an area of land around many cities, is under increasing pressure. Plans to build new roads are strongly opposed by organizations trying to protect the countryside. Protesters set up camps to prevent, or at least delay, the building work. America has many areas of wild and beautiful scenery, and there are many areas, especially in the West in states like Montana and Wyoming, where few people live. In the New England states, such as Vermont and New Hampshire, it is common to see small farms surrounded by hills and green areas. In Ohio, Indiana, Illinois and other Midwestern states, fields of corn or wheat reach to the horizon and there are many miles between towns. Only about 20% of Americans live outside cities and towns. Life may be difficult for people who live in the country. Services like hospitals and schools may be further away and going shopping can mean driving long distances. Some people even have to drive from their homes to the main road where their mail is left in a box. In spite of the disadvantages, many people who live in the country say that they like the safe, clean, attractive environment. But their children often move to a town or city as soon as they can. As in Britain, Americans like to go out to the country at weekends. Some people go on camping or fishing trips, others go hiking in national parks.
We can see from the passage that in the countryside of Britain ______.

A. none of the areas faces the sea

B. only a few farms are publicly owned

C. most beautiful areas are not well preserved

D. it is difficult to travel from one farm to another

Câu 445 :
What does the word “they” in paragraph 2 refer to?

A. Those who dream of living in the country

B. Those who go to fruit farms in summer

C. Those who go to the country for a picnic

D. Those who commute to work in towns

Câu 446 :
Which of the following threatens the countryside in Britain?

A. Protests against the building work

B. Modern farming practices

C. Plants and wildlife

D. The green belt around cities

Câu 447 :
According to the passage, all of the following are true EXCEPT ______.

A. the use of chemicals harms the environment of the countryside

B. camps are set up by protesters to stop the construction work

C. the green belt is under pressure because of the need for land

D. all organizations strongly oppose plans for road construction

Câu 448 :
The phrase “reach to the horizon” in paragraph 4 is closest in meaning to ______.

A. are limited

B. are endless

C. are horizontal

D. are varied reach to

Câu 449 :
According to the passage, some Americans choose to live in the country because ___.

A. their children enjoy country life

B. they enjoy the safe, clean, attractive environment there

C. hospitals, schools and shops are conveniently located there

D. life there may be easier for them

Câu 450 :
Which of the following is NOT mentioned in the passage?

A. The majority of American people live in cities and towns.

B. Many British people think of the country as a place of peace and relaxation.

C. Towns in some Midwestern states in the US are separated by long distances.

D. Both British and American people are thinking of moving to the countryside.

Câu 451 :
Read the following passage and mark the letter A, B, C, or D on your answer sheet to indicate the correct answer to each of the questions
Traditionally in America, helping the poor was a matter for private charities or local governments. Arriving immigrants depended mainly on predecessors from their homeland to help them start a new life. In the late 19th and early 20th centuries, several European nations instituted public-welfare programs. But such a movement was slow to take hold in the United States because the rapid pace of industrialization and the ready availability of farmland seemed to confirm the belief that anyone who was willing to work could find a job. Most of the programs started during the Depression era were temporary relief measures, but one of the programs Social Security – has become an American institution. Paid for by deductions from the paychecks of working people, Social Security ensures that retired persons receive a modest monthly income and also provides unemployment insurance, disability insurance, and other assistance to those who need it. Social Security payments to retired persons can start at age 62, but many wait until age 65, when the payments are slightly higher. Recently, there has been concern that the Social Security fund may not have enough money to fulfill its obligations in the 21st century, when the population of elderly Americans is expected to increase dramatically. Policy makers have proposed various ways to make up the anticipated deficit, but a long-term solution is still being debated. In the years since Roosevelt, other American presidents have established assistance programs. These include Medicaid and Medicare; food stamps, certificates that people can use to purchase food; and public housing which is built at federal expense and made available to persons on low incomes. Needy Americans can also turn to sources other than the government for help. A broad spectrum of private charities and voluntary organizations is available. Volunteerism is on the rise in the United States, especially among retired persons. It is estimated that almost 50 percent of Americans over age 18 do volunteer work, and nearly 75 percent of U.S. households contribute money to charity. 
New immigrants to the U.S. could seek help from ______.

A. the people who came earlier

B. the US government agencies

C. only charity organizations

D. volunteer organizations

Câu 452 :
It took welfare programs a long time to gain a foothold in the U.S. due to the fast growth of______.

A. industrialization

B. modernization

C. urbanization

D. population

Câu 453 :
The Social Security program has become possible thanks to ______.

A. deductions from wages

B. people’s willingness to work

C. donations from companies

D. enforcement laws

Câu 454 :
Most of the public assistance programs ______ after the severe economic crisis.

A. were introduced into institutions

B. did not become institutionalized

C. functioned fruitfully in institutions

D. did not work in institutions

Câu 455 :
That Social Security payments will be a burden comes from the concern that ______.

A. elderly people ask for more money

B. the program discourages working people

C. the number of elderly people is growing

D. younger people do not want to work

Câu 456 :
Persons on low incomes can access public housing through ______.

A. low rents

B. state spending

C. donations

D. federal expenditure

Câu 457 :
The passage mainly discusses ______.

A. public assistance in America

B. immigration into America

C. funding agencies in America

D. ways of fund-raising in America

Câu 458 :
Read the following passage and mark the letter A, B, C, or D on your answer sheet to indicate the correct answer to each of the questions
Millions of people are using cellphones today. In many places, it is actually considered unusual not to use one. In many countries, cellphones are very popular with young people. They find that the phones are more than a means of communication – having a mobile phone shows that they are cool and connected. The explosion in mobile phone use around the world has made some health professionals worried. Some doctors are concerned that in the future many people may suffer health problems from the use of mobile phones. In England, there has been a serious debate about this issue. Mobile phone companies are worried about the negative publicity of such ideas. They say that there is no proof that mobile phones are bad for your health. On the other hand, medical studies have shown changes in the brain cells of some people who use mobile phones. Signs of change in the tissues of the brain and head can be detected with modern scanning equipment. In one case, a traveling salesman had to retire at young age because of serious memory loss. He couldn’t remember even simple tasks. He would often forget the name of his own son. This man used to talk on his mobile phone for about six hours a day, every day of his working week, for a couple of years. His family doctor blamed his mobile phone use, but his employer’s doctor didn’t agree. What is it that makes mobile phones potentially harmful? The answer is radiation. High-tech machines can detect very small amounts of radiation from mobile phones. Mobile phone companies agree that there is some radiation, but they say the amount is too small to worry about. As the discussion about their safety continues, it appears that it’s best to use mobile phones less often. Use your regular phone if you want to talk for a long time. Use your mobile phone only when you really need it. Mobile phones can be very useful and convenient, especially in emergencies. In the future, mobile phones may have a warning label that says they are bad for your health. So for now, it’s wise not to use your mobile phone too often. 
According to the passage, cellphones are especially popular with young people because ______.

A. they are indispensable in everyday communications

B. they make them look more stylish

C. they keep the users alert all the time

D. they cannot be replaced by regular phones

Câu 459 :
The changes possibly caused by the cellphones are mainly concerned with ______.

A. the mobility of the mind and the body

B. the smallest units of the brain

C. the arteries of the brain

D. the resident memory

Câu 460 :
The word “means” in the passage most closely means ______.

A. “meanings”

B. “expression”

C. “method”

D. “transmission”

Câu 461 :
Doctors have tentatively concluded that cellphones may ________.

A. damage their users’ emotions

B. cause some mental malfunction

C. change their users’ temperament

D. change their users’ social behaviours

Câu 462 :
The man mentioned in the passage, who used his cellphone too often, ______.

A. suffered serious loss of mental ability

B. could no longer think lucidly

C. abandoned his family

D. had a problem with memory

Câu 463 :
According to the passage, what makes mobile phones potentially harmful is ______.

A. their radiant light

B. their raiding power

C. their power of attraction

D. their invisible rays

Câu 464 :
According to the writer, people should ______.

A. only use mobile phones in urgent cases

B. only use mobile phones in medical emergencies

C. keep off mobile phones regularly

D. never use mobile phones in all cases

Câu 465 :
The most suitable title for the passage could be ______.

A. “The Reasons Why Mobile Phones Are Popular”

B. “Technological Innovations and Their Price”

C. “The Way Mobile Phones Work”

D. “Mobile Phones: A Must of Our Time”

Câu 466 :
Read the following passage and mark the letter A, B, C, or D on your answer sheet to indicate the correct answer to each of the questions
Supernovas are the most powerful and spectacular outbursts known in nature. What is called a Type II supernova is due to the collapse of a massive star, at least eight times as massive as the sun, that has used up its main nuclear fuel and produced a nickel-iron core. When this core can no longer support the pressure of the star’s outer layer, it collapses to form a neutron star of immense density. Over 2,500 million tons of neutron star packed into a matchbox. Its temperature is around 100,000 million degrees centigrade. Multitudes of neutrons are produced in the collapsed star, which pass directly through the star into space, and this release of neutrons causes the core to respond with a shock wave that moves outward. When it meets the material that is falling inward, the result is a catastrophic explosion. Sometimes most of the star’s material is blown away, leaving only a small, incredibly dense remnant that may be a neutron star or, in extreme cases, a black hole. A supernova is often more than 500 million times as luminous as the sun. A supernova remnant (SNR) may be detectable as a pulsar, an example of which is the Crab Nebula, known to be a remnant of the supernova observed in the year 1054. The 1987 supernova in the Large Cloud of Magellan had a low peak luminosity by supernova standards, only about 250 million times that of the sun. As its brightest the supernova shone as a star between magnitudes 2 and 3, even though it was 170,000 light-years away. 
What is the main topic of the passage?

A. The heat of supernovas

B. The formation and power of a supernova

C. The role of shock waves in a supernova

D. The density of a neutron star.

Câu 467 :
The word “it” refers to __________. 

A. core of the collapsed star

B. neutron star 

C. shock wave 

D. catastrophic explosion

Câu 468 :
According to the passage, which of the following is TRUE about the 1987 supernova?

A. It was the brightest supernova.

B. It was brighter than the sun.

C. It shone as a star between magnitudes 3 and 4.

D. It had the lowest peak luminously.

Câu 469 :
The author of this passage is most likely __________________ .

A. an astronomer

B. an economist

C. a mathematician

D. a botanist

Câu 470 :
The word “multitudes” is closest in meaning to _________ .

A. small numbers

B. groups

C. lots

D. temperatures

Câu 471 :
The word “luminous” is closest in meaning to _________ .

A. powerful

B. bright

C. hot

D. distant

Câu 472 :
What can be inferred from the passage about supernovas?

A. They sometimes result in a black hole

B. The sun is a remnant of a supernova

C. They occur when two stars collide

D. They only happen to pulsars

Câu 473 :
Read the following passage and mark the letter A, B, C, or D on your answer sheet to indicate the correct answer to each of the questions
In the 1960s, The Beatles were probably the most famous pop group in the whole world. Since then, there have been a great many groups that have achieved enormous fame, so it is perhaps difficult now to imagine how sensational The Beatles were at that time. They were four boys from the north of England and none of them had any training in music. They started by performing and recording songs by black Americans and they had some success with these songs. Then they started writing their own songs and that was when they became really popular. The Beatles changed pop music. They were the first pop group to achieve great success from songs they had written themselves. After that it became common for groups and singers to write their own songs. The Beatles did not have a long career. Their first hit record was in 1963 and they split up in 1970. They stopped doing live performances in 1966 because it had become too dangerous for them - their fans were so excited that they surrounded them and tried to take their clothes as souvenirs! However, today some of their songs remain as famous as they were when they first came out. Throughout the world, many people can sing part of a Beatles song if you ask them.
The passage is mainly about __________

A. why the Beatles split up after 7 years

B. many people's ability to sing a Beatles song

C. the Beatles' fame and success

D. how the Beatles became more successful than other groups

Câu 474 :
The word 'sensational' is closest in meaning to __________.

A. popular

B. shocking

C. notorious

D. bad

Câu 475 :
The first songs of the Beatles were __________

A. written by black Americans

B. paid a lot of money

C. broadcast on the radio

D. written by themselves

Câu 476 :
What is NOT true about the Beatles?

A. They had a long stable career.

B. They became famous when they wrote their own songs.

C. The members had no training in music.

D. They changed pop music.

Câu 477 :
The Beatles stopped their live performances because _________

A. they had earned enough money

B. they did not want to work with each other

C. they spent more time writing their own songs

D. they were afraid of being hurt by fans

Câu 478 :
Some songs of the Beatles now __________

A. are the most famous

B. became too old to sing

C. are still famous as they used to be

D. are sung by crazy fans

Câu 479 :
The tone of the passage is that of __________

A. admiration

B. criticism

C. sarcasm

D. neutral

Câu 480 :
Read the following passage and mark the letter A, B, C, or D on your answer sheet to indicate the correct answer to each of the questions
Most languages have several levels of vocabulary that may be used by the same speakers. In English, at least three have been identified and described. Standard usage includes those words and expressions understood, used, and accepted by a majority of the speakers of a language in any situation regardless of the levels of formality. As such, these words and expressions are well defined and listed in standard dictionaries. Colloquialisms, on the other hand, are familiar words and idioms that are understood by almost all speakers of a language and used in informal speech or writing, but not considered acceptable for more formal situations. Almost all idiomatic expressions are colloquial language. Slang, however, refers to words and expressions understood by a large number of speakers but not accepted as appropriate formal usage by the majority. Colloquial expressions and even slang may be found in standard dictionaries but will be so identified. Both Colloquial usage and slang are more common in speech than in writing. Colloquial speech often passes into standard speech. Some slang also passes into standard speech, but other slang expressions enjoy momentary popularity followed by obscurity. In some cases, the majority never accepts certain slang phrases but nevertheless retains them in their collective memories. Every generation seems to require its own set of words to describe familiar objects and events. It has been pointed out by a number of linguists that three cultural conditions are necessary for the creation of a large body of slang expressions. First, the introduction and acceptance of new objects and situations in the society; second, a diverse population with a large number of subgroups; third, association among the subgroups and the majority population. Finally, it is worth noting that the terms "standard", "colloquial" and "slang" exist only as abstract labels for scholars who study language. Only a tiny number of the speakers of any language will be aware that they are using colloquial or slang expressions. Most speakers of English will, during appropriate situations, select and use all three types of expressions. 
Which of the following is the main topic of the passage?

A. Standard speech

B. Idiomatic phrases

C. Dictionary usage

D. Different types of vocabulary

Câu 481 :
How is slang defined by the author?

A. Words and phrases understood by the majority but not found in standard dictionaries.

B. Words and phrases accepted by the majority for formal usage.

C. Words and phrases that are understood by a restricted group of speakers.

D. Words and phrases understood by a large number of speakers but not accepted as formal usage.

Câu 482 :
The word "obscurity" could best be replaced by __________

A. qualification

B. tolerance

C. disappearance

D. influence

Câu 483 :
The word "them" refers to __________ 

A. slang phrases 

B. memories 

C. the majority 

D. words

Câu 484 :
The statement: "Colloquialisms, on the other hand, are familiar words and idioms that are understood by almost all speakers of a language and used in informal speech or writing, but not considered acceptable for more formal situations." means:

A. Most of the speakers of a language can use both formal and informal speech in appropriate situations.

B. Familiar situations that are experienced by most people are called colloquialisms.

C. Familiar words and phrases are found in both speech and writing in formal settings.

D. Informal languag

Câu 485 :
Which of the following is true of standard usage?

A. It can be used in formal or informal settings.

B. It is only understood be the upper classes.

C. It limited to written language.

D. It is constantly changing.

Câu 486 :
The author mentions all of the following as requirements for slang expressions to be created EXCEPT ......

A. new situations

B. a new generation

C. interaction among diverse groups

D. a number of linguists

Câu 487 :
It can be inferred from the passage that the author ....

A. approves of colloquial speech in some situations, but not slang

B. approves of slang and colloquial speech in appropriate situations

C. does not approve of either slang or colloquial speech in any situation

D. does not approve of colloquial usage in writing

Câu 488 :
Read the following passage and mark the letter A, B, C, or D on your answer sheet to indicate the correct answer to each of the questions
Mandatory volunteering made many members of Maryland's high school class of 97 grumble with indignation. They didn't like a new requirement that made them take part in the school's community service program. Future seniors, however, probably won't be as resistant now that the program has broken in. Some, like John Maloney, already have completed their required hours of approved community service. The Bowie High School sophomore earned his hours in eighth grade by volunteering two nights a week at the Larkin- Chase Nursing and Restorative Center in Bowie.He played shuffle board, cards, and other games with the senior citizens. He also helped plan parties for them and visited their rooms to keep them company. John, fifteen, is not finished volunteering. Once a week he videotapes animals at the Prince George's County animal shelter in Forestville. His footage is shown on the Bowie public access television channel in hopes of finding homes for the animals. "Volunteering is better than just sitting around," says John, "and I like animals; I don't want to see them put to sleep." He's not the only volunteer in his family. His sister, Melissa, an eighth grader, has completed her hours also volunteering at Larkin-Chase. "It is a good idea to have kids go out into the community, but it's frustrating to have to write essays about the works," she said. It makes you feel like you're doing it for the requirement and not for yourself." The high school's service learning office, run by Beth Ansley, provides information on organizations seeking volunteers so that students will have an easier time fulfilling their hours. "It's ridiculous that people are opposing the requirements," said Amy Rouse, who this summer has worked at the Ronald McDonald House and has helped to rebuild a church in Clinton. "So many people won't do the service unless it's mandatory," Rouse said, "but once they start doing it, they'll really like it and hopefully it will become a part of their lives - like it has become a part of mine." 
The best title of the passage could be ______.

A. "Students Who Volunteer to Work with Senior Citizens"

B. "Students Who Earn Extra Money after School"

C. "A Volunteer Program at Bowie High School"

D. "The High School Class of 1977"

Câu 489 :
The word "frustrating" in paragraph 7 is closest in meaning to ______.

A. interesting

B. happy

C. satisfying

D. upset

Câu 490 :
The word "it" in paragraph 7 refers to ______.

A. completing requirements

B. writing essays

C. doing volunteer work

D. going out in the community

Câu 491 :
From paragraphs 6 and 7, we can infer that Melissa Maloney ______.

A. volunteers because it's a requirement

B. is frustrated by her volunteer job

C. volunteers because it makes her feel good

D. doesn't like to write essays about her volunteer work

Câu 492 :
According to the last two paragraphs, Amy Rouse thinks that ______.

A. most people don't like volunteering, so they don't want to do it

B. most people will discover they enjoy volunteering if they try it

C. the volunteer program shouldn't be mandatory

D. people should be able to choose whether they want to volunteer

Câu 493 :
Which of the following volunteer activities is NOT mentioned in the passage?

A. Videotaping animals in a shelter.

B. Rebuilding a church.

C. Tutoring children.

D. Visiting elderly people.

Câu 494 :
In the passage, the author gives the explanation of the concept of mandatory volunteer programs by ______.

A. describing one volunteer program

B. classifying different types of volunteer programs

C. arguing in favor of volunteer programs

D. comparing two volunteer programs

Câu 495 :
Read the following passage and mark the letter A, B, C, or D on your answer sheet to indicate the correct answer to each of the questions
The mineral particles found in soil range in size from microscopic clay particles to large boulders. The most abundant particles - sand, silt, and clay - are the focus of examination in studies of soil texture. Texture is the term used to describe the line composite sizes of particles in a soil sample, typically several representative handfuls . To measure soil texture, the sand, silt, and clay particles are sorted out by size and weight. The weights of each size are then expressed as a percentage of the sample weight. In the field, soil texture can be estimated by extracting a handful of sod and squeezing the damp soil into three basic shapes; (1) cast, a lump formed by squeezing a sample in a clenched fist; (2) thread, a pencil shape formed by rolling soil between the palms; and (3) ribbon, a flatfish shape formed by squeezing a small sample between the thumb and index finger. The behavioral characteristics of the soil when molded into each of these shapes, if they can be formed at all, provides the basis for a general textural classification. The behavior of the soil in the hand test is determined by the amount of clay in the sample. Clay particles are highly cohesive, and when dampened , behave as a plastic. Therefore the higher the clay content in a sample, the more refined and durable the shapes into which it can be molded. Another method of determining soil texture involves the use of devices called sediment sieves, screens built with a specified mesh size. When the soil is filtered through a group of sieves, each with a different mesh size, the particles become grouped in corresponding size categories. Each category can be weighed to make a textural determination. Although sieves work well for silt, sand, and larger particles, they are not appropriate for clay particles. Clay is far too small to sieve accurately; therefore, in soils with a high proportion of clay, the fine particles are measured on the basis of their settling velocity when suspended in water. Since clays settle so slowly, they are easily segregated from sand and silt. The water can be drawn off and evaporated, leaving a residue of clay, which can be weighed. 
The author mentions "several representative handfuls" in the passage in order to show ______.

A. the range of soil samples

B. the requirements for an adequate soil sam

C. the process by which soil is weighed

D. how small soil particles are

Câu 496 :
It can be inferred that the names of the three basic shapes mentioned in paragraph 2 reflect ______.

A. the way the soil is extracted

B. the need to check more than one handful

C. the difficulty of forming different shapes

D. the results of squeezing the soil

Câu 497 :
The word "dampened" in the passage is closest in meaning to _____.

A. damaged

B. stretched

C. moistened

D. examined

Câu 498 :
It can be inferred from the passage that a soil sample with little or no clay in it ______.

A. does not have a durable shape

B. is not very heavy

C. does not have a classifiable texture

D. may not hold its shape when molded

Câu 499 :
The word "they" in the passage refers to ______.

A. larger particles

B. sieves

C. categories

D. clay particles

Câu 500 :
It can be inferred from the passage that the sediment sieve has an advantage over the hand test in determining soil texture because ______.

A. the sieve allows for a more exact measure

B. less training is required to use the sieve

C. using the sieve takes less time

D. the sieve can measure clay

Câu 501 :
The mineral particles found in soil range in size from microscopic clay particles to large boulders. The most abundant particles - sand, silt, and clay - are the focus of examination in studies of soil texture. Texture is the term used to describe the line composite sizes of particles in a soil sample, typically several representative handfuls . To measure soil texture, the sand, silt, and clay particles are sorted out by size and weight. The weights of each size are then expressed as a percentage of the sample weight. In the field, soil texture can be estimated by extracting a handful of sod and squeezing the damp soil into three basic shapes; (1) cast, a lump formed by squeezing a sample in a clenched fist; (2) thread, a pencil shape formed by rolling soil between the palms; and (3) ribbon, a flatfish shape formed by squeezing a small sample between the thumb and index finger. The behavioral characteristics of the soil when molded into each of these shapes, if they can be formed at all, provides the basis for a general textural classification. The behavior of the soil in the hand test is determined by the amount of clay in the sample. Clay particles are highly cohesive, and when dampened , behave as a plastic. Therefore the higher the clay content in a sample, the more refined and durable the shapes into which it can be molded. Another method of determining soil texture involves the use of devices called sediment sieves, screens built with a specified mesh size. When the soil is filtered through a group of sieves, each with a different mesh size, the particles become grouped in corresponding size categories. Each category can be weighed to make a textural determination. Although sieves work well for silt, sand, and larger particles, they are not appropriate for clay particles. Clay is far too small to sieve accurately; therefore, in soils with a high proportion of clay, the fine particles are measured on the basis of their settling velocity when suspended in water. Since clays settle so slowly, they are easily segregated from sand and silt. The water can be drawn off and evaporated, leaving a residue of clay, which can be weighed.
During the procedure described in paragraph 3, when clay particles are placed into water they ______.

A. stick to the sides of the water container

B. dissolve quickly

C. take some time to sink to the bottom

D. separate into different sizes

Câu 502 :
The word "fine" in the passage is closest in meaning to ______.

A. many

B. excellent

C. tiny

D. various

Câu 503 :
Read the following passage and mark the letter A, B, C, or D on your answer sheet to indicate the correct answer to each of the questions
Thanksgiving is celebrated in the US on the fourth Thursday in November. For many Americans it is the most important holiday apart from Christmas. Schools, offices and most businesses close for Thanksgiving, and many people make the whole weekend a vacation. Thanksgiving is associated with the time when Europeans first came to North America. In 1620 the ship the Mayflowers arrived, bringing about 150 people who today are usually called Pilgrims. They arrived at the beginning of a very hard winter and could not find enough to eat, so many of them died. But in the following summer Native Americans showed them what foods were safe to eat, so that they could save food for the next winter. They held a big celebration to thank God and the Native Americans for the fact that they had survived. Today people celebrate Thanksgiving to remember these early days. The most important part of the celebration is a traditional dinner with foods that come from North America. The meal includes turkey, sweet potatoes (also called yams) and cranberries, which are made into a kind of sauce or jelly. The turkey is filled with stuffing or dressing, and many families have their own special recipe. Dessert is pumpkin made into a pie. On Thanksgiving there are special television programs and sports events. In New York there is the Macy's Thanksgiving Day Parade, when a long line of people wearing fancy costumes march through the streets with large balloons in the shape of imaginary characters. Thanksgiving is considered the beginning of the Christmas period, and the next day many people go out to shop for Christmas presents. 
What is the purpose of the passage?

A. To compare how Thanksgiving was celebrated in the past and today.

B. To give an overview of a popular celebration in the US.

C. To explain the existence of a celebration in the US.

D. To introduce a Native American culture.

Câu 504 :
In the United States, Thanksgiving is ______.

A. a religious celebration held by Christians only.

B. celebrated as a public holiday.

C. more important than Christmas.

D. apart from Christmas.

Câu 505 :
According to the passage, Pilgrims are ______.

A. people who traveled to America by ships

B. trips that religious people make to a holy place

C. people who left their home and went to live in North America in 1620s

D. Native Americans who live in North America

Câu 506 :
All of the following statements are mentioned EXCEPT ______.

A. People celebrate Thanksgiving to thank God.

B. People usually have traditional dinners on Thanksgiving.

C. There are lots of entertainments on Thanksgiving.

D. People go to churches for religious services on Thanksgiving.

Câu 507 :
Which of the following statements is not TRUE about Thanksgiving ______?

A. turkey, yams and pumpkin pies are served.

B. people join in the Macy's Thanksgiving Day Parade.

C. people wear colored costumes marching through the streets.

D. people go out to shop for Christmas presents.

Câu 508 :
In the second paragraph, the pronoun "them" refers to which of the following?

A. Pilgrims

B. the Mayflowers

C. Native Americans

D. Schools

Câu 509 :
Which of the following is NOT true?

A. In the US, Thanksgiving is not a national holiday; it's a religious holiday.

B. Christmas comes less than a month after Thanksgiving.

C. Thanksgiving was originally celebrated by the first Europeans in North America to thank God for their survival.

D. The Macy's Thanksgiving Day Parade is colorful and exciting.

Câu 510 :
Read the following passage and mark the letter A, B, C, or D on your answer sheet to indicate the correct answer to each of the questions
A rather surprising geographical feature of Antarctica is that a huge freshwater lake, one of the world’s largest and deepest, lies hidden there under four kilometers of ice. Now known as Lake Vostok, this huge body of water is located under the ice block that comprises Antarctica. The lake is able to exist in its unfrozen state beneath this block of ice because its waters are warmed by geothermal heat from the earth’s core. The thick glacier above Lake Vostok actually insulates it from the frigid temperatures on the surface. The lake was first discovered in the 1970s while a research team was conducting an aerial survey of the area. Radio waves from the survey equipment penetrated the ice and revealed a body of water of indeterminate size. It was not until much more recently that data collected by satellite made scientists aware of the tremendous size of the lake; the satellite-borne radar detected an extremely flat region where the ice remains level because it is floating on the water of the lake. The discovery of such a huge freshwater lake trapped under Antarctica is of interest to the scientific community because of the potential that the lake contains ancient microbes that have survived for thousands of years, unaffected by factors such as nuclear fallout and elevated ultraviolet light that have affected organisms in more exposed areas. The downside of the discovery, however, lies in the difficulty of conducting research on the lake in such a harsh climate and in the problems associated with obtaining uncontaminated samples from the lake without actually exposing the lake to contamination. Scientists are looking for possible ways to accomplish this. 
The word “hidden” in paragraph 1 is closest in meaning to ______.

A. undrinkable

B. untouched

C. unexploitable

D. undiscovered

Câu 511 :
What is true of Lake Vostok?

A. It is completely frozen.

B. It is saltwater lake.

C. It is beneath a thick slab of ice.

D. It is heated by the sun.

Câu 512 :
Which of the following is closest in meaning to “frigid” in paragraph 1?

A. extremely cold

B. easily broken

C. quite harsh

D. lukewarm

Câu 513 :
All of the following are true about the 1970 survey of Antarctica EXCEPT that it______.

A. was conducted by air

B. made use of radio waves

C. could not determine the lake’s exact size

D. was controlled by a satellite

Câu 514 :
It can be inferred from the passage that the ice would not be flat if______.

A. there were no lake underneath

B. the lake were not so big

C. Antarctica were not so cold

D. radio waves were not used

Câu 515 :
The word “microbes” in paragraph 3 could best be replaced by which of the following?

A. Pieces of dust

B. Tiny bubbles

C. Tiny organisms

D. Rays of light

Câu 516 :
Lake Vostok is potentially important to scientists because it________.

A. can be studied using radio waves

B. may contain uncontaminated microbes

C. may have elevated levels of ultraviolet light

D. has already been contaminated

Câu 517 :
The purpose of the passage is to ________.

A. explain how Lake Vostok was discovered

B. provide satellite data concerning Antarctica

C. discuss future plans for Lake Vostok

D. present an unexpected aspect of Antarctica’s geography

Câu 518 :
Read the following passage and mark the letter A, B, C, or D on your answer sheet to indicate the correct answer to each of the questions
In developing countries, where three fourths of the world's population live, sixty percent of the people who can’t read and write are women. Being illiterate doesn’t mean they are not intelligent. It does mean it is difficult for them to change their lives. They produce more than half of the food. In Africa eighty percent of all agricultural work is done by women. There are many programs to help poor countries develop their agriculture. However, for years, these programs provided money and training for men. In parts of Africa, this is a typical day for a village woman. At 4:45 a.m, she gets up, washes and eats. It takes her a half hour to walk to the fields, and she works there until 3:00 p.m. She collects firewood and gets home at 4:00. She spends the next hour and a half preparing food to cook. Then she collects water for another hour. From 6:30 to 8:30 she cooks. After dinner, she spends an hour washing the dishes and her children. She goes to bed at 9:30 p.m. International organizations and programs run by developed nations are starting to help women, as well as men, improve their agricultural production. Governments have already passed some laws affecting women because of the UN Decade for Women. The UN report will affect the changes now happening in the family and society. 
What does the word "run" in the last paragraph mean?

A. move quickly

B. push

C. managed

D. organized

Câu 519 :
Why do people say women produce more than half of the food in Africa?

A. because 60 percent of women are illiterate.

B. because 80 percent of all agricultural work is done by women.

C. most women are not intelligent.

D. all are correct.

Câu 520 :
Why do people say that African women’s lives are hard?

A. Because these women are busy with housework.

B. Because they work all day in the fields.

C. Both A and B are correct.

D. Because they are illiterate.

Câu 521 :
A typical African woman spends _______ collecting firewood every day.

A. 3 hour

B. 2 hours

C. 1 hour

D. 4 hours

Câu 522 :
Which of these statements is NOT TRUE?

A. Women’s roles in the family and society are changing nowadays.

B. It is difficult for women to change their lives because of their illiteracy.

C. In the past only men in poor countries got benefit from many international programs.

D. Because they are illiterate, women are not intelligent.

Câu 523 :
By whom (what) was the Decade for Women organized?

A. by the United Nations Organization.

B. by developing countries

C. by the World Health Organization.

D. by many African countries.

Câu 524 :
The passage would most likely be followed by details about _______.

A. changes in life between men and women in the family and in the society.

B. negative effects of the UNO law

C. positive effects of the roles of women

D. women and men's roles in their family

Câu 525 :
Read the following passage and mark the letter A, B, C, or D on your answer sheet to indicate the correct answer to each of the questions
In Science, a theory is a reasonable explanation of observed events that are related. A theory often involves an imaginary model that helps scientists picture the way an observed event could be produced. A good example of this is found in the kinetic molecular theory, in which gases are pictured as being made up of many small particles that are in constant motion. After a theory has been publicized, scientists design experiments to test the theory. If observations confirm the scientists’ predictions, the theory is supported. If observations do not confirm the predictions, the scientists must search further. There may be a fault in the experiment, or the theory may have to be revised or rejected. Science involves imagination and creative thinking as well as collecting information and performing experiments. Facts by themselves arc not science. As the mathematician Jules Henri Poincare said: "Science is built with facts just as a house is built with bricks, but a collection of facts cannot be called science any more than a pile of bricks can be called a house." Most scientists start an investigation by finding out what other scientists have learned about a particular problem. After known facts have been gathered, the scientist comes to the part of the investigation that requires considerable imagination. Possible solutions to the problem are formulated. These possible solutions are called hypotheses. In a way, any hypothesis is a leap into the unknown. It extends the scientist's thinking beyond the known facts. The scientist plans experiments, performs calculations, and makes observations to test hypotheses. For without hypotheses, further investigation lacks purpose and direction. When hypotheses are confirmed, they are incorporated into theories. 
Which of the following is the main subject of the passage?

A. The importance of models in scientific theories.

B. The sorts of facts that scientists find most interesting.

C. The ways that scientists perform different types of experiments.

D. The place of theory and hypothesis in scientific investigation.

Câu 526 :
The word "related" in paragraph 1 is closest in meaning to _______.

A. described

B. identified

C. connected

D. completed

Câu 527 :
According to the second paragraph, a useful theory is one that helps scientists to _______.

A. observe events

B. publicize new findings

C. make predictions

D. find errors in past experiment

Câu 528 :
The word "supported" in paragraph 2 is closest in meaning to _______.

A. upheld

B. finished

C. adjusted

D. investigated

Câu 529 :
“Bricks” are mentioned in paragraph 3 to indicate how _______.

A. science is more than a collection of facts

B. scientific experiments have led to improved technology

C. mathematicians approach science

D. building a house is like performing experiments

Câu 530 :
In the fourth paragraph, the author implies that imagination is most important to scientists when they _______.

A. formulate possible solutions to a problem

B. evaluate previous work on a problem

C. close an investigation

D. gather known facts

Câu 531 :
In the last paragraph, what does the author imply is a major function of hypotheses?

A. Linking together different theories.

B. Communicating a scientist's thoughts to others.

C. Providing direction for scientific research.

D. Sifting through known facts.

Câu 532 :
Which of the following statements is supported by the passage?

A. Theories are simply imaginary models of past events.

B. A scientist's most difficult task is testing hypotheses.

C. A good scientist needs to be creative.

D. It is better to revise a hypothesis than to reject it.

Câu 533 :
Read the following passage and mark the letter A, B, C, or D on your answer sheet to indicate the correct answer to each of the questions
Although speech is the most advanced form of communication, there are many ways of communicating without using speech. Signals, signs, symbols, and gestures may be found in every known culture. The basic function of signal is to impinge upon the environment in such a way that it attracts attention, as, for example, the dots and dashes of a telegraph circuit. Coded to refer to speech, the potential for communication is very great. Less adaptable to the codification of words, signs also contain meaning in and of themselves. A stop sign or a barber pole conveys meaning quickly and conveniently. Symbols are more difficult to describe than either signals or signs because of their intricate relationship with the receiver’s cultural perceptions. In some cultures, applauding in a theater provides performers with an auditory symbol of approval. Gestures such as waving and handshaking also communicate certain cultural messages. Although signals, signs, symbols, and gestures are very useful, they do have a major disadvantage in communication. They usually do not allow ideas to be shared without the sender being directly adjacent to the receiver. Without an exchange of ideas, interaction comes to a halt. As a result, means of communication intended to be used for long distances and extended periods must be based upon speech. To radio, television, and the telephone, one must add fax, paging systems, electronic mail, and the Internet, and no one doubts but that there are more means of communication on the horizon. 
Which of the following would be the best title for the passage?

A. Gestures

B. Signs and signals

C. Speech

D. Communication

Câu 534 :
According to this passage, what is a signal?

A. A form of communication that interrupts the environment.

B. The most difficult form of communication to describe.

C. A form of communication which may be used across long distances.

D. The form of communication most related to cultural perception.

Câu 535 :
The word “it” in paragraph 1 refers to ____________.

A. way

B. environment

C. function

D. signal

Câu 536 :
The word “intricate” in paragraph 2 could best be replaced by ___________.

A. inefficient

B. complicated

C. historical

D. uncertain

Câu 537 :
Applauding was cited as an example of ____________.

A. a signal

B. a sign

C. a gesture

D. a symbol

Câu 538 :
Why were the telephone, radio, and television invented?

A. Because people were unable to understand signs, signals, and symbols.

B. Because people believed that signs, signals, and symbols were obsolete.

C. Because people wanted to communicate across long distances.

D. Because people wanted new forms of communication.

Câu 539 :
It may be concluded from this passage that ____________.

A. Only some cultures have signs, signals, and symbols.

B. Signs, signals, symbols, and gestures are forms of communication.

C. Symbols are very easy to define and interrupt.

D. Waving and handshaking are not related to culture.

Câu 540 :
Read the following passage and mark the letter A, B, C, or D on your answer sheet to indicate the correct answer to each of the questions
Smallpox was the first widespread disease to be eliminated by human intervention. Known as a highly contagious viral disease, it broke out in Europe, causing the deaths of millions of people until the vaccination was invented by Edward Jenner around 1800. In many nations, it was a terror, a fatal disease until very recently. Its victims suffer high fever, vomiting and painful, itchy, pustules that left scars. In villages and cities all over the world, people were worried about suffering smallpox.In May, 1966, the World Health Organization (WHO), an agency of the United Nations was authorized to initiate a global campaign to eradicate smallpox. The goal was to eliminate the disease in one decade. At the time, the disease posed a serious threat to people in more than thirty nations. Because similar projects for malaria and yellow fever had failed, few believed that smallpox could actually be eradicated but eleven years after the initial organization of the campaign, no cases were reported in the field.The strategy was not only to provide mass vaccinations but also to isolate patients with active smallpox in order to contain the spread of the disease and to break the chain of human transmission. Rewards for reporting smallpox assisted in motivating the public to aid health workers. One by one, each small-pox victim was sought out, removed from contact with others and treated. At the same time, the entire village where the victim had lived was vaccinated.By April of 1978 WHO officials announced that they had isolated the last known case of the disease but health workers continued to search for new cases for additional years to be completely sure. In May, 1980, a formal statement was made to the global community. Today smallpox is no longer a threat to humanity. Routine vaccinations have been stopped worldwide. 
Which of the following is the best title for the passage?

A. The eradication of smallpox

B. The World Health Organization

C. Infectious disease

D. Smallpox vaccinations

Câu 541 :
The word “contagious” is closest in meaning to ___________.

A. courteous

B. arresting

C. numerous

D. catching

Câu 542 :
The global community considered the smallpox ___________.

A. a minor illness

B. a deadly illness

C. a mental illness

D. a rare illness

Câu 544 :
According to the passage, what way was used to eliminate the spread of smallpox?

A. Vaccination of entire villages

B. Treatment of individual victims

C. Isolation of victims and mass vaccinations

D. Extensive reporting of outbreaks

Câu 545 :
How was the public motivated to help the health workers?

A. By educating them.

B. By rewarding them for reporting smallpox cases.

C. By isolating them from others.

D. By giving them vaccinations.

Câu 546 :
Which one of the statements doesn’t refer to smallpox?

A. Previous project had failed.

B. People are no longer vaccinated for it.

C. The WHO set up a worldwide campaign to eradicate the disease.

D. It was a fatal threat.

Câu 547 :
It can be inferred from the passage that ___________.

A. yellow fever have been reported this year.

B. no new cases of smallpox have been reported this year.

C. smallpox victims no longer die when they contact the disease.

D. smallpox is not transmitted from one person to another.

Câu 548 :
Read the following passage and mark the letter A, B, C, or D on your answer sheet to indicate the correct answer to each of the questions
The American type of football was developed in the 19th century from soccer and rugby football. Played by professionals, amateurs, college and high school students, or young children, football in American is one of the most popular sports besides basketball and baseball. It attracts millions of fans each fall and people are very supportive of their favourite teams. The football playing field of today is rectangular in shape and measures 100 yards long and 53.5 yards wide. White lines are painted on the playing field to mark off the distances to the end zone. The games is divided into four quarters, each fifteen minutes long. The first two quarters are known as the first half. There is a rest period between two halves which usually last about fifteen minutes. Each team has eleven players. Each team has offensive players who play when the team has possession of the ball and defensive players who play when the other team has the possession of the ball. Because of the body contact players have during the game, helmets are worn to protect their head and face area, whereas pads are worn to protect the shoulders, arms, and legs. Also, there are officials carrying whistles and flags to make certain that the rules of the game are followed during the game. The football is made of leather and is brown in colour. It is shaped much like an oval and has white rings near each end of the football. These rings help the players see the ball when it is thrown or someone is running with it. The eight stitches on the top of the football help the players to grip the ball when throwing or passing. The most famous game of the year is Super Bowl that is played in January or February. It is televised around the world and is watched by millions of people each year. 
What do officials do during the game of football?rrect answer to each of

A. build up excitement among the fans

B. supervise the game

C. take up tickets

D. spectate the game

Câu 549 :
As mentioned in the text, who are the most active when their team has the ball?

A. offensive players

B. defensive players

C. the officials

D. the fans

Câu 550 :
Playing American football is the most similar to playing .........

A. rugby football

B. basketball

C. baseball

D. volleyball

Câu 551 :
The American type of football was developed in the 19th century from soccer and rugby football. Played by professionals, amateurs, college and high school students, or young children, football in American is one of the most popular sports besides basketball and baseball. It attracts millions of fans each fall and people are very supportive of their favourite teams. The football playing field of today is rectangular in shape and measures 100 yards long and 53.5 yards wide. White lines are painted on the playing field to mark off the distances to the end zone. The games is divided into four quarters, each fifteen minutes long. The first two quarters are known as the first half. There is a rest period between two halves which usually last about fifteen minutes. Each team has eleven players. Each team has offensive players who play when the team has possession of the ball and defensive players who play when the other team has the possession of the ball. Because of the body contact players have during the game, helmets are worn to protect their head and face area, whereas pads are worn to protect the shoulders, arms, and legs. Also, there are officials carrying whistles and flags to make certain that the rules of the game are followed during the game. The football is made of leather and is brown in colour. It is shaped much like an oval and has white rings near each end of the football. These rings help the players see the ball when it is thrown or someone is running with it. The eight stitches on the top of the football help the players to grip the ball when throwing or passing. The most famous game of the year is Super Bowl that is played in January or February. It is televised around the world and is watched by millions of people each year.
What do pads help the players to protect?

A. their legs and arms

B. their heads

C. the whole body

D. their faces

Câu 552 :
Which is the most popular sport in the US?

A. Rugby football

B. Soccer

C. American football

D. It's not mentioned

Câu 553 :
When is the most famous football game held annually?

A. in the fall

B. January

C. February

D. January or February

Câu 554 :
Why are there white rings on each end of the football?

A. to mark off the distances to the end zone

B. to help players run

C. to help players score

D. to help players see the ball

Câu 555 :
The word "grip" in the passage means to ..........

A. avoid something

B. take something away

C. old something tightly

D. detect something

Câu 556 :
Read the following passage and mark the letter A, B, C, or D on your answer sheet to indicate the correct answer to each of the questions
One day in 1924, five men who were camping in the Cascade Mountains of Washington saw a group of huge apelike creatures coming out of the woods. They hurried back to their cabin and locked themselves inside. While they were in, the creatures attacked them by throwing rocks against the walls of the cabin. After several hours, these strange hairy giants went back into the woods. After this incident the men returned to the town and told the people of their adventure. However, only a few people accepted their story. These were the people who remembered hearing tales about footprints of an animal that walked like a human being. The five men, however, were not the first people to have seen these creatures called Bigfoot. Long before their experience, local Native Americans were certain that a race of apelike animals had been living in the neighboring mountain for centuries. They called these creatures Sasquatch. In 1958, workmen, who were building a road through the jungles of Northern California often found huge footprints in the earth around their camp. Then in 1967, Roger Patterson, a man who was interested in finding Bigfoot went into the Northern California jungles with a friend. While riding, they were suddenly thrown off from their horses. Patterson saw a tall apelike animal standing not far away. He managed to shoot seven rolls of film of the hairy creature before the animal disappeared in the hushes. when Patterson's film was shown to the public, not many people believed his story. In another incident, Richard Brown, a music teacher and also an experienced hunter spotted a similar creature. He saw the animal clearly through the telescopic lens of his rifle. He said the creature looked more like a human than an animal. Later many other people also found deep footprints in the same area. In spite of regular reports of sightings and footprints, most experts still do not believe that Bigfoot really exists. 
The word neighboring would BEST be replaced with ............

A. remote

B. nearby

C. far-off

D. far-away

Câu 557 :
Did the town people believe the story of the five men about their meeting with Bigfoot?

A. No, not everyone believed their story.

B. All the people believed what they said.

C. Some said the five men were making up their own story.

D. Only those who had heard the same tale the second time believed them.

Câu 558 :
Which of the following pairs is INCORRECT?

A. spotted - saw

B. creatures - animals

C. woods - jungles

D. huge - hairy

Câu 559 :
Who were the first people to have seen these apelike creatures before the five campers?

A. Richard Brown, a music teacher and a hunter.

B. Roger Patterson and his friend.

C. The local Native Americans.

D. The workers who built the road in the jungles of Northern California.

Câu 560 :
The BEST title for this passage would be ________

A. The adventures of Bigfoot.

B. The experts and the existence of Bigfoot.

C. The creature called Bigfoot.

D. The adventures of the five campers.

Câu 561 :
What did the five campers do when they saw a group of apelike creatures?

A. They threw rocks against the walls of their cabin to frighten the creatures away.

B. They attacked the creatures by throwing rocks at them.

C. They ran into the woods and hid there for several hours.

D. They quickly ran back into their cabin and locked the cabin door.

Câu 562 :
Who called the apelike creatures 'Sasquatch'?

A. Richard Brown

B. The local Native Americans

C. The five campers

D. Roger Patterson

Câu 563 :
Read the following passage and mark the letter A, B, C, or D on your answer sheet to indicate the correct answer to each of the questions
My family consists of four people. There's my father whose name is Jan, my mother whose name is Marie, my brother, Peter and of course, me. I have quite a large extended family as well but, only the four of us live together in our apartment in a block of flats. My father is fifty-two years old. He works as an accountant in an insurance company. He is tall and slim, has got short brown and gray hair and blue eyes. My father likes gardening very much as well as listening to music and reading books about political science. His special hobbies are bird watching and travelling. Now I’ll describe my mother and my brother. My mother is fortyseven and she works as a nurse in a hospital. She is small, and slim, has short brown hair and green eyes. She likes bird watching and travelling too, so whenever my parents are able to they go someplace interesting for nature watching. Since we have a cottage with a garden they both spend a lot of time there. My brother is sixteen. He is slim and has short brown hair and blue eyes. He also attends high school. He is interested in computers and sports like football and hockey. He also spends a lot of time with his friends. I have only one grandmother left still living. She is in pretty good health even at the age of seventyeight so she still lives in her own flat. I enjoy spending time with her when I can. Both my grandfathers died from cancer because they were smokers, which was really a great tragedy because I didn't get chance to know them. My other grandmother died just a few years ago. I also have a lot of aunts, uncles, and cousins. The cousin I’m closest to is my uncle’s daughter Pauline. We have a lot in common because we are both eighteen and so we are good friends. My parents have assigned me certain duties around the house. I don't mind helping out because everyone in a family should contribute in some way. I help with the washing up, the vacuuming and the shopping. Of course I also have to help keep my room tidy as well. My brother is responsible for the dusting and mopping. He also has to clean his own room. Even though my brother and I sometimes fight about who has to do what job, we are still very close. I am also very close to my parents and I can rely on them to help me. My parents work together to keep our home well maintained and it seem they always have a project or another that they are working on. They respect each other's opinions and even if they disagree they can always come to a compromise. I hope in the future that 1 have a family like ours. 
What does the writer's father do?

A. a cashier

B. an accountant

C. a receptionist

D. a writer

Câu 564 :
What does the writer’s mother look like?

A. She is slim and small.

B. She is small and has grey hair.

C. She has blue eyes.

D. She has long brown hair.

Câu 565 :
What does the writer’s brother do?

A. a computer programmer

B. a high school student

C. a college student

D. a football player

Câu 566 :
What happened to the writer’s grandfathers?

A. They died because they smoked too much.

B. They got lung cancer a few years ago.

C. They had to leave their own flat.

D. They had an accident and died.

Câu 567 :
The word "tragedy" in the third paragraph mostly means ........

A. bad luck

B. sudden accident

C. sad event

D. boring result

Câu 568 :
The word "assigned" in the fourth paragraph mostly means ......

A. allowed

B. appointed

C. forced

D. encouraged

Câu 569 :
Which of the following statements is NOT true?

A. The writer's mother likes travelling.

B. One of the writer's grandmothers is living with her.

C. The writer’s brother has to clean his own room.

D. The writer has a cousin whose age is the same as hers.

Câu 570 :
Read the following passage and mark the letter A, B, C, or D on your answer sheet to indicate the correct answer to each of the questions
In early civilizations, citizens were educated informally, usually within the family unit. Education meant simply learning to live. As civilizations became more complex, however, education became more formal, structured and comprehensive. Initial efforts of the ancient Chinese and Greek societies concentrated solely on the education of males. The post-Babylonian Jews and Plato were exceptions to this pattern. Plato was apparently the first significant advocate of the equality of the sexes. Women, in his ideal state, would have the same rights and duties and the same educational opportunities as men. This aspect of Platonic philosophy, however, had little or no effect on education for many centuries, and the concept of a liberal education for men only, which had been espoused by Aristotle, prevailed. In ancient Rome, the availability of an education was gradually extended to women, but they were taught separately from men. The early Christians and medieval Europeans continued this trend, and singlesex schools for the privileged classes prevailed through the Reformation period. Gradually, however, education for women on a separate but equal basis to that provided for men was becoming a clear responsibility of society. Martin Luther appealed for civil support of schools for all children. At the Council of Trent in the 16th century, the Roman Catholic Church encouraged the establishment of free primary schools for children of all classes. The concept of universal primary education, regardless of sex, had been born, but it was still in the realm of the single-sex school. In the late 19th and early 20th centuries, co-education became a more widely applied principle of educational philosophy. In Britain, Germany, and the Soviet Union the education of boys and girls in the same classes became an accepted practice. Since World War II, Japan and the Scandinavian countries have also adopted relatively universal co-educational systems. The greatest negative reaction to coeducation has been felt in the teaching systems of the Latin countries, where the sexes have usually been separated at both primary and secondary levels, according to local conditions. A number of studies have indicated that girls seem to perform better overall and in science in particular in single-sex classes: during the adolescent years, pressure to conform to stereotypical female gender roles may disadvantage girls in traditionally male subjects, making them reluctant to volunteer for experimental work while taking part in lessons. In Britain, academic league tables point to high standards achieved in girls' schools. Some educationalists therefore suggest segregation of the sexes as a good thing, particularly in certain areas, and a number of schools are experimenting with the idea. 
Ancient education generally focused its efforts on _______.

A. male learners

B. both sexes

C. female learners

D. young people only

Câu 571 :
Education in early times was mostly aimed at _______.

A. teaching skills

B. learning to live

C. learning new lifestyles

D. imparting survival skills

Câu 572 :
The first to support the equality of the sexes was _______.

A. the Chinese

B. the Greek

C. Plato

D. the Jews

Câu 573 :
The word "informally" in this context mostly refers to an education occurring _______.

A. in classrooms

B. outside the school

C. in a department

D. ability

Câu 574 :
When education first reached women, they were _______.

A. locked up in a place with men

B. isolated from normal life

C. deprived of opportunities

D. separated from men

Câu 575 :
When the concept of universal primary education was introduced, education _______.

A. was given free to all

B. was intended for all the sexes

C. focused on imparting skills

D. was intended to leave out female learners

Câu 576 :
The word "espouse” is contextually closest in meaning to "_______"

A. to support

B. to put off

C. to give

D. to induce

Câu 577 :
Co-education was negatively responded to in _______.

A. Japan

B. the Scandinavian countries

C. South American countries

D. conservative countries

Câu 578 :
Read the following passage and mark the letter A, B, C, or D on your answer sheet to indicate the correct answer to each of the questions
Ever since humans have inhabited the earth, they have made use of various forms of communication. Generally, this expression of thoughts and feelings has been in the form of oral speech. When there is a language barrier, communication is accomplished through sign language in which motions stand for letters, words and ideas. Tourists, the deaf, and the mute have no resort to this form of expression. Many of these symbols of the whole words are very picturesque and exact and can be used internationally: spelling, however, can not. Body language transmits ideas or thoughts by certain actions, either intentionally or unintentionally. A wink can be a way of flirting or indicating that the party is only joking. A nod signifies approval, while shaking the head indicates a negative reaction. Other forms of nonlinguistic language can be found in Braille (a system of raised dots read with fingertips), signal flags, Morse code, and smoke signals. Road maps and picture signs also guide, warn and instruct people. While verbalization is the most common form of language, other systems and techniques also express human thoughts and feelings. 
Which of the following best summarizes this passage?

A. Everybody uses only one form of communication.

B. When language is a barrier, people will find other forms of communication.

C. Nonlinguistic language is invaluable to foreigners.

D. Although other forms of communication exist, verbalization is the fastest.

Câu 579 :
The word “these" is the first passage refers to_________

A. tourists

B. the deaf and the mute

C. thoughts and feelings

D. sign language motions

Câu 580 :
All of the following statements are true EXCEPT______________

A. the deaf and the mute use an oral form of communication

B. verbalization is the most common form of communication

C. there are many forms of communication in existence today

D. ideas and thoughts can be transmitted by body language

Câu 581 :
Which form other than oral speech would be most commonly used among the blind people?

A. Body language

B. Picture signs

C. Braille

D. Signal flags

Câu 584 :
People need to communicate in order to__________

A. keep from reading with their fingertips

B. create language barriers

C. express thoughts and feelings

D. be picturesque and exact

Câu 585 :
What is the best title for the passage?

A. The Importance of Sign Language

B. Picturesque symbols of communication

C. Ways of expressing feelings

D. Many forms of communication

Câu 586 :
Read the following passage and mark the letter A, B, C, or D on your answer sheet to indicate the correct answer to each of the questions
Many people believe that watching television has resulted in lower reading standards in schools. However, the link between television and printed books is not as simple as that. In many cases, television actually encourages people to read; for example, when a book is turned into a TV series, its sales often go up. One study of this link examined six-year-old children who were viewing a special series of 15-minute programs at school. The series was designed to encourage love of books, as well as to develop the basic mechanical skills of reading. Each program is an animated film of a children's book. The story is read aloud and certain key phrases from the book appear on the screen, beneath the picture. Whenever a word is read, it is also highlighted on the TV screen. One finding was that watching these programs was very important to the children. If anything prevented them from seeing a program, they were very disappointed. What's more, they wanted to read the books which the different parts of the series were based on. The programs also gave the children more confidence when looking at these books. As a result of their familiarity with the stories, they would sit in pairs and read the stories aloud to each other. On each occasion, the children showed great sympathy when discussing a character in a book because they themselves had been moved when watching the character on television. 
When does television encourage people to read?

A. When children are watching an animated film on TV

B. When a book is made into a TV series

C. When there are no interesting programs on TV

D. When a book appears on the screen

Câu 587 :
How did children feel when viewing a special series of 15-minute program at school?

A. They felt confident.

B. They felt bored.

C. They felt disappointed.

D. They felt excited.

Câu 588 :
What is the main idea of the passage?

A. The ways of children reading books and watching

B. The simple link between television and books

C. The influence of television on reading books

D. The advantage of books over television

Câu 589 :
What advantage did children have from this program?

A. They became more interested in watching Triệu-Vũ

B. They became more sympathetic.

C. They made more friends.

D. They read books with more interest and confidence

Câu 590 :
What kind of film is each program?

A. An animated film of a children's book

B. A story film of a children's book

C. An educational film

D. A documentary film of children's life

Câu 591 :
Who were the subjects of the study?

A. all people

B. Six-year-old children

C. All school children

D. Adolescents

Câu 592 :
The series was designed _____________ 

A. to examine children’s mind 

B. to promote the children’s love of books 

C. to develop their listening skill 

D. to sale more books

Câu 593 :
Read the following passage and mark the letter A, B, C, or D on your answer sheet to indicate the correct answer to each of the questions
What is the main topic of this passage?
Over the past 600 years, English has grown from a language of few speakers to become the dominant language of international communication. English as we know it today emerged around 1350, after having incorporated many elements of French that were introduced following the Norman invasion of 1066. Until the 1600s, English was, for the most part, spoken only in England and had not extended even as far as two centuries, English began to spread around the globe as a result of exploration, trade (including slave trade), colonization, and missionary work. Thus, small enclaves of English speakers became established and grew in various parts of the world. As these communities proliferated, English gradually became the primary language of international business, banking and diplomacy. Currently, about 80 percent of the information stored on computer systems worldwide is English. Two thirds of the world’s science writing is in English, and English is the main language of technology, advertising, media, international airports, and air traffic controllers. Today there are more than 700 million English users in the world, and over half of these are nonnative speakers, constituting the largest number of non-native users than any other language in the world..

A. The number of non-native users of English.

B. The French influence on the English Language.

C. The expansion of English as an international language.

D. The use of English for science and technology.

Câu 594 :
In the first paragraph, the word “emerged” is closest in meaning to____.

A. appeared

B. disappeared

C. frequented

D. engaged

Câu 595 :
In the first paragraph, the word “elements” is closest in meaning to____.

A. declaration

B. features

C. curiosities

D. customs

Câu 596 :
Approximately when did English begin to be used beyond England?

A. In 1066

B. around 1350

C. before 1600

D. after 1600

Câu 597 :
According to the passage, all of the following contributed to the spread of English around the world _________ .

A. the slave trade

B. the Norman invasion

C. missionaries

D. colonization

Câu 599 :
According to the passage, approximately how many non-native users of English are there in the world today?

A. a quarter million

B. half a million

C. 350 million

D. 700 million

Câu 600 :
Read the following passage and mark the letter A, B, C, or D on your answer sheet to indicate the correct answer to each of the questions
Experts in climatology and other scientists are becoming extremely concerned about the changes to our climate which are taking place. Admittedly, climate changes have occurred on our planet before. For example, there have been several ice ages or glacial periods. These climatic changes, however, were different from the modern ones in that they occurred gradually and, as far as we know, naturally. The changes currently being monitored are said to be the result not of natural causes, but of human activity. Furthermore, the rate of change is becoming alarmingly rapid. The major problem is that the planet appears to be warming up. According to some experts, this warming process, known as global warming, is occurring at a rate unprecedented in the last 10,000 years. The implications for the planet are very serious. Rising global temperatures could give rise to such ecological disasters as extremely high increases in the incidence of flooding and of droughts. These in turn could have a harmful effect on agriculture. It is thought that this unusual warming of the Earth has been caused by so-called greenhouse gases, such as carbon dioxide, being emitted into the atmosphere by car engines and modern industrial processes, for example. Such gases not only add to the pollution of the atmosphere, but also create a greenhouse effect, by which the heat of the sun is trapped. This leads to the warming up of the planet. Politicians are also concerned about climate change and there are now regular summits on the subject, attended by representatives from around 180 of the world’s industrialized countries. Of these summits, the most important took place in Kyotoin Japanin 1997. There it was agreed that the most industrialized countries would try to reduce the volume of greenhouse gas emissions and were given targets for this reduction of emissions. It was also suggested that more forests should be planted to create so-called sinks to absorb greenhouse gases. At least part of the problem of rapid climate change has been caused by too drastic deforestation. Sadly, the targets are not being met. Even more sadly, global warnings about climate changes are often still being regarded as scaremongering. 
According to the passage, in what way did the climate changes in the ice ages differ from the modern ones?

A. They occurred naturally over a long period of time.

B. They were partly intended.

C. They were wholly the result of human activity.

D. They were fully monitored by humans.

Câu 601 :
Experts in climatology and other scientists are becoming extremely concerned about the changes to our climate which are taking place. Admittedly, climate changes have occurred on our planet before. For example, there have been several ice ages or glacial periods. These climatic changes, however, were different from the modern ones in that they occurred gradually and, as far as we know, naturally. The changes currently being monitored are said to be the result not of natural causes, but of human activity. Furthermore, the rate of change is becoming alarmingly rapid. The major problem is that the planet appears to be warming up. According to some experts, this warming process, known as global warming, is occurring at a rate unprecedented in the last 10,000 years. The implications for the planet are very serious. Rising global temperatures could give rise to such ecological disasters as extremely high increases in the incidence of flooding and of droughts. These in turn could have a harmful effect on agriculture. It is thought that this unusual warming of the Earth has been caused by so-called greenhouse gases, such as carbon dioxide, being emitted into the atmosphere by car engines and modern industrial processes, for example. Such gases not only add to the pollution of the atmosphere, but also create a greenhouse effect, by which the heat of the sun is trapped. This leads to the warming up of the planet. Politicians are also concerned about climate change and there are now regular summits on the subject, attended by representatives from around 180 of the world’s industrialized countries. Of these summits, the most important took place in Kyotoin Japanin 1997. There it was agreed that the most industrialized countries would try to reduce the volume of greenhouse gas emissions and were given targets for this reduction of emissions. It was also suggested that more forests should be planted to create so-called sinks to absorb greenhouse gases. At least part of the problem of rapid climate change has been caused by too drastic deforestation. Sadly, the targets are not being met. Even more sadly, global warnings about climate changes are often still being regarded as scaremongering.
The word “alarmingly” in paragraph 2 is closest in meaning to ____.

A. disapprovingly

B. disappointingly

C. surprisingly

D. worryingly

Câu 602 :
According to the passage, agriculture could ____.

A. make the global warming more serious

B. be indirectly affected by the global temperature rises

C. give rise to many ecological disasters

D. be directly damaged by the rises in global temperature

Câu 603 :
Greenhouse gases cause the warming up of the Earth because they ____.

A. are emitted by car engines

B. trap heat from the sun

C. do not add to atmosphere pollution

D. are unusual gases

Câu 604 :
According to the passage, 1997 witnessed ____.

A. the largest number of summits on the subject of climate change

B. the most important summit on climate change taking place in Kyoto, Japan

C. the highest attendance by representatives from 180 industrialised countries

D. widespread concern about climate change

Câu 605 :
It can be inferred from the passage that the countries which are mainly responsible for global warming are ____.

A. countries with the warmest climate

B. developing countries

C. developed countries

D. the most industrialized countries

Câu 606 :
The word “There” in paragraph 5 refers to ____.

A. the world’s industrialized countries

B. regular summits on climate change

C. the most industrialized countries

D. the 1997 summit in Kyoto, Japan

Câu 607 :
Which of the following is NOT true, according to the passage?

A. Carbon dioxide is one of the gases that may cause the so-called greenhouse effect.

B. The so-called sinks created by forests can absorb greenhouse gases.

C. The problem of rapid climate change has been caused mainly by deforestation.

D. Politicians are among those who are concerned about climate change.

Câu 608 :
Read the following passage and mark the letter A, B, C, or D on your answer sheet to indicate the correct answer to each of the questions
Many flowering plants woo insect pollinators and gently direct them to their most fertile blossoms by changing the color of individual flowers from day to day. Through color cues, the plant signals to the insect that it would be better off visiting one flower on its bush than another. The particular hue tells the pollinator that the flower is full of far more pollen than are neighboring blooms. That nectar-rich flower also happens to be fertile and ready to disperse its pollen or to receive pollen the insect has picked up from another flower. Plants do not have to spend precious resources maintaining reservoirs of nectar in all their flowers. Thus, the color-coded communication system benefits both plants and insects. For example, on the lantana plant, a flower starts out on the first day as yellow, when it is rich with pollen and nectar. Influenced by an asyet-unidentified environmental signal, the flower changes color by triggering the production of the pigment anthromyacin. It turns orange on the second day and red on the third. By the third day, it has no pollen to offer insects and is no longer fertile. On any given lantana bush, only 10 to 15 per cent of the blossoms are likely to be yellow and fertile. But in tests measuring the responsiveness of butterflies, it was discovered that the insects visited the yellow flowers at least 100 times more than would be expected from haphazard visitation. Experiments with paper flowers and painted flowers demonstrated that the butterflies were responding to color cues rather than, say, the scent of the nectar. In other types of plants, blossoms change from white to red, others from yellow to red, and so on. These color changes have been observed in some 74 families of plants. 
The first paragraph of the passage implies that insects benefit from the color-coded communication system because __________

A. they can gather pollen efficiently.

B. the colors hide them from predators.

C. the bright colors attract fertile females.

D. other insect species can not understand the code.

Câu 609 :
The word “woo” is closest in meaning to __________

A. frighten

B. trap

C. deceive

D. attract

Câu 610 :
The word “it” refers to __________ 

A. a plant 

B. a blossom 

C. an insect 

D. a signal

Câu 611 :
Which of the following describes the sequence of color changes that lantana blossom undergo?

A. Red to purple

B. Yellow to orange to red

C. Red to yellow to white

D. White to red

Câu 612 :
The word “triggering” is closest in meaning to __________

A. maintaining

B. renewing

C. activating

D. limiting

Câu 613 :
The passage implies that insects would be most attracted to lantana blossoms __________

A. on the first day that they bloom

B. after they produce anthromyacin

C. when they turn orange

D. on the third day that they bloom

Câu 614 :
The word “haphazard” is closest in meaning to which of the following?

A. Fortunate

B. Expected

C. Dangerous

D. Random

Câu 615 :
The topic of this passage is ___________. 

A. infants and allergies

B. food and nutrition 

C. eactions to foods

D. a good diet

Câu 616 :
According to the passage, the difficulty in diagnosing allergies to foods is due to ___________.

A. the vast number of different foods we eat

B. lack of a proper treatment plan

C. the similarity of symptoms of the allergy to other problems

D. the use of prepared formula to feed babies

Câu 617 :
The word "symptoms" is closest in meaning to ___________.

A. prescriptions

B. diet

C. diagnosis

D. indications

Câu 618 :
What can be inferred about babies from this passage?

A. They can eat almost anything.

B. They should have a carefully restricted diet as infants.

C. They gain little benefit from being breast fed.

D. They may become hyperactive if fed solid food too early.

Câu 619 :
The author states that the reason that infants need to avoid certain foods related to allergies has to do with the infant's ___________ .

A. lack of teeth

B. poor metabolism

C. inability to swallow solid foods

D. underdeveloped

Câu 620 :
The word "these" refers to ___________. 

A. food additives 

B. food colorings 

C. innutritious foods 

D. foods high in salicylates

Câu 621 :
Which of the following was a suggested treatment for migraines in the passage?

A. Using Vitamin B in addition to a good diet

B. Avoiding all Oriental foods

C. Getting plenty of sodium nitrate

D. Eating more ripe bananas

Câu 622 :
According to the article the Feingold diet is NOT ___________.

A. available in book form

B. verified by researchers as being consistently effective

C. beneficial for hyperactive children

D. designed to eliminate foods containing certain food additives

Câu 623 :
Read the following passage and mark the letter A, B, C, or D on your answer sheet to indicate the correct answer to each of the questions
Fungi are a group of organisms that, despite being plants, have no leaves or flowers. In fact, fungi do not even share the green colour that most other plants display. Scientists estimate that there are over 1.5 million different species of fungi in the world. Though, to date, only 100.000 have been identified, leaving many more that have not been found. One interesting feature of fungi is that they often interact with oilier organisms in order to survive. These relationships are at times beneficial to both organisms. Other times, the fungus benefits without causing harm to the other organism. Many types of fungus have beneficial relationships with plants. Initially many gardeners would be concerned to know that their plants were colonised by a fungus. This is because some fungi can cause plants to die. In fact, the Irish potato famine was caused by a fungus that killed entire crops of potatoes. However, many plants actually depend on certain types of fungi to help it stay healthy. Fungi are important to plants because they help plants absorb more minerals from the soil than they could on their own. The reason for this has to do with how fungi obtain food. Unlike green plants, fungi cannot make their own food. They must absorb their food. When the fungi absorb minerals from the soil, they draw the nutrients closer to the roots of the plants, so the plant is able to use them as well. The fungus also benefits from this relationship. Using the minerals from the soil, as well as sunlight, the plants is able to produce sugars and other nutrients. Then the fungus absorbs the nutrients from plant roots and uses them to survive. Not all relationships are beneficial for both organisms: in some interactions, only the fungus benefits. Still, for some fungus species, contact with other organisms is essential. And though the fungi do not provide any benefits for the other organism, they do not harm it either. One example of this is a species called Pilobolus. This fungus relies on other animals to help it reproduce. The Pilobolus grows in animal dung. When it becomes mature, it shoots its spores away from the dung pile. The spores land in the grass where cows graze. The spores are consumed by the animal but do not grow while inside the stomach. They travel through the body of the animal until they are passed and deposited in another area, where they continue to grow. 
It can be inferred from paragraph 1 that___________

A. Many species of fungi have yet to be discovered

B. Fungi do harm to the organism they interact with

C. Fungi can have green or dull brown colors

D. There are 100,000 species of fungi left no identity

Câu 624 :
The word "absorb" in the passage 2 is closest in meaning to_________

A. consume

B. get in

C. deprive

D. take in

Câu 625 :
According to paragraph 2, which of the following is not true about fungus growing on or near plants?

A. It can help maintain in the plant’s health

B. It neither benefits nor harms the plant

C. It is sometimes dangerous for plants

D. It grows near the roots of the plant

Câu 626 :
The word "them" in the passage 2 refers to_________.

A. minerals

B. plant roots

C. fungi

D. nutrients

Câu 627 :
The author discusses cows and horses in paragraph 3 in order to_________

A. illustrate how fungus can be harmful

B. explain how a type of fungus reproduces

C. give examples of animals that eat fungu

D. show that fungus can be found anywhere

Câu 628 :
According to the passage 3, Pilobolus fungus___________

A. doesn’t benefit from its relationship with animals

B. needs other organisms in order to reproduce

C. matures inside the stomachs of cows and horses

D. causes horses and cows to become ill

Câu 630 :
Read the following passage and mark the letter A, B, C, or D on your answer sheet to indicate the correct answer to each of the questions
Traditional methods of teaching no longer suffice in this techological world. Currently there are more than 100,000 computers in schoolrooms in the United States. Students mediocre and bright alike, from the first stage through high school not only are not intimidated by my computers, but have become avid participants in the computer epoch. Kids operating computer implement their curriculum with great versatility. A music student can program musical notes so that the computer will play Beethoven or the Beatles. For a biology class, the computer can produce a picture of the intricate envisage human biology in a profound way. A nuclear reactor is no longer an enigma to students who can see its workings in minute detail on a computer. In Wisconsi, the Chippewa Indians are studying their ancient and almost forgotten language with the aid of a computer. More commonly, the computer is used for drilling math and language concepts so that youngsters may learn at their own speed without trying the patience of their human teachers. The simplest computers aid the handicapped, who learn more rapidly from the computer than from humans. Once irksome, remedial drills and exercises now on computer are conducive to learning because the machine responds to correct answers with praise and to incorrect answers with frowns and even an occasional tear. Adolescents have become so exhilarated by computers that they have developed their own jargon, easily understood by their peers but leaving their disconcerted parents in the dark. They have shown so much fervor for computer that they have formed computer clubs, beguile their leisure hours in computer stores, and even attend computer camps. A Boy Scout can get a computer merit badge. One ingenious young student devised a computer game for Atari will earn him $100,000 in royalties This is definitely the computer age. Manufacturers of computers are presently getting tax write – offs for donating equipment to colleges and universities and are pushing for legislation to obtain further deductions for contributions to elementary and high school. Furthermore, the price of computer for home or office is being sold for less than $100. At that price every class in the country will soon have computer kids. 
The expression “traditional methods of teaching” is the first sentence refers to ______

A. technological methods of teaching

B. teachers who punish students for not learning 

C. teachers, textbooks and class drills 

D. teaching the three Rs i.e readung, wrting and arithmetic

Câu 631 :
In order to operate a computer, a student does not have to be _______

A. in grade school

B. versatile

C. especially bright

D. musica

Câu 632 :
Today’s students with the aid of computer__________

A. try to confuse their parents

B. have more trouble learning

C. can understand more complex concepts

D. build nuclear reactors

Câu 633 :
When the author says parents are “left in the dark”, he means that they ____________

A. didn’t pay the electrical bill

B. don’t understand

C. have deficient eyesight

D. go out at night

Câu 634 :
Computers are used most for ________ 

A. Boy Scout merit badges 

B. scientific subjects 

C. language instruction 

D. drills and exercises

Câu 635 :
According to the passage, one exceedingly clever student has ___________

A. overcome a handicap

B. learned an Idian language

C. invented a video game

D. played music on a computer

Câu 636 :
It can be inferred from the passage that _____________

A. computers are difficult to operate 

B. students today have to be smarter than their parents 

C. computer make learning today easier than it was in the past 

D. anyone who can operate a computer is an genius

Câu 637 :
The price of a computer is now _____________ 

A. within the range of most schools’ budget 

B. too high for most schools 

C. higher than ever before

C. higher than ever before D. preventing school from buying computers

Câu 638 :
Read the following passage and mark the letter A, B, C, or D on your answer sheet to indicate the correct answer to each of the questions
It is commonly believed in the United States that school is where people go to get an education. Nevertheless, it has been said that today children interrupt their education to go to school. The distinction between schooling and education implied by this remark is important. Education is much more open-ended and all-inclusive than schooling. Education knows no bounds. It can take place anywhere, whether in the shower or on the job, whether in a kitchen or on a tractor. It includes both the formal learning that takes place in schools and the whole universe of informal learning. The agents of education can range from a revered grandparent to the people debating politics on the radio, from a child to a distinguished scientist. Whereas schooling has a certain predictability, education quite often produces surprises. A chance conversation with a stranger may lead a person to discover how little is known of other religions. People are engaged in education from infancy on. Education, then, is a very broad, inclusive term. It is a lifelong process, a process that starts long before the start of school, and one that should be an integral part of one's entire life. Schooling, on the other hand, is a specific, formalized process, whose general pattern varies little from one setting to the next. Throughout a country, children arrive at school at approximately the same time, take assigned seats, are taught by an adult, use similar textbooks, do homework, take exams, and so on. The slices of reality that are to be learned, whether they are the alphabet or an understanding of the workings of government, have usually been limited by the boundaries of the subject being taught. For example, high school students know that they are not likely to find out in their classes the truth about political problems in their communities or what the newest filmmakers are experimenting with. There are definite conditions surrounding the formalized process of schooling. 
What does the author probably mean by using the expression “children interrupt their education to go to school” (lines 2-3) ?

A. Going to several different schools is educationally beneficial.

B. School vacations interrupt the continuity of the school year.

C. Summer school makes the school year too long.

D. All of life is an education.

Câu 639 :
The word “bounds” in line 5 is closest in meaning to ........

A. rules

B. experience

C. limits

D. exceptions

Câu 640 :
The word “ integral” in line 13 is closest in meaning to .....

A. equitable

B. profitable

C. pleasant

D. essential

Câu 641 :
The word “they” in line 17 refers to ...... 

A. slices of reality

B. similar textbooks 

C. boundaries 

D. seats

Câu 642 :
The phrase “For example,” line 19, introduces a sentence that gives examples of ........

A. similar textbooks

B. the results of schooling

C. the workings of a government

D. the boundaries of classroom subjects

Câu 643 :
The passage supports which of the following conclusions?

A. Without formal education, people would remain ignorant.

B. Education systems need to be radically reformed.

C. Going to school is only part of how people become educated.

D. Education involves many years of professional training.

Câu 644 :
The passage is organized by ........... 

A. listing and discussing several educational problems 

B. contrasting the meanings of two related words 

C. narrating a story about excellent teachers 

D. giving examples of different kinds of schools

Câu 645 :
Read the following passage and mark the letter A, B, C, or D on your answer sheet to indicate the correct answer to each of the questions
Orchids are unique in having the most highly developed of all blossoms, in which the usual male and female reproductive organs are fused in a single structure called the column. The column is designed so that a single pollination will fertilize hundreds of thousands, and in some cases millions, of seeds, so microscopic and light they are easily carried by the breeze. Surrounding the column are three sepals and three petals, sometimes easily recognizable as such, often distorted into gorgeous, weird, but always functional shapes. The most noticeable of the petals is called the labellum, or lip. It is often dramatically marked as an unmistakable landing strip to attract the specific insect the orchid has chosen as its pollinator. To lure their pollinators from afar, orchids use appropriately intriguing shapes, colors, and scents. At least 50 different aromatic compounds have been analyzed in the orchid family, each blended to attract one, or at most a few, species of insects or birds. Some orchids even change their scents to interest different insects at different times. Once the right insect has been attracted, some orchids present all sorts of one-way obstacle courses to make sure it does not leave until pollen has been accurately placed or removed. By such ingenious adaptations to specific pollinators, orchids have avoided the hazards of rampant crossbreeding in the wild, assuring the survival of species as discrete identities. At the same time they have made themselves irresistible to collectors. 
What does the passage mainly discuss?

A. Birds

B. Insects

C. Flowers

D. Perfume

Câu 646 :
The orchid is unique because of ......

A. the habitat in which it lives

B. the structure of its blossom

C. the variety of products that can be made from it

D. the length of its life

Câu 647 :
The word "fused" in line 2 is closest in meaning to .........

A. combined

B. hidden

C. fertilized

D. produced

Câu 648 :
How many orchid seeds are typically pollinated at one time?

A. 200

B. 2,000

C. 20,000

D. 200,000

Câu 649 :
Which of the following is a kind of petal?

A. The column

B. The sepal

C. The stem

D. The labellum

Câu 650 :
The “labellum” (line 7) is most comparable to ........

A. a microscope

B. an obstacle course

C. an airport runway

D. a racetrack

Câu 651 :
Orchids are unique in having the most highly developed of all blossoms, in which the usual male and female reproductive organs are fused in a single structure called the column. The column is designed so that a single pollination will fertilize hundreds of thousands, and in some cases millions, of seeds, so microscopic and light they are easily carried by the breeze. Surrounding the column are three sepals and three petals, sometimes easily recognizable as such, often distorted into gorgeous, weird, but always functional shapes. The most noticeable of the petals is called the labellum, or lip. It is often dramatically marked as an unmistakable landing strip to attract the specific insect the orchid has chosen as its pollinator. To lure their pollinators from afar, orchids use appropriately intriguing shapes, colors, and scents. At least 50 different aromatic compounds have been analyzed in the orchid family, each blended to attract one, or at most a few, species of insects or birds. Some orchids even change their scents to interest different insects at different times. Once the right insect has been attracted, some orchids present all sorts of one-way obstacle courses to make sure it does not leave until pollen has been accurately placed or removed. By such ingenious adaptations to specific pollinators, orchids have avoided the hazards of rampant crossbreeding in the wild, assuring the survival of species as discrete identities. At the same time they have made themselves irresistible to collectors.
The word "their" in line 12 refers to .......

A. orchids

B. birds

C. insects

D. species

Câu 652 :
The word "discrete" in line 16 is closest in meaning to .....

A. complicated

B. separate

C. inoffensive

D. functional

Câu 653 :
The text is mainly about __________. 

A. a strange man 

B. an unusual party 

C. how people enjoyed themselves in the 18th century

D. how roller skating began

Câu 654 :
The word “astonished” in paragraph 4 can be best replaced by__________

A. amazed

B. tired

C. polite

D. embarrassed

Câu 655 :
Merlin put wheels under his shoes in order to __________

A. arrive at the party sooner

B. show his skill in walking on wheels

C. test his invention

D. impress the party guests

Câu 656 :
The word “ball” in paragraph 2 probably means ............

A. party

B. round object

C. match

D. game

Câu 657 :
People thought Merlin was a dreamer because he__________.

A. was a gifted musician

B. often gave others surprises

C. invented the roller skates

D. was full of imagination

Câu 658 :
What is the main point the writer is trying to make in the last paragraph?

A. Merlin got himself into trouble.

B. Merlin succeeded beyond expectation.

C. The roller skates needed further improvement.

D. The party guests took Merlin for a fool.

Câu 659 :
The word “These” in paragraph 3 refers to__________.

A. wheels

B. shoes

C. roller skates

D. different ways

Câu 660 :
Read the following passage and mark the letter A, B, C, or D on your answer sheet to indicate the correct answer to each of the questions
Technology has utterly transformed our ability to communicate with each other. Linking to each other both literally and figuratively, many of us connect through cell phones, email, instant messaging, blogs, and networking web sites, yet we may be less connected to each other than we think. According to a study, Americans are becoming increasingly socially isolated. The study reveals, for example, that one quarter of Americans say that they have no one to discuss important personal issues with, and that the number of close friends that American have has dropped from three to two. Meanwhile, the Boston Globe reports that this spreading isolation is experienced more sharply among those with less education, people of color, and older Americans. Unsurprisingly, those who are young, white, and well educated tend to have stronger social networks. From my own experience I have to say that I’ve never felt more connected, thanks to a web of friends, family, and colleagues. One of my closest friends is someone I met through an online discussion group who lives hundreds of miles away from me. We have met faceto-face only twice, yet our regular electronic correspondence and cell phone calls sustain our close friendship. And, speaking of blogging, my blog has introduced me to people I would never have met otherwise and has led to enduring and important friendships. On the other hand, I recently saw a scene unfold that proved to me how deeply disconnected we as Americans have become. I had just wrapped up a presentation on mediation at a family therapy center. As I was leaving, I noticed a mother and her teenage son who had just completed their session with their family therapist. After making their next appointment, they both took out their cell phones, placed calls, and began loud conversations with whoever was on the other end. I walked out behind them to the parking lot to my car. They both jumped into their SUV, and, as I saw them drive off, they were still talking on their cell phones. But, alas, not to each other. 
Which of the following has nothing to do with the isolation among Americans?

A. Skin color

B. Gender

C. Education

D. Age

Câu 661 :
What can be said about the mother and son in the passage?

A. There was not much communication between them.

B. They came for therapy in the same car.

C. They were quite close to each other.

D. They preferred talking to each other on cell phones.

Câu 662 :
The writer uses the phrase “On the other hand” in the last paragraph to .......

A. show something similar

B. give a different opinion

C. introduce a new idea

D. prove that something is wrong

Câu 663 :
The word “those” in paragraph 2 refers to __________.

A. network

B. networks

C. general people

D. close friends

Câu 664 :
What does the passage lead you to believe?

A. Technology plays a bigger role in American society.

B. Americans are more socially isolated today.

C. Americans don’t make good use of technology.

D. Americans are more socially connected today.

Câu 665 :
Which of the following is NOT true, according to the passage?

A. Modern people link to each other through telecommunications.

B. Americans seem to have fewer and fewer close friends.

C. The writer has never met some of his important friends.

D. Users of SUV enjoy talking to each other on the cell phones.

Câu 666 :
The word “sustain” in paragraph 3 is closest in meaning to__________.

A. make something continue to exist

B. let someone suffer a lot

C. making something remain secure

D. support someone emotionally

Câu 667 :
The writer himself __________ . 

A. has a wide range of friends because of technology 

B. has a wide range of friends because of technology 

C. can only find true friendship through the Internet 

D. can’t make true friends in actual life

Câu 668 :
The purpose of the passage is to __________. 

A. explain how Lake Vostok was discovered 

B. provide satellite data concerning Antarctica 

C. discuss future plans for Lake Vostok 

D. present an unexpected aspect of Antarctica’s geography

Câu 669 :
The word “lies” in the first paragraph could be best be replaced by ______.

A. sleeps

B. sits

C. tells falsehoods

D. inclines

Câu 670 :
What is true of Lake Vostok? 

A. It is completely frozen 

B. It is not a saltwater lake 

C. It is beneath a thick slab of ice 

D. It is heated by the sun

Câu 671 :
Which of the following is closest in meaning to “ Frigid” in the first paragraph?

A. extremely cold

B. never changing

C. quite harsh

D. rarely recorded

Câu 672 :
All of the following are true about the 1970 survey of Antarctica EXCEPT that it________.

A. was conducted by air

B. made use of radio waves

C. did not measure the exact size of the lake

D. was controlled by a satellite

Câu 673 :
It can be interfered from the passage that ice would not be flat if _______.

A. there were no lake

B. the lake were not so big

C. Antarctica were not so cold

D. radio waves were not used

Câu 674 :
The word “microbes” in paragraph 3 could best be replaced by which of the following?

A. pieces of dust

B. trapped bubbles

C. tiny organism

D. rays of light

Câu 675 :
The passage mentions which of the following as a reason for the importance of Lake Vostok to scientists?

A. It can be studied using radio waves

B. It may contain uncontaminated microbes

C. It may have elevated levels of ultraviolet light

D. It has already been contaminated

Câu 676 :
The word “downside” in paragraph 3 is closest in meaning to_________

A. bottom level

B. negative aspect

C. underside

D. buried section

Câu 677 :
Read the following passage and mark the letter A, B, C, or D on your answer sheet to indicate the correct answer to each of the questions
Harvard University, today recognized as part of the top echelon of the world’s universities, came from very inauspicious and humble beginning. This oldest of American universities was founded in 1636, just sixteen years after the Pilgrims landed at Plymouth. Included in the puritan emigrants to the Massachusetts colony during this period were more than 100 graduates of England’s prestigious Oxford and Cambridge universities, and these universities graduates in the New Word were determined that their sons would have the same educational opportunities that they themselves had had, Because of this support in the colony for an institution of higher learning, the General Court of Massachusetts appropriated 400 pounds for a college in October of 1636 and early the following year decided on a parcel of land for the school; this land was in an area called Newetowne, which was later renamed Cambridge after its English cousin and is the site of t he present-day university. When a young minister named John Harvard, who came from the neighboring town of Charlestowne, died from tuberculosis in 1638, he willed half of his estate of 1,700 pounds to the fledgling college. In spite of the fact that only half of the bequest was actually paid, the General Court named the college after the minister in appreciation for what he had done. The amount of the bequest may not have been large, particularly by today’s standard, but it was more than the General Court had found it necessary to appropriate in order to open the college. Henry Dunster was appointed the first president of Harvard in 1640, and it should be noted that in addition to serving as president, he was also the entire faculty, with an entering freshmen class of four students. Although the staff did expand somewhat, for the first century of its existence the entire teaching staff consisted of the president and three or four tutors 
The main idea of this passage is that_________.

A. Harvard is one of the world’s most prestigious universities

B. What is today a great university started out small

C. John Harvard was key to the development of a great university

D. Harvard University developed under the auspices of the General Court of Massachusetts

Câu 678 :
The passage indicates that Harvard is___________.

A. one of the oldest universities in the world

B. the oldest university in the world

C. one of the oldest universities in America

D. the oldest university in America

Câu 679 :
It can be inferred from the passage that the Puritans who traveled to the Massachusetts colony were __________.

A. rather well educated

B. rather rich

C. rather supportive of the English government

D. rather undemocratic

Câu 680 :
The pronoun “they” in the second paragraph refers to_____.

A. Oxford and Cambridge universities

B. university graduates

C. sons

D. educational opportunities

Câu 681 :
The “pounds” in the second paragraph are probably_______.

A. Types of books

B. college students

C. units of money

D. school campuses

Câu 682 :
The “ English cousin” in the second paragraph refers to a_______.

A. city

B. relative

C. person

D. court

Câu 683 :
Which of the following is NOT mentioned about John Harvard?

A. What he died of

B. Where he came from

C. Where he was buried

D. How much he bequeathed to Harvard

Câu 685 :
The passage implies that_______. 

A. Henry Dunster was an ineffective president 

B. Someone else really served as president of Harvard before Henry Dunster 

C. Henry DUnster spent much of his time as president managing the Harvard faculty 

D. The position of president of Harvard was not merely an administrative position in the early ears

Câu 686 :
Read the following passage and mark the letter A, B, C, or D on your answer sheet to indicate the correct answer to each of the questions
We find that bright children are rarely held back by mixed-ability teaching. On the contrary, both their knowledge and experience are enriched. We feel that there are many disadvantages in streaming pupils. It does not take into account the fact that children develop at different rates. It can have a bad effect on both the bright and the notso-bright child. After all, it can be quite discouraging to be at the bottom of the top grade! Besides, it is rather unreal to grade people just according to their intellectual ability. This is only one aspect of their total personality. We are concerned to develop the abilities of all our pupils to the full, not just their academic ability. We also value personal qualities and social skills, and we find that mixed-ability teaching contributes to all these aspects of learning. In our classroom, we work in various ways. The pupils often work in groups; this gives them the opportunity to learn to co-operate, to share, and to develop leadership skills. They also learn how to cope with the personal problems as well as learning how to think, to make decisions, to analyze and evaluate, to communicate effectively. The pupils learn from each other as well as from the teachers. Sometimes the pupils work in pairs; sometimes the work on individual tasks and assignments, they can do this at their own speed. They also have some formal class teaching when this is appropriate. We encourage our pupils to use the library, and we teach them the skills they need in order to do this effectively. An advanced pupil can do advanced works; it does not matter what age the child is. We expect our pupils to do their best, not their least, and we give them every encouragement to attain this goal.
The author’s purpose of writing this passage is to_______.

A. recommend pair work and group work classroom activities

B. emphasize the importance of appropriate formal classroom teaching

C. offer advice on the proper use of the school library

D. argue for teaching bright and not-so-bright pupils in the same class

Câu 687 :
As it can be inferred from the passage, which of the following is an advantage of mixed-ability teaching?

A. Pupils as individuals always have the opportunities to work on their own.

B. Pupils can be hindered from an all-round development.

C. Formal class teaching is the important way to give the pupils essential skills such as those to be used in the library.

D. A pupil can be at the bottom of a class.

Câu 688 :
The phrase "held back” in paragraph 1 means________

A. forced to study in lower classes

B. prevented from advancing

C. made to lag behind

D. made to remain in the same classes

Câu 689 :
Which of the following statements can best summarize the main idea of the passage?

A. Various ways of teaching should be encouraged in class.

B. The aim of education is to find out how to teach the bright and not-sobright pupils.

C. Bright children do benefit from mixed-class teaching.

D. Children, in general, develop at different rates.

Câu 690 :
In paragraph 1, “streaming pupils"________. 

A. will help the pupils learn best 

B. is quite discouraging

C. is the act of putting pupils into classes according to their academic abilities 

D. aims at enriching both their knowledge and experience

Câu 691 :
In the passage, the author’s attitude towards “mixed-ability teaching” is_____.

A. questioning

B. objective

C. ritical

D. approving

Câu 692 :
According to the passage, which of the following is NOT true?

A. Pupils cannot develop in the best way if they are streamed into classes of different intellectual abilities.

B. There is no fixed method in teaching pupils to develop themselves to the full.

C. It’s not good for a bright child to find out that he performs worst in a mixed-ability class.

D. Development of pupils as individuals is not the aim of group work.

Câu 693 :
Read the following passage and mark the letter A , B , C , or D on your answer sheet to indicate the correct answer to each of the questions
The advent of the Internet may be one of the most important technological developments in recent years . Never before have so many people had access to so many different sources of information . For all of the Internet's advantages , however , people are currently becoming aware of some of its drawbacks and are looking for creative solutions . Among the current problems , which include a general lack of reliability and numerous security concerns , the most crucial is speed .
First of all , the Internet has grown very quickly . In 1990 , only a few academics had ever heard of the Internet . In 1996 , over 50 million people used it . Every year , the number of people with access to the Internet doubles . The rapid growth has been a problem . The computer systems which run the Internet have not been able to keep up with the demand . Also , sometimes , a request for information must pass through many routing computers before the information can be obtained . A request for information made in Paris might have to go through computers in New York , Los Angeles and Tokyo in order to obtain the required information . Consequently , the service is often slow and unpredictable. Service also tends to be worse when the Internet is busiest - during the business day of the Western Hemisphere - which is also when companies need its service the most.
Some people are trying to harness the power of networked computers in such a way as to avoid this problem . In 1995 , a group of American universities banded together to form what has come to be known as Internet II . Internet II is a smaller , more specialized system intended for academic use . Since it is more specialized , fewer users are allowed access. Consequently , the time required to receive information has decreased . Businesses are beginning to explore a possible analogue to the Internet II . Many businesses are creating their own " Intranets " . These are systems that can only be used by the members of the same company . In theory , fewer users should translate into a faster system . Intranets are very useful for large national and international companies whose branches need to share information . Another benefit of an Intranet is an increased amount of security . Since only company employees have access to the information on the Intranet , their information is protected from competitors . While there is little doubt that the Internet will eventually be a fast and reliable service , industry and the academic community have taken their own steps toward making more practical global networks . 
In the passage , which following is NOT true of the Internet ?

A. It has become increasingly less popular. 

B. It has created a sense of financial security 

C. It tends to be unreliable 

D. It is sometimes too slow to access

Câu 694 :
As it can be inferred from the passage, what benefits does Internet II have over the Internet I?

A. Small businesses pay higher premiums to access to the Internet

B. Internet II contains more information than the Internet.

C. Internet II has fewer users and therefore is faster to access

D. There is no governmental intervention regulating Internet II

Câu 695 :
The word "analogue" in paragraph 5 most nearly means_____.

A. solution

B. alternative

C. similarity

D. use

Câu 696 :
The word “harness" in paragraph 4 is closest in meaning to .........

A. block

B. steal

C. utilize

D. disguise

Câu 697 :
According to the author, what is one reason why the Internet is some times slow

A. Phone lines are often too busy with phone calls and fax transmissions to handle Internet traffic. 

B. Most people do not have computers that are fast enough to take advantage of the Internet. 

C. Often a request must travel through many computers before it reaches its final destination.

D. Scientists take up too much time on the Internet , thus slowing it down for everyone else.

Câu 698 : All of the following are advantages of business “In tranets ” mentioned in the passage EXCEPT__________

A. they move data faster. 

B. they share information with other company branches. 

C. they provide a higher level of security. 

D. they are cheaper than other alternatives

Câu 699 :
With which of the following conclusions would the author probably agree?

A. Fewer academic communities need to create their own Internet systems.

B. An Internet system with fewer users would be quicker.

C. The technology used by Internet creators is too complex for computer owners to understand.

D. Companies who develop their own Intranets are limiting their information data base.

Câu 700 :
According to the passage, which of the following statements was true in 1990?

A. The Internet was a secure means to gain information.

B. Internet data proved to be impractical.

C. The Internet experienced enormous growth rates.

D. Few people were using the Internet.

Câu 701 :
Read the following passage and mark the letter A, B, C, or D on your answer sheet to indicate the correct answer to each of the questions
Algae is a primitive form of life, a single-celled or simple multiplecelled organism that is able to conduct the process of photosynthesis. It is generally found in water but can also be found elsewhere, growing on such surfaces as rocks or trees. The various types of algae are classified according to pigment. Blue-green algae, or Cyanophyta, can grow at very high temperatures and under high-intensity light. This type of algae is the oldest form of life with photosynthetic capabilities. Fossilized remains of blue-green algae more than 3.4 billion years old have been found in parts of Africa. Green algae, or Chlorophyta, is generally found in fresh water. It reproduces on the surfaces of enclosed bodies of water such as ponds or lakes and has the appearance of a fuzzy green coating on the surface of the water. Brown algae, or Phaeophyta, grows in shallow, temperate water. This type of algae is the largest in size and is most recognizable as a type of seaweed. Its long stalks can be enmeshed on the ocean floor, or it can float freely on the ocean's surface. Red algae, or Rhodophyta, is a small, delicate organism found in the deep waters of the subtropics. This type of algae has an essential role in the formation of coral reefs: it secretes lime from the seawater to foster the formation of limestone deposits.
What is the author's main purpose?

A. To show what color algae is

B. To differentiate the various classifications of algae

C. To describe where algae is found

D. To clarify the appearance of different types of algae

Câu 702 :
Which of the following is NOT true about algae?

A. All types have one cell only.

B. It can be found out of water.

C. It can use photosynthesis.

D. It is not a relatively new form of life.

Câu 703 :
The word "pigment" at the end of the first paragraph means________.

A. size

B. shape

C. composition

D. color

Câu 704 :
Algae remnants found in Africa are________.

A. still flourishing

B. photogenic

C. extremely old

D. red in color

Câu 705 :
Green algae is generally found________. 

A. on the ocean floor 

B. on top of the water 

C. throughout ponds and lakes 

D. surrounding enclosed bodies of water

Câu 706 :
Brown algae would most likely be found________.

A. on trees

B. near green algae

C. on rocks

D. in the ocean

Câu 707 :
According to the passage, red algae is________.

A. sturdy

B. huge

C. fragile

D. found in shallow water

Câu 708 :
It can be inferred from the passage that limestone deposits serve as the basis of ...........

A. coral reefs

B. red algae

C. subtropical seawater

D. secret passages

Câu 709 :
Read the following passage and mark the letter A, B, C, or D on your answer sheet to indicate the correct answer to each of the questions
Sound moves form its source to the ear by wavelike fluctuations in air pressure, something like the crests and troughs of ocean waves. Once way to keep from hearing sound is to use ear plugs. Another way is to cancel out the sound with anti-sound. Using a noisemarker controlled by a microprocessor, engineers have produced sound waves that are half a wavelength out of phase with those of the noise to be quieted-each crest is matched to a trough, and vice versa. Once the researchers have recorded the offending sound, a microprocessor calculates the amplitude and wavelength of sound that will cancel out the crests and troughs of noise. It then produces an electric current that is amplified and fed to a loudspeaker, which produces antisound and wipes out the noise. If the anti-sound goes out of synchronization, a microphone picks up the leftover sound and sends it back to the microprocessor, which changes the phase of the anti- sound just enough to cause complete silence. The research team has concentrated on eliminating low-frequency noise from ship engines, which causes fatigue that can impair the efficiency and alertness of the crew, and may mask the warning sounds of alarm and fog signals. 
What is the main purpose of the passage?

A. To discuss a physical handicap

B. To warn about a growing danger

C. To describe the structure of the ear

D. To report on a new invention

Câu 710 :
The passage compares sound to_________. 

A. the rising and falling of water in the ocean 

B. the crests and valleys of mountain ranges 

C. a flag waving in the air 

D. a machine for gauging air pressure

Câu 711 :
The passage discusses a way to deal with an offensive noise by_________.

A. diverting people's attention from it

B. masking it with a louder noise

C. canceling it out electronically

D. removing its source

Câu 712 :
The microprocessor described in the passage will probably be used for_________.

A. composing music

B. repairing alarm systems

C. eliminating engine noises

D. intensifying for warning sounds

Câu 713 :
The researcher mentioned in the passage are concerned about unwanted noise because it can_________.

A. cause deafness

B. create hazardous working conditions

C. influence ocean waves

D. damage loudspeakers and sound equipment

Câu 714 :
According to the passage, what group of people will probably first from the use of the microprocessor?

A. Ship's crews

B. Research engineers

C. People with insomnia

D. Engine repair teams

Câu 715 :
A paragraph following the passage would most probably discuss_________.

A. the nature of fog

B. a way to improve alarm systems

C. other causes of fatigue

D. other uses for the microprocessor

Câu 716 :
Read the following passage and mark the letter A, B, C, or D on your answer sheet to indicate the correct answer to each of the questions.
The Hindu cultural celebrates marriage as a pure and pristine rite enabling two individuals start their journey of life together. It puts emphasis on the values of happiness, harmony and growth and could be traced back from the Vedic times. Months before the wedding ceremony, an engagement is held which is called "mangni". The couple is blessed here with gifts, jewelry and clothes. Another important ritual is the "mehendi" which is a paste made from the leaves of henna plant. It is the traditional art of adorning the hands and the feet of the bride with mehendi and the name of the groom is also hidden in the design. On the day of marriage, the couple exchanges garlands as a gesture of acceptance of one another and a pledge to respect one another as partners which is known as "jaimala". This is followed by "kanyadaan", where the father of the bride places her hand in the groom's hand requesting him to accept her as an equal partner. Another ritual is the "havan" in which the couple invokes Agni, the god of Fire, to witness their commitment to each other. Crushed sandalwood, herbs, sugar rice and oil are offered to the ceremonial fire. The "gath bandhan" takes place where scarves of the bride and groom are tied together symbolizing their eternal bonThis signifies their pledge before God to love each other and remain loyal. The couple then takes four "mangal pheras" or walk around the ceremonial fire, representing four goals in life: "Dharma", religious and moral duties; "Artha", prosperity; "Kama" earthly pleasures; "Moksha", spiritual salvation. The couple also takes seven steps together to begin their journey, called the "saptapardi". Then the ritual of "sindoor" takes place where the groom applies a small dot of vermilion, a red powder to the bride's forehead and welcomes her as his partner for life. This signifies the completion of the marriage. The parents of the bride and the groom then give their blessings, "ashirwad" to the newlywed couple as they touch the feet of their parents. 
What does the word magni stand for?

A. the wedding

B. the paste made from the leave of one another

C. the engagement

D. the gesture of acceptance of one another

Câu 717 :
What do the Hindu people think about marriage?

A. It is entirely a spiritual traditional ritual allowing two individuals to live together.

B. It is a belief in the growth of a family newly formed by two individuals.

C. It is a wish of happiness and harmony to come to two individuals.

D. It is just a living-together announcement of two individuals.

Câu 718 :
What can the word adorning be best replaced by?

A. decorating

B. painting

C. repairing

D. dying

Câu 719 :
What can the word invokes be best replaced by?

A. tells

B. says

C. prays

D. talks

Câu 720 :
Why does the couple exchange garlands?

A. to do a ritual

B. to show both their acceptance of and the swear to respect the partner.

C. to express their acceptance of one another and a wish of happiness.

D. to express their vow to respect each other for the whole life.

Câu 721 :
When is the wedding ceremony completed?

A. When the parents of the bride and the groom give their blessings to the couple.

B. When the couple touches their parents' feet.

C. When the couple makes seven steps together.

D. When the groom applies a small dot of vermillion of the bride's forehead.

Câu 722 :
Read the following passage and mark the letter A, B, C, or D on your answer sheet to indicate the correct answer to each of the questions
A Japanese construction company plans to create a huge independent city-state, akin to the legendary Atlantis, in the middle of the Pacific Ocean. The city, dubbed “Marinnation”, would have about one million inhabitants, two airports, and possibly even a space port. Marinnation, if built, would be a separate country but could serve as a home for international organisations such as the United Nations and the World Bank. Aside from the many political and social problems that would have to be solved, the engineering task envisaged is monumental. The initial stage requires the building of a circular dam eighteen miles in diameter attached to the sea bed in a relatively shallow place in international waters. Then, several hundred powerful pumps, operating for more than a year, would suck out the sea water from within the dam. When empty and dry, the area would have a city constructed on it. The actual land would be about 300 feet below the sea level. According to designers, the hardest task from an engineering point of view would be to ensure that the dam is leak proof and earthquake proof. If all goes well, it is hoped that Marinnation could be ready for habitation at the end of the second decade of the twenty-first century. Whether anyone would want to live in such an isolated and artificial community, however, will remain an open question until that time. 
According to the text, Marrination, when built, will be a(an)__________ city.

A. underwater

B. underground

C. marine

D. legendary

Câu 724 :
Which of the following would NOT be a problem for the construction plan?

A. social

B. political

C. engineering

D. financial

Câu 725 :
Which of the following is NOT true according to the text?

A. Marinnation would have about one million inhabitants.

B. Marinnation will be built in a deep place in the Pacific Ocean.

C. Marinnation will be located in the middle of the Pacific Ocean.

D. Marinnation could be served as home for international organisations.

Câu 726 :
The word ‘monumental’ in Paragraph 2 means __________.

A. important and difficult

B. important and difficult

C. important and historic

D. serving as a monument

Câu 729 :
What does the author imply in the last sentence of the passage?

A. People will ask an open question about living in Marinnation.

B. The construction of Marinnation could never be completed in time.

C. People might not want to live in an isolated and artificial community.

D. People will ask how they can live in such an isolated and artificial city.

Câu 730 :
Read the following passage and mark the letter A, B, C, or D on your answer sheet to indicate the correct answer to each of the questions
Hummingbirds are small, often brightly colored birds of the family Trochilidae that live exclusively in the Americas. About 12 species are found in North America, but only the ruby-throated hummingbird breeds in eastern North America and is found from Nova Scotia to Florida. The greatest variety and number of species are found in South America. Another hummingbird species is found from southeastern Alaska to northern California. Many hummingbirds are minute. But even the giant hummingbird found in western South America, which is the largest known hummingbird, is only about 8 inches long and weighs about two-thirds of an ounce. The smallest species, the bee hummingbird of Cuba and the Isle of Pines, measures slightly more than 5.5 centimeters and weighs about two grams. Hummingbirds' bodies are compact, with strong muscles. They have wings shaped like blades. Unlike the wings of other birds, hummingbird wings connect to the body only at the shoulder joint, which allows them to fly not only forward but also straight up and down, sideways, and backward. Because of their unusual wings hummingbirds can also hover in front of flowers so they can suck nectar and find insects. The humming- bird's bill, adapted for securing nectar from certain types of flowers, is usually rather long and always slender, and it is curved slightly downward in many species. The hummingbird’s body feathers are sparse and more like scales than feathers. The unique character of the feathers produces brilliant and iridescent colors, resulting from the refraction of light by the feathers. Pigmentation of other feathers also contributes to the unique color and look. Male and female hummingbirds look alike in some species but different in most species; males of most species are extremely colorful. The rate at which a hummingbird beats its wings does not vary, regardless of whether it is flying forward, flying in another direction, or merely hovering. But the rate does vary with the size of the bird - the larger the bird, the lower the rate, ranging from 80 beats per second for the smallest species to 10 times per second for larger species. Researchers have not yet been able to record the speed of the wings of the bee humming-bird but imagine that they beat even faster. Most hummingbirds, especially the smaller species, emit scratchy, twittering, or squeaky sounds. The wings, and sometimes the tail feathers, often produce humming, hissing, or popping sounds, which apparently function much as do the songs of other birds. 
According to the passage, where are hummingbirds found?

A. Throughout the world

B. In South America only

C. In North America only

D. In North and South America

Câu 731 :
The author indicates that the ruby-throated hummingbird is found _______.

A. throughout North America.

B. in California.

C. in South America.

D. in the eastern part of North.

Câu 732 :
The word “minute” in the second paragraph is closest in meaning to_______.

A. extremely tiny

B. extremely fast

C. unique

D. organized

Câu 733 :
The word which in the second paragraph refers to _______.

A. western South America.

B. the giant hummingbird.

C. all hummingbirds.

D. Florida hummingbirds.

Câu 734 :
What does the author imply about the rate hummingbirds’ wings beat?

A. Although the bee hummingbird is the smallest, its wings don't beat the fastest. 

B. The hummingbird's wings beat faster when it is sucking nectar than when it is just flying. 

C. The rate is not much different than that of other birds of its size. 

D. The speed at which a bee hummingbird’s wings beat is not actually known.

Câu 735 :
The author indicates that a hummingbird’s wings are different from those of other birds because .........

A. they attach to the body at one point only. 

B. they attach to the body at more points than other birds. 

C. they attach and detach from the body. 

D. they are controlled by a different section of the brain.

Câu 736 :
According to the passage, what causes the unique color and look of hummingbirds?

A. The color of the feathers

B. The structure of the feathers as well as pigmentation 

C. The rapidity of flight 

D. The pigmentation of the body

Câu 737 :
Read the following passage and mark the letter A, B, C, or D on your answer sheet to indicate the correct answer to each of the questions
If we believe that clothing has to do with covering the body, and costumes with the choice of a particular form of garment for a particular use, then we can say that clothing depends primarily on such physical conditions as climate, health, and textile manufacture, whereas costumes reflect social factors such as religious beliefs, aesthetics, personal status, and the wish to be distinguished from or to emulate our fellows. The ancient Greeks and the Chinese believed that we first covered our bodies for some physical reason such as protecting ourselves from the weather elements. Ethnologists and psychologists have invoked psychological reasons: modesty in the case of ancients, and taboo, magical influence and the desire to please for the moderns. In early history, costumes must have fulfilled a function beyond that of simple utility, perhaps through some magical significance, investing primitive man with the attributes of other creatures. Ornaments identified the wearer with animals, gods, heroes or other men. This identification remains symbolic in more sophisticated societies. We should bear in mind that the theater has its distant origins in sacred performances, and in all periods children at play have worn disguises, so as to adapt gradually to adult life. Costumes helped inspire fear or impose authority. For a chieftain, costumes embodied attributes expressing his power, while a warrior's costume enhanced his physical superiority and suggested he was superhuman. In more recent times, professional or administrative costume has been devised to distinguish the wearer and express personal or delegated authority; this purpose is seen clearly in the judge's robes and the police officer’s uniform. Costume denotes power, and since power is usually equated with wealth, costume came to be an expression of social caste and material prosperity. Military uniform denotes rank and is intended to intimidate to protect the body and to express membership in a group. At the bottom of the scale, there are such compulsory costumes as the convict’s uniform. Finally, costume can possess a religious significance that combines various elements: an actual or symbolic identification with a god, the desire to express this in earthly life, and the desire to enhance the wearer's position of respect.
The passage mainly discusses costume in terms of its _______

A. physical protection

B. religious significance

C. social function

D. beauty and attractiveness

Câu 738 :
What is the purpose of the paragraph 1?

A. To describe the uses of costume 

B. To contrast costume with the clothing 

C. To trade the origins of costume 

D. To point out that clothing developed before costume

Câu 739 :
Psychological reasons for wearing garments include _______.

A. protection from cold

B. availability of materials

C. prevention of illness

D. wishing to give pleasure

Câu 740 :
The word “Ornaments” in paragraph 3 is closet in meaning to _______.

A. garments

B. representation

C. details

D. decorations

Câu 741 :
It can be inferred from paragraph 3 that _______.

A. The function of costume has become very sophisticated.

B. Children like to identify with other creature by wearing costumes.

C. Primitive people wore cloths only for sacred performances.

D. Costume no longer fulfills a function beyond simple utility.

Câu 742 :
Why does the author mention the police officer's uniform?

A. To illustrate the aesthetic function of costume

B. To identify the wearer with a hero

C. To suggest that police are superhuman

D. To show how costume signifies authority

Câu 743 :
The word “denotes” in line paragraph 4 is closest in meaning to_______.

A. disguises

B. describes

C. indicates

D. denigrates

Câu 744 :
Which of the following would most likely NOT be reflected in a person's costume, as it is defined in the passage?

A. Having a heart condition

B. Playing in a baseball game

C. Working in a hospital

D. Participating in a religious ceremony

Câu 745 :
Read the following passage and mark the letter A, B, C, or D on your answer sheet to indicate the correct answer to each of the questions
Most forms of property are concrete and tangible , such as houses , cars , furniture or anything else that is included in one's possessions . Other forms of property can be intangible , and copyright deals with intangible forms of property . Copyright is a legal protection extended to authors of creative works , for example books magazine articles , maps , films , plays , television shows , software , paintings , photographs , music , choreography in dance and all other forms of intellectual or artistic property .
 Although the purpose of artistic property is usually public use and enjoyment, copyright establishes the ownership of the creator. When a person buys a copyrighted magazine, it belongs to this individual as a tangible object. However, the authors of the magazine articles own the research and the writing that went into creating the articles. The right to make and sell or give away copies of books or articles belongs to the authors, publishers, or other individuals or organizations that hold the copyright. To copy an entire book or a part of it, permission must be received from the copyright owner, who will most likely expect to be paid. Copyright law distinguishes between different types of intellectual property. Music may be played by anyone after it is published. However, if it is performed for profit, the performers need to pay a fee called a royalty. A similar principle applies to performances of songs and plays. On the other hand, names, ideas and book titles are excepted. Ideas do not become copyrighted property until they are published in a book a painting or a musical work. Almost all artistic work created before the 20th century is not copyrighted because it was created before the copyright law was passed. The two common ways of infringing upon the copyright are plagiarism and piracy. Plagiarizing the work of another person means passing it off as one’s own. The word plagiarism is derived from the Latin plaglarus, which means “abductor”. Piracy may be an act of one person, but, in many cases, it is a joint effort of several people who reproduce copyrighted material and sell it for profit without paying royalties to the creator. Technological innovations have made piracy easy and anyone can duplicate a motion picture on videotape a computer program, or a book. Video cassette recorders can be used by practically anyone to copy movies and television programs, and copying software has become almost as easy as copying a book. Large companies zealously monitor their copyrights for slogans, advertisements, and brand names, protected by a trademark 
What does the passage mainly discuss?

A. Legal rights of property owners 

B. Legal ownership of creative work 

C. Examples of copyright piracy 

D. Copying creating work for profit

Câu 746 :
The word “principle” in paragraph 3 is closest in meaning to ........

A. crucial point

B. cardinal role

C. fundamental rule

D. formidable force

Câu 747 :
Which of the following properties is NOT mentioned as protected by copyright?

A. music and plays 

B. paintings and maps 

C. printed medium 

D. scientific discoveries

Câu 748 :
t can be inferred from the passage that it is legal if __________

A. two songs, written by two different composers, have the same melody 

B. two books, written by two different authors, have the same titles 

C. two drawings, created by two different artists, have the same images 

D. two plays, created by two different playwrights, have the same plot and characters

Câu 749 :
With which of the following statements is the author most likely to agree?

A. Teachers are not allowed to make copies of published materials for use by their students 

B. Plays written in the 16th century cannot be performed in theaters without permission 

C. Singers can publicly sing only the songs for which they wrote the music and the lyrics 

D. It is illegal to make photographs when sightseeing or traveling

Câu 750 :
The phrase “infringing upon” in paragraph 4 is closest in meaning to _____

A. impinging upon

B. inducting for

C. violating

D. abhorring

Câu 751 :
Read the following passage and mark the letter A, B, C, or D on your answer sheet to indicate the correct answer to each of the questions
Most forms of property are concrete and tangible , such as houses , cars , furniture or anything else that is included in one's possessions . Other forms of property can be intangible , and copyright deals with intangible forms of property . Copyright is a legal protection extended to authors of creative works , for example books magazine articles , maps , films , plays , television shows , software , paintings , photographs , music , choreography in dance and all other forms of intellectual or artistic property .
 Although the purpose of artistic property is usually public use and enjoyment, copyright establishes the ownership of the creator. When a person buys a copyrighted magazine, it belongs to this individual as a tangible object. However, the authors of the magazine articles own the research and the writing that went into creating the articles. The right to make and sell or give away copies of books or articles belongs to the authors, publishers, or other individuals or organizations that hold the copyright. To copy an entire book or a part of it, permission must be received from the copyright owner, who will most likely expect to be paid. Copyright law distinguishes between different types of intellectual property. Music may be played by anyone after it is published. However, if it is performed for profit, the performers need to pay a fee called a royalty. A similar principle applies to performances of songs and plays. On the other hand, names, ideas and book titles are excepted. Ideas do not become copyrighted property until they are published in a book a painting or a musical work. Almost all artistic work created before the 20th century is not copyrighted because it was created before the copyright law was passed. The two common ways of infringing upon the copyright are plagiarism and piracy. Plagiarizing the work of another person means passing it off as one’s own. The word plagiarism is derived from the Latin plaglarus, which means “abductor”. Piracy may be an act of one person, but, in many cases, it is a joint effort of several people who reproduce copyrighted material and sell it for profit without paying royalties to the creator. Technological innovations have made piracy easy and anyone can duplicate a motion picture on videotape a computer program, or a book. Video cassette recorders can be used by practically anyone to copy movies and television programs, and copying software has become almost as easy as copying a book. Large companies zealously monitor their copyrights for slogans, advertisements, and brand names, protected by a trademark 
 The purpose of copyright law is most comparable with the purpose of which of the following?

A. A law against theft 

B. A law against smoking 

C. A school policy 

D. A household rule

Câu 752 :
According to the passage, copyright law is _______

A. meticulously observed 

B. routinely ignored 

C. frequently debated 

D. zealously enforced

Câu 753 :
Read the following passage and mark the letter A, B, C, or D on your answer sheet to indicate the correct answer to each of the questions
Cooperation is the common endeavor of two or more people to perform a task or reach a jointly cherished goal. Like competition and conflict, there are different forms of cooperation, based on group organization and attitudes. In the first form, known as primary cooperation, group and individual fuse. The group contains nearly all of each individual’s life. The rewards of the group’s work are shared with each member. There is an interlocking identity of individual, group and task performed. Means and goals become one, for cooperation itself is valued. While primary cooperation is most often characteristic of preliterate societies, secondary cooperation is characteristic of many modem societies. In secondary cooperation, individuals devote only part of their lives to the group. Cooperation itself is not a value. Most members of the group feel loyalty, but the welfare of the group is not the first consideration. Members perform tasks so that they can separately enjoy the fruits of their cooperation in the form of salary prestige, or power. Business offices and professional athletic teams are examples of secondary cooperation. In the third type called tertiary cooperation or accommodation, latent conflict underlies the shared work. The attitudes of the cooperating parties are purely opportunistic: the organization is loose and fragile. Accommodation involves common means to achieve antagonistic goals: it breaks down when the common means cease to aid each party in reaching its goals. This is not, strictly speaking cooperation at all, and hence the somewhat contradictory term antagonistic cooperation is sometimes used for this relationship. 
What is the author’s main purpose in the first paragraph of to passage?

A. To urge readers to cooperate more often

B. To offer a brief definition of cooperation

C. To explain how cooperation differs from competition and conflict

D. To show the importance of group organization and attitudes

Câu 754 :
The word cherished in paragraph 1 is closest in meaning to_______________

A. defined

B. agreed on

C. prized

D. set up

Câu 755 :
Which of the following statements about primary cooperation is supported by information in the passage?

A. It is usually the first stage of cooperation achieved by a group of individuals attempting to cooperate

B. It is most commonly seen among people who have not yet developed reading and writing skills

C. It is an ideal that can never be achieved

D. It was confined to prehistoric times

Câu 756 :
According to the passage, why do people join groups that practice secondary cooperation?

A. To experience the satisfaction of cooperation

B. To associate with people who have similar backgrounds

C. To get rewards for themselves

D. To defeat a common enemy

Câu 757 :
Which of the following is an example of the third form of cooperation as it is defined in the fourth paragraph?

A. Students form a study group so that all of them can improve their grades

B. Members of a farming community share work and the food that they grow

C. Two rival political parties temporarily work together to defeat a third party

D. A new business attempts to take customers away from an established company

Câu 758 :
Which of the following is NOT given as a name for the third type of cooperation?

A. Tertiary cooperation

B. Antagonistic cooperation

C. Accommodation

D. Latent conflict

Câu 759 :
The word “fragile” in paragraph 4 is closest in meaning to .........

A. involuntary

B. poorly planned

C. inefficient

D. easily broken

Câu 760 :
Read the following passage and mark the letter A , B , C , or D on your answer sheet to indicate the correct answer to each of the questions
Most people can remember a phone number for up to thirty seconds . When this short amount of time elapses , however , the numbers are erased from the memory . How did the information get there in the first place ? Information that makes its way to the short term memory ( STM ) does so via the sensory storage area . The brain has a filter which only allows stimuli that is of immediate interest to pass on to the STM , also known as the working memory . There is much debate about the capacity and duration of the short term memoiy . The most accepted theory comes from George A. Miller , a cognitive psychologist who suggested that humans can remember approximately seven chunks of information . A chunk is defined as a meaningful unit of information such as a word or name rather than just a letter or number . Modern theorists suggest that one can increase the capacity of the short term memory by chunking , or classifying similar information together . By organizing information , one can optimize the STM , and improve the chances of a memory being passed on to long term storage When making a conscious effort to memorize something , such as information for an exam many people engage in " rote rehearsal " . By repeating something over and over again , one is able to keep a memory alive . Unfortunately , this type of memory maintenance only succeeds if there are no interruptions . As soon as a person stops rehearsing the information , it has the tendency to disappear . When a pen and paper are not handy people often attempt to remember a phone number by repeating it aloud . If the doorbell rings or the dog barks to come in before a person has the opportunity to make a phone call , he will likely forget the number instantly . Therefore rote rehearsal is not an efficient way to pass information from the short term to long term memory . A better way is to practice " elaborate rehearsal " . This involves assigning semantic meaning to a piece of information so that it can be filed along with other pre - existing long term memories .
Encoding information semantically also makes it more retrievable . Retrieving information can be done by recognition or recall . Humans can easily recall memories that are stored in the long term memory and used often- however , if a memory seems to be forgotten , it may eventually be retrieved by prompting . The more cues a person is given ( such as pictures ) , the more likely a memory can be reùieved . This is why multiple choice tests are often used for subjects that require a lot of memorization . 
According to the passage , how do memories get transferred to the STM ?

A. They revert from the long term memory . 

B. They are filtered from the sensory storage area . 

C. They get chunked when they enter the brain . 

D. They enter via the nervous system .

Câu 761 :
All of the following are mentioned as places in which memories are stored EXCEPT the:

A. STM

B. long term memory

C. sensory storage area

D. maintenance area

Câu 762 :
Why does the author mention a dog’s bark?

A. To give an example of a type of memory

B. To provide a type of interruption

C. To prove that dogs have better memories than humans

D. To compare another sound that is loud like a doorbell

Câu 763 :
The word “elaborate” in paragraph 3 is closest in meaning to ...........

A. complex

B. efficient

C. pretty

D. regular

Câu 764 :
Which of the following is NOT supported by the passage?

A. The working memory is the same as the short term memory.

B. A memory is kept alive through constant repetition,

C. Cues help people to recognize information.

D. Multiple choice exams are the most difficult.

Câu 765 :
How do theorists believe a person can remember more information in a short time?

A. By organizing it 

B. By repeating it 

C. By giving it a name 

D. By drawing it

Câu 766 :
The author believes that rote rotation is ........

A. the best way to remember something

B. more efficient than chunking

C. ineffective in the long run

D. an unnecessary interruption

Câu 767 :
The word “cues” in the passage is closest in meaning to ........

A. Questions

B. Clues

C. Images

D. Tests

Câu 768 :
Read the following passage and mark the letter A, B, C, or D on your answer sheet to indicate the correct answer to each of the questions
The lack of printing regulations and the unenforceability of British copyright law in the American colonies made it possible for colonial printers occasionally to act as publishers. Although they rarely undertook major sj publishing project because it was difficult to sell books as cheaply as they could be imported from Europe printers 9 in Philadelphia did publish work that required only small amounts of capital, paper, and type. Broadsides could be published with minimal financial risk. Consisting of only one sheet of paper and requiring small amounts of type broadsides involved lower investments of capital than longer works. Furthermore, the broadside format lent itself to subjects of high, if temporary, interest, enabling them to meet with ready sale. If the broadside printer miscalculated, however, and produced a sheet that did not sell, it was not likely to be a major loss and the printer would know this immediately, There would be no agonizing wait with large amounts of capital tied up books gathering dust on the shelves, and creditors impatient for payment. In addition to broadsides, books and pamphlets, consisting mainly of political tracts, catechisms, primers, and chapbooks were relatively inexpensive to print and to buy. Chapbook were pamphlet-sized books, usually containing popular tales, ballads, poems, short plays, and jokes, small, both in formal and number of pages, they were generally bound simply, in boards (a form of cardboard) or merely stitched in paper wrappers (a sewn antecedent of modernday paperbacks). Pamphlets and chapbooks did not require fine paper or a great deal of type to produce they could thus be printed in large, costeffective editions and sold cheaply. By far, the most appealing publishing investments were to be found in small books that had proven to be steady sellers, providing a reasonably reliable source of income for the publisher. They would not, by nature, be highly topical or political, as such publications would prove of fleeting interest. Almanacs, annual publications that contained information on astronomy and weather patterns arranged according to the days, week, and months of a given year, provided the perfect steady seller because their information pertained to the locale in which they would be used. 
Which aspect of colonial printing does the passage mainly discuss?

A. Laws governing the printing industry.

B. Competition among printers

C. Types of publications produced

D. Advances in printing technology

Câu 769 :
According to the passage, why did colonial printers avoid major publishing projects?

A. Few colonial printers owned printing machinery that was large enough to handle major projects.

B. There was inadequate shipping available in the colonies.

C. Colonial printers could not sell their work for a competitive price.

D. Colonial printers did not have the skills necessary to undertake large publishing projects.

Câu 770 :
Broadsides could bs published with little risk to colonial printers because they .........

A. required a small financial investment and sold quickly

B. were in great demand in European markets

C. were more popular with colonists than chapbooks and pamphlets

D. generally dealt with topics of long-term interest to many colonists

Câu 771 :
The word "they" refers to ........ 

A. chapbooks 

B. tales 

C. jokes

D. pages

Câu 772 :
The word "appealing" is closest in meaning to ........

A. dependable

B. respectable

C. enduring

D. attractive

Câu 773 :
What were "steady sellers"? 

A. Printers whose incomes were quite large 

B. People who traveled from town to town selling Books and pamphlets 

C. Investors who provided reliable financial Support for new printers 

D. Publications whose sales were usually consistent from year to year

Câu 774 :
All of the following are defined in the passage EXCEPT ............

A. "Broadsides"

B. "catechisms"

C. "chapbooks"

D. "Almanacs"

Câu 775 :
Read the following passage and mark the letter A, B, C, or D on your answer sheet to indicate the correct answer to each of the questions
Dodder is an unusual and unwanted plant that attacks other plants. Except for its flowers, the plant looks like spaghetti noodles. Its almost leafless, thread–like stems hang down atop other plants that dodder needs to stay alive. Dodder does not produce its own food. Instead, it steals it from other plants. It feeds by sucking juices from the plant it is wrapped around, often making its host very weak or even killing it. Dodder can find other plants by their smell. When a dodder seedling starts growing, it follows the scent of plants it prefers, like tomato plants, potato plants, or other farm crops. Unlike most plants that usually grow in the direction of light or warmth, a dodder plant will grow in the direction of, for example, tomato odor––if a tomato happens to be growing nearby. However, a young dodder plant must find a host plant quickly. If it cannot catch a whiff of a potential host within a few days, it will dry up and disappear — even if there is plenty of water around. Once it finds a host, the young dodder plant will attach itself to it and start growing faster. At that point, the dodder plant will drop its root. Dodder is thus a difficult weed to manage and a real headache for farmers. When it does get out of hand, dodder can greatly reduce a farmer’s harvest or even destroy crops completely. Before sowing their produce, farmers in warm parts of the world often check to make sure no unwanted dodder seeds have intermingled with their crop seeds. This is a good way to stop dodder plants from sneaking their way into a crop field.
What is the main topic of the passage?

A. A new variety of farm crop

B. Plants that are harmful to humans

C. Recent improvements in farming methods

D. The special abilities of a dangerous plant

Câu 776 :
Why does the author mention spaghetti?

A. To analyze the content of some food

B. To describe the shape of a dodder plant

C. To explain where the dodder plant came from

D. To argue that dodder plants can be used to make food

Câu 777 :
The word it refers to _________. 

A. host 

B. food 

C. plant 

D. dodder

Câu 778 :
What causes dodder to grow in a particular direction?

A. Light

B. Water

C. Odor

D. Temperature

Câu 779 :
The expression catch a whiff is closest in meaning to .........

A. Eat the seeds

B. find the location

C. notice the smell

D. determine the size

Câu 780 :
What will happen if a dodder plant starts growing where there are no other plants around?

A. It will soon die

B. It will grow deeper roots.

C. It will attract other plants

D. It will cover the entire area of soil

Câu 781 :
The expression get out of hand is closest in meaning to .........

A. Dry up and die

B. change its color

C. become hard to find

D. grow uncontrollably

Câu 782 :
Read the following passage and mark the letter A, B, C, or D on your answer sheet to indicate the correct answer to each of the questions
Mickey Mantle was one of the greatest baseball players of all time. He played for the New York Yankees in their years of glory. From the time Mantle began to play professionally in 1951 to his last year in 1968, baseball was the most popular game in the United States. For many people, Mantle symbolized the hope, prosperity, and confidence of America at that time. Mantle was a fast and powerful player, a “switch-hitter” who could bat both right-handed and lefthanded. He won game after game, one World Series championship after another, for his team. He was a wonderful athlete, but this alone cannot explain America’s fascination with him. Perhaps it was because he was a handsome, red-haired country boy, the son of a poor miner from Oklahoma. His career, from the lead mines of the West to the heights of success and fame, was a fairy-tale version of the American dream. Or perhaps it was because America always loves a “natural”: a person who wins without seeming to try, whose talent appears to come from an inner grace. That was Mickey Mantle. But like many celebrities, Mickey Mantle had a private life that was full of problems. He played without complaint despite constant pain from injuries. He lived to fulfill his father’s dreams and drank to forget his father’s early death. It was a terrible addiction that finally destroyed his body. It gave him cirrhosis of the liver and accelerated the advance of liver cancer. Even when Mickey Mantle had turned away from his old life and warned young people not to follow his example, the destructive process could not be stopped. Despite a liver transplant operation that had all those who loved and admired him hoping for a recovery, Mickey Mantle died of cancer at the age of 63. 
What is the main idea of the passage?

A. Mickey Mantle as the greatest baseball player of all time.

B. Mickey Mantle’s success and private life full of problems.

C. Mickey Mantle and his career as a baseball player.

D. Mickey Mantle and the history of baseball.

Câu 783 :
According to the passage, Mantle could ______.

A. hit the ball to score from a long distance.

B. bat better with his left hand than with his right hand.

C. hit with the bat on either side of his body.

D. give the most powerful hit in his team.

Câu 784 :
The word “this” as used in paragraph 2 refers to ___________.

A. Mantle’s being a fast and powerful player

B. Mantle’s being a wonderful athlete

C. Mantle’s being fascinated by many people

D. Mantle’s being a “switch-hitter”

Câu 785 :
The author uses the word “But” in paragraph 4 to ______.

A. change the topic of the passage

B. give an argument in favor of Mantle’s success and fame

C. explain how Mantle got into trouble

D. give an example of the trouble in Mantle’s private life

Câu 786 :
The word “accelerated” in paragraph 5 is closest in meaning to ______.

A. worsened

B. bettered

C. delayed

D. quickened

Câu 787 :
Which of the following is mentioned as the main cause of the destruction of Mantle’s body?

A. His loneliness

B. His way of life

C. His liver transplant operation

D. His own dream

Câu 788 :
What causes dodder to grow in a particular direction?

A. Light

B. Water

C. Odor

D. Temperature

Câu 789 :
Read the following passage and mark the letter A, B, C, or D on your answer sheet to indicate the correct answer to each of the questions
Humans have struggled against weeds since the beginnings of agriculture. Marring our gardens is one of the milder effects of weeds - any plants that thrive where they are unwanted. They clog waterways, destroy wildlife habitats, and impede farming. Their spread eliminates grazing areas and accounts for one-third of all crop loss. They compete for sunlight, nutrients, and water with useful plants. The global need for weed control had been answered mainly by the chemical industry. Its herbicides are effective and sometimes necessary, but some pose serious problems, particularly if misused. Toxic compounds threaten animal and public health when they accumulate in food plants, groundwater, and drinking water. They also harm workers who apply them. In recent years, the chemical industry has introduced several herbicides that are more ecologically sound. Yet new chemicals alone cannot solve the world's weed problems. Hence, scientists are exploring the innate weed-killing powers of living organisms, primarily insects and microorganisms. The biological agents now in use are environmentally benign and are harmless to humans. They can be chosen for their ability to attack selected targets and leave crops and other plants untouched. In contrast, some of the most effective chemicals kill virtually all the plants they come in contact with, sparing only those that are naturally resistant or have been genetically modified for resistance. Furthermore, a number of biological agents can be administered only once, after which no added applications are needed. Chemicals typically must be used several times per growing season. 
With what topic does this passage primarily deal?

A. The importance of the chemical industry.

B. The dangers of toxic chemicals.

C. A proposal to ban the use of all herbicides

D. Advantages of biological agents over chemical ones.

Câu 790 :
The word "Marring" in paragraph 1 is closest in meaning to_______________.

A. spoiling

B. planting

C. dividing

D. replacing

Câu 791 :
Which of the following terms does the author define in the first paragraph?

A. weeds

B. nutrients

C. wildlife habitats

D. grazing area

Câu 792 :
Which of the following statements about the use of chemical agents as herbicides would the author most likely agree?

A. It is safe but inefficient.

B. It is occasionally required.

C. It should be increased.

D. It has become more dangerous recently.

Câu 793 :
Which of the following is NOT given as an advantage of using biological agents over chemical herbicides?

A. They are safer for workers.

B. They are less likely to destroy desirable plants.

C. They are more easily available.

D. They do not have to be used as often.

Câu 794 :
The word "innate" in paragraph 3 is closest in meaning to __________.

A. effective

B. organic

C. natural

D. active

Câu 795 :
According to the passage, biological agents mainly consist of ____________.

A. herbicides

B. useful plants

C. weeds

D. insects and microorganisms

Câu 796 :
According to the passage, biological agents mainly consist of ____________.

A. herbicides

B. useful plants

C. weeds

D. insects and microorganisms

Câu 797 :
Read the following passage and mark the letter A, B, C, or D on your answer sheet to indicate the correct answer to each of the questions
Quite different from storm surges are the giant sea waves called tsunamis, which derive their name from the Japanese expression for “high water in a harbor.” These waves are also referred to by the general public as tidal waves, although they have relatively little to do with tides. Scientists often referred to them as seismic sea waves, far more appropriate in that they do result from undersea seismic activity. Tsunamis are caused when the sea bottom suddenly moves, during an underwater earthquake or volcano for example, and the water above the moving earth is suddenly displaced. This sudden shift of water sets off a series of waves. These waves can travel great distances at speeds close to 700 kilometers per hour. In the open ocean, tsunamis have little noticeable amplitude, often no more than one or two meters. It is when they hit the shallow waters near the coast that they increase in height, possibly up to 40 meters. Tsunamis often occur in the Pacific because the Pacific is an area of heavy seismic activity. Two areas of the Pacific well accustomed to the threat of tsunamis are Japan and Hawaii. Because the seismic activity that causes tsunamis in Japan often occurs on the ocean bottom quite close to the islands, the tsunamis that hit Japan often come with little warning and can, therefore, prove disastrous. Most of the tsunamis that hit the Hawaiian Islands, however, originate thousands of miles away near the coast of Alaska, so these tsunamis have a much greater distance to travel and the inhabitants of Hawaii generally have time for warning of their imminent arrival. Tsunamis are certainly not limited to Japan and Hawaii. In 1755, Europe experienced a calamitous tsunami, when movement along the fault lines near the Azores caused a massive tsunami to sweep onto the Portuguese coast and flood the heavily populated area around Lisbon. The greatest tsunami on record occurred on the other side of the world in 1883 when the Krakatoa volcano underwent a massive explosion, sending waves more than 30 meters high onto nearby Indonesian islands; the tsunami from this volcano actually traveled around the world and was witnessed as far away as the English Channel. 
The paragraph preceding this passage most probably discusses _________ .

A. tides

B. storm surges

C. tidal waves

D. underwater earthquakes

Câu 798 :
According to the passage, all of the following are true about tidal waves EXCEPT that _____ .

A. they are caused by sudden changes in high and low tides

B. this terminology is not used by the scientific community

C. they are the same as tsunamis

D. they refer to the same phenomenon as seismic sea waves

Câu 799 :
The word “displaced” in line 6 is closest in meaning to _________ .

A. not pleased

B. located

C. moved

D. filtered

Câu 800 :
It can be inferred from the passage that tsunamis ________ .

A. are often identified by ships on the ocean

B. generally reach heights greater than 40 meters

C. are far more dangerous on the coast than in the open ocean

D. cause severe damage in the middle of the ocean

Câu 801 :
Quite different from storm surges are the giant sea waves called tsunamis, which derive their name from the Japanese expression for “high water in a harbor.” These waves are also referred to by the general public as tidal waves, although they have relatively little to do with tides. Scientists often referred to them as seismic sea waves, far more appropriate in that they do result from undersea seismic activity. Tsunamis are caused when the sea bottom suddenly moves, during an underwater earthquake or volcano for example, and the water above the moving earth is suddenly displaced. This sudden shift of water sets off a series of waves. These waves can travel great distances at speeds close to 700 kilometers per hour. In the open ocean, tsunamis have little noticeable amplitude, often no more than one or two meters. It is when they hit the shallow waters near the coast that they increase in height, possibly up to 40 meters. Tsunamis often occur in the Pacific because the Pacific is an area of heavy seismic activity. Two areas of the Pacific well accustomed to the threat of tsunamis are Japan and Hawaii. Because the seismic activity that causes tsunamis in Japan often occurs on the ocean bottom quite close to the islands, the tsunamis that hit Japan often come with little warning and can, therefore, prove disastrous. Most of the tsunamis that hit the Hawaiian Islands, however, originate thousands of miles away near the coast of Alaska, so these tsunamis have a much greater distance to travel and the inhabitants of Hawaii generally have time for warning of their imminent arrival. Tsunamis are certainly not limited to Japan and Hawaii. In 1755, Europe experienced a calamitous tsunami, when movement along the fault lines near the Azores caused a massive tsunami to sweep onto the Portuguese coast and flood the heavily populated area around Lisbon. The greatest tsunami on record occurred on the other side of the world in 1883 when the Krakatoa volcano underwent a massive explosion, sending waves more than 30 meters high onto nearby Indonesian islands; the tsunami from this volcano actually traveled around the world and was witnessed as far away as the English Channel.
A main difference between tsunamis in Japan and in Hawaii is that tsunamis in Japan are more likely to _________ .

A. come from greater distances

B. originate in Alaska

C. arrive without warning

D. be less of a problem

Câu 802 :
A “calamitous” tsunami, in line 17, is one that is _________

A. disastrous

B. expected

C. extremely calm

D. at fault

Câu 803 :
From the expression “on record” in line 19, it can be inferred that the tsunami that accompanied the Krakatoa volcano ___________ .

A. was filmed as it was happening

B. occurred before efficient records were kept

C. was not as strong as the tsunami in Lisbon

D. might not be the greatest tsunami ever

Câu 804 :
The passage suggests that the tsunami resulting from the Krakatoa volcano .....

A. was unobserved outside of the Indonesian islands

B. resulted in little damage

C. was far more destructive close to the source than far away

D. caused volcanic explosions in the English Channel

Câu 805 :
Read the following passage and mark the letter A, B, C, or D on your answer sheet to indicate the correct answer to each of the questions
A lot of advice is available for college leavers heading for their first job . In this article we consider the move to a second job . We are not concerned with those looking for a second temporary position while hunting for a permanent job . Nor are we concerned with those leaving an unsatisfactory job within the first few weeks . Instead , we will be dealing with those of you taking a real step on the career ladder , choosing a job to fit in with your ambitions now that you have learnt your way around , acquired some skills and have some idea of where you want to go . What sort of job should you look for ? Much depends on your long - term aim . You need to ask yourself whether you want to specialize in a particular field , work your way up to higher levels of responsibility or out of your current employment into a broader field .
Whatever you decide, you should choose your second job very carefully. You should be aiming to stay in it for two or three years. This job will be studied very carefully when you send your letter of application for your next job. It should show evidence of serious career planning. Most important, it should extend you, develop you and give you increasing responsibility. Incidentally, if you are interested in traveling, now is the time to pack up and go. You can do temporary work for a while when you return, pick up where you left off and get the second job then. Future potential employers will be relieved to see that you have got it out of your system, and are not likely to go off again. Juliette Davidson spend her first year after leaving St. Aldate’s College working for three lawyers. It was the perfect first job in that “ OK ... they were very supportive people. I was gently introduced to the work, learnt my way around an office and improve my word processing skills. However, there was no scope for advancement. One day, I gave my notice, bought an air ticket and traveled for a year. Juliette now works as a Personal Assistant to Brenda Cleverdon, the Chief Executive of business in the Community. “In two and a half years I have become more able and my job has really grown, “ she says. “ Right from the beginning my boss was very keen to develop me. My job title is the same as it was when I started but the duties have changed. From mainly typing and telephone work, I have progressed to doing most of the correspondence and budgets. I also have to deal with a variety of queries, coming from chairmen of large companies to people wanting to know how to start their own business. Brenda involves me in all her work but also gives me specific projects to do and events to organize.” 
Who is intended to benefit from the advice given in the article?

A. students who have just finished their studies 

B. people who are unhappy with their current job 

C. those who are interested in establishing a career

D. people who change jobs regularly

Câu 806 :
According to the writer, why is the choice of your second job important?

A. It will affect your future job prospects. 

B. It will last longer than your first job. 

C. It will be difficult to change if you don’t like it. 

D. It should give you the opportunity to study.

Câu 807 :
"It” in the passage refers to_______. 

A. first job 

B. second job 

C. application

D. career

Câu 808 :
If you have a desire to travel, when does the writer suggest that you do it?

A. straight after you have left college

B. when you are unable to find a permanent job

C. after you have done some temporary work

D. between the first and second job

Câu 809 :
What does the phrase “you have got it out of your system” in passage mean?

A. You have planned your career sensibly.

B. You are an experienced traveler.

C. You have satisfied your wish to travel.

D. You have learned to look after yourself.

Câu 810 :
How did Juliette Davidson benefit from the experience of her first job?

A. It was good introduction to working in an office.

B. She met a variety of interesting people.

C. It enabled her to earn enough money to travel.

D. She learnt how to use a word processor.

Câu 811 :
In what way is Juliette’s current job better her first job?

A. She has more impressive job title. 

B. She now know how to start her own business . 

C. She has been able to extend her skills. 

D. She is more involve in the community.

Câu 812 :
Facial expressions __________. 

A. cannot convey emotions 

B. cannot reinforce spoken words 

C. can only express negative attitudes 

D. can be either visible or fleeting

Câu 813 :
Gestures __________. 

A. can do nothing with a conversation 

B. can clarify the meaning of verbal messages an

C. may interrupt the flow of a conversation usual

D. can end a conversation more quickly than

Câu 814 :
According to the writer, “A picture is worth a thousand words” means __________

A. a picture of a face is more valuable than a thousand words

B. a picture is more important than a thousand words

C. facial gestures can convey a lot of meanings

D. he has just bought a picture with a thousand words on it

Câu 818 :
From what we read we know that now Cambridge is _______.

A. visited by international tourists

B. a city without wall

C. a city of growing population

D. a city that may have a wall around

Câu 819 :
Around what time did the university begin to appear?

A. In the 8th century

B. In the 13th century

C. In the 9th century

D. In the 15th century

Câu 820 :
Why do most visitors come to Cambridge?

A. To see the university

B. To study in the colleges in Cambridge

C. To find the classroom buildings

D. To use the libraries of the university

Câu 821 :
After which year did the town really begin developing?

A. 800

B. 875

C. 1845

D. 1951

Câu 822 :
Read the following passage and mark the letter A, B, C, or D on your answer sheet to indicate the correct answer to each of the questions
When John Mills was going to fly in an aeroplane for the first time, he was frightened. He did not like the idea of being thousands of feet up in the air. “ I also didn‟t like the fact that I wouldn‟t be in control,” says John. “I‟m a terrible passenger in the car. When somebody else is driving, I tell them what to so. It drives everybody crazy.” However John couldn‟t avoid flying any longer. It was the only way he could visit his grandchildren in Canada. “I had made up my mind that I was going to do it, I couldn‟t let my son, his wife and their three children travel all the way here to visit me. It would be so expensive for them and I know Tom‟s business isn‟t doing so well at the moment – it would also be tiring for the children – it‟s a nine-hour flight!” he says. To get ready for the flight John did lots of reading about aeroplanes. When he booked his seat, he was told that he would be flying on a Boeing 747, which is better known as a jumbo jet. “I needed to know as much as possible before getting in that plane. I suppose it was a way of making myself feel better. The Boeing 747 is the largest passenger aircraft in the world at the moment. The first one flew on February 9th 1969 in the USA. It can carry up to 524 passengers and 3.400 pieces of luggage. The fuel for aeroplanes is kept in the wings and the 747’s wings are so big that they can carry enough fuel for an average car to be able to travel 16,000 kilometres a year for 70 years. Isn‟t that unbelievable? Even though I had discovered all this very interesting information about the jumbo, when I saw it for the first time, just before I was going to travel to Canada, I still couldn‟t believe that something so enormous was going to get up in the air and fly. I was even more impressed when I saw how big it was inside with hundreds of people!” The biggest surprise of all for John was the flight itself. “The take-off itself was much smoother than I expected although I was still quite scared until we were in the air. In the end, I managed to relax, enjoy the food and watch one of the movies and the view from the window was spectacular. I even managed to sleep for a while! Of course,” continues John, “the best reward of all was when I arrived in Canada and saw my son and his family, particularly my beautiful grandchildren. Suddenly, I felt so silly about all the years when I couldn‟t even think of getting on a plane. I had let my fear of living stop me from seeing the people I love most in the world. I can visit my son and family as often as I like now!” 
Why did John Mills fly in an aeroplane?

A. He wanted to go on holiday

B. He wanted to try it.

C. He wanted to see his family

D. He had to travel on business

Câu 823 :
Why did John read about aeroplane?

A. He wanted to know how they work.

B. It was his hobby.

C. It made him feel safer.

D. He had found a book on them.

Câu 824 :
What happened when he saw the jumbo jet for the first time?

A. He felt much safer.

B. He liked the shape of it.

C. He couldn‟t believe how big it was.

D. He thought the wings were very small.

Câu 825 :
How did John feel when the aeroplane was taking off?

A. excited

B. happy

C. sad

D. frightened

Câu 826 :
What surprised John most about the flight?

A. that he liked the food

B. that he was able to sleep

C. that there was a movie being shown

D. that the view was good

Câu 827 :
How did John feel about his fears in the end?

A. He thought he had wasted time being afraid.

B. He realized it was okay to be afraid.

C. He hoped his grandchildren weren‟t afraid of flying.

D. He realized that being afraid kept him safe.

Câu 828 :
Read the following passage and mark the letter A, B, C, or D on your answer sheet to indicate the correct answer to each of the questions
Although noise, commonly defined as unwanted sound, is a widely recognized form of pollution, it is very difficult to measure because the discomfort experienced by different individuals is highly subjective and, therefore, variable. Exposure to lower levels of noise may be slightly irritating, whereas exposure to higher levels may actually cause hearing loss. Particularly in congested urban areas, the noise produced as a byproduct of our advancing technology causes physical and psychological harm, and detracts from the quality of life for those who are exposed to it. Unlike the eyes, which can be covered by the eyelids against strong light, the ear has no lid, and is, therefore, always open and vulnerable; noise penetrates without protection. Noise causes effects that the hearer cannot control and to which the body never becomes accustomed. Loud noises instinctively signal danger to any organism with a hearing mechanism, including human beings. In response, heartbeat and respiration accelerate, blood vessels constrict, the skin pales, and muscles tense. In fact, there is a general increase in functioning brought about by the flow of adrenaline released in response to fear, and some of these responses persist even longer than the noise, occasionally as long as thirty minutes after the sound has ceased. Because noise is unavoidable in a complex, industrial society, we are constantly responding in the same way that we would respond to danger. Recently, researchers have concluded that noise and our response may be much more than an annoyance. It may be a serious threat to physical and psychological health and well-being, causing damage not only to the ear and brain but also to the heart and stomach. We have long known that hearing loss is America‟s number one nonfatal health problem, but now we are learning that some of us with heart disease and ulcers may be victims of noise as well. Fetuses exposed to noise tend to be overactive, they cry easily, and they are more sensitive to gastrointestinal problems after birth. In addition, the psychic effect of noise is very important. Nervousness, irritability, tension, and anxiety increase affecting the quality of rest during sleep, and the efficiency of activities during waking hours, as well as the way that we interact with each other. 
Which of the following is the author‟s main point?

A. Hearing loss is America‟s number one nonfatal health problem.

B. Loud noises signal danger.

C. Noise may pose a serious threat to our physical and psychological health.

D. The ear is not like the eye.

Câu 829 :
According to the passage, what is noise?

A. A byproduct of technology.

B. Physical and psychological harm.

C. Congestion.

D. Unwanted sound

Câu 830 :
Why is noise difficult to measure?

A. All people do not respond to it in the same way

B. It causes hearing loss.

C. It is unwanted.

D. People become accustomed to it.

Câu 831 :
The word ‘congested’ in paragraph 1 could best be replaced by ___.

A. hazardous

B. crowded

C. polluted

D. rushed

Câu 832 :
The word it in the first paragraph refers to ___.

A. the quality of life

B. advancing technology

C. the noise

D. physical and psychological harm

Câu 835 :
The phrase ‘as well as’ in paragraph 4 is closest in meaning to ___.

A. after all

B. instead

C. also

D. regardless

Câu 836 :
It can be inferred from the passage that the eye ___.

A. responds to fear

B. enjoys greater protection than the ear

C. increases functions

D. is damaged by noise

Câu 837 :
Read the following passage and mark the letter A, B, C, or D on your answer sheet to indicate the correct answer to each of the questions
Herman Melville, an American author best known today for his novel Moby Dick, was actually more popular during his lifetime for some of his other works. He traveled extensively and used the knowledge gained during his travels as the basis for his early novels. In 1837, at the age of eighteen, Melville signed as a cabin boy on a merchant ship that was to sail from his Massachusetts home to Liverpool, England. His experiences on this trip served as a basis for the novel Redburn (1849). In 1841 Melville set out on a whaling ship headed for the South Seas. After jumping ship in Tahiti, he wandered around the islands of Tahiti and Moorea. This South Sea island sojourn was a backdrop to the novel Omoo (1847). After three years away from home, Melville joined up with a U.S. naval frigate that was returning to the eastern United States around Cape Horn. The novel White-Jacket (1850) describes this lengthy voyage as a navy seaman. With the publication of these early adventure novels, Melville developed a strong and loyal following among readers eager for his tales of exotic places and situations. However, in 1851, with the publication of Moby Dick, Melville's popularity started to diminish. Moby Dick, on one level the saga of the hunt for the great white whale, was also a heavily symbolic allegory of the heroic struggle of man against the universe. The public was not ready for Melville's literary metamorphosis from romantic adventure to philosophical symbolism. It is ironic that the novel that served to diminish Melville's popularity during his lifetime is the one for which he is best known today. 
The main subject of the passage is_________.

A. Melville’s travels

B. Moby Dick

C. Melville’s personal background

D. the popularity of Melville’s novels.

Câu 838 :
The word “basis” in paragraph 1 is closest in meaning to_________.

A. background

B. message

C. bottom

D. dissertation

Câu 839 :
According to the passage, Melville’s early novels were__________.

A. published while he was traveling

B. completely fictional

C. all about his work on whaling ships

D. based on his travel experience

Câu 840 :
The passage implies that Melville stayed in Tahiti because____________.

A. he had unofficially left his ship

B. he was on leave while his ship was in port

C. he had finished his term of duty

D. he had received permission to take a vacation in Tahiti

Câu 841 :
How did the publication of Moby Dick affect Melville’s popularity?

A. His popularity remained as strong as ever.

B. It caused his popularity to decrease.

C. His popularity increased immediately.

D. It had no effect on his popularity.

Câu 842 :
According to the passage, Moby Dick is__________.

A. symbolic of humanity fighting the universe

B. a single-faceted work

C. a short story about a whale

D. a 47 adventure

Câu 844 :
The word “metamorphosis” in paragraph 2 is closest in meaning to_________.

A. descent

B. circle

C. mysticism

D. change

Câu 845 :
Read the following passage and mark the letter A, B, C, or D on your answer sheet to indicate the correct answer to each of the questions
The handling and delivery of mail has always been a serious business, underpinned by the trust of the public in requiring timeliness, safety, and confidentiality. After early beginnings using horseback and stagecoach, and although cars and trucks later replaced stagecoaches and wagons, the Railway Mail Service still stands as one of America’s most resourceful and exciting postal innovations. This service began in 1832, but grew slowly until the Civil War. Then from 1862, by sorting the mail on board moving trains, the Post Office Department was able to decentralize its operations as railroads began to crisscross the nation on a regular basis, and speed up mail delivery. This service lasted until 1974. During peak decades of service, railway mail clerks handled 93% of all non-local mail and by 1905 the service had over 12,000 employees. Railway Post Office trains used a system of mail cranes to exchange mail at stations without stopping. As a train approached the crane, a clerk prepared the catcher arm which would then snatch the incoming mailbag in the blink of an eye. The clerk then booted out the outgoing mailbag. Experienced clerks were considered the elite of the Postal Service’s employees, and spoke with pride of making the switch at night with nothing but the curves and feel of the track to warn them of an upcoming catch. They also worked under the greatest pressure and their jobs were considered to be exhausting and dangerous. In addition to regular demands of their jobs they could find themselves the victims of train wrecks and robberies. As successful as it was, “mail-on-the-fly” still had its share of glitches. If they hoisted the train’s catcher arm too soon, they risked hitting switch targets, telegraph poles or semaphores, which would rip the catcher arm off the train. Too late, and they would miss an exchange. 
Which of the following can be inferred from the first paragraph?

A. There was a high turnover of railway mail clerks.

B. The development of the mail roads during the second half of the 19th century enabled Post Office Department to focus on timeliness.

C. The Post Office Department was more concerned about speeding up mail delivery than the safety of its clerks.

D. Mail was often lost or damaged as it was exchanged on the mail crane.

Câu 846 :
The word “elite” in the second paragraph is closest in meaning to___________.

A. majority

B. superior

C. more capable

D. leader

Câu 847 :
What does the passage mainly discuss?

A. How the mail cranes exchanged the mail.

B. Improvements in mail handling and delivery.

C. How Post Office Trains handled the mail without stopping.

D. The skills of experienced clerks.

Câu 849 :
The word “glitches” in the third paragraph can be replaced by________.

A. accidents

B. blames

C. advantages

D. problems

Câu 850 :
Which of the following is TRUE according to the passage?

A. The clerk booted out the outgoing mailbag before snatching the incoming bag.

B. Clerks couldn’t often see what they were doing.

C. The Railway Mail clerk’s job was considered elite because it was safe and exciting.

D. Despite their success, railway mail clerks only handled a small proportion of all non-local mail.

Câu 851 :
The handling and delivery of mail has always been a serious business, underpinned by the trust of the public in requiring timeliness, safety, and confidentiality. After early beginnings using horseback and stagecoach, and although cars and trucks later replaced stagecoaches and wagons, the Railway Mail Service still stands as one of America’s most resourceful and exciting postal innovations. This service began in 1832, but grew slowly until the Civil War. Then from 1862, by sorting the mail on board moving trains, the Post Office Department was able to decentralize its operations as railroads began to crisscross the nation on a regular basis, and speed up mail delivery. This service lasted until 1974. During peak decades of service, railway mail clerks handled 93% of all non-local mail and by 1905 the service had over 12,000 employees. Railway Post Office trains used a system of mail cranes to exchange mail at stations without stopping. As a train approached the crane, a clerk prepared the catcher arm which would then snatch the incoming mailbag in the blink of an eye. The clerk then booted out the outgoing mailbag. Experienced clerks were considered the elite of the Postal Service’s employees, and spoke with pride of making the switch at night with nothing but the curves and feel of the track to warn them of an upcoming catch. They also worked under the greatest pressure and their jobs were considered to be exhausting and dangerous. In addition to regular demands of their jobs they could find themselves the victims of train wrecks and robberies. As successful as it was, “mail-on-the-fly” still had its share of glitches. If they hoisted the train’s catcher arm too soon, they risked hitting switch targets, telegraph poles or semaphores, which would rip the catcher arm off the train. Too late, and they would miss an exchange.
The public expects the following three services in handling and delivery of mail except ______.

A. safety

B. accuracy

C. confidentiality

D. timeliness

Câu 852 :
Read the following passage and mark the letter A, B, C, or D on your answer sheet to indicate the correct answer to each of the questions
Some animal behaviorists argue that certain animals can remember past events, anticipate future ones, make plans and choices, and coordinate activities within a group. These scientists, however, are cautious about the extent to which animals can be credited with conscious processing. Explanations of animal behavior that leave out any sort of consciousness at all and ascribe actions entirely to instinct leave many questions unanswered. One example of such unexplained behavior: Honeybees communicate the sources of nectar to one another by doing a dance in a figure-eight pattern. The orientation of the dance conveys the position of the food relative to the sun's position in the sky, and the speed of the dance tells how far the food source is from the hive. Most researchers assume that the ability to perform and encode the dance is innate and shows no special intelligence. But in one study, when experimenters kept changing the site of the food source, each time moving the food 25 percent farther from the previous site, foraging honeybees began to anticipate where the food source would appear next. When the researchers arrived at the new location, they would find the bees circling the spot, waiting for their food. No one has yet explained how bees, whose brains weigh four ten-thousandths of an ounce, could have inferred the location of the new site. Other behaviors that may indicate some cognition include tool use. Many animals, like the otter who uses a stone to crack mussel shells, are capable of using objects in the natural environment as rudimentary tools. One researcher has found that mother chimpanzees occasionally show their young how to use tools to open hard nuts. In one study, chimpanzees compared two pairs of food wells containing chocolate chips. One pair might contain, say, five chips and three chips, the other our chips and three chips. Allowed to choose which pair they wanted, the chimpanzees almost always chose the one with the higher total, showing some sort of summing ability. Other chimpanzees have learned to use numerals to label quantities of items and do simple sums. 
What does the passage mainly discuss?

A. Observations that suggest consciousness in animal behavior.

B. The use of food in studies of animal behavior.

C. The role of instinct in animal behavior.

D. Differences between the behavior of animals in their natural environments and in laboratory experiments.

Câu 853 :
Which of the following is NOT discussed as an ability animals are thought to have?

A. Communicating emotions

B. Remembering past experiences

C. Selecting among choices

D. Anticipating events to come

Câu 854 :
What did researchers discover in the study of honeybees discussed in paragraph 2?

A. Bees are able to travel at greater speeds than scientists thought.

B. The bees were able to determine in advance where scientists would place their food.

C. Changing the location of food caused bees to decrease their dance activity.

D. The bees could travel 25% farther than scientists expected.

Câu 855 :
It can be inferred from the passage that brain size is assumed to ___________.

A. be an indicator of cognitive ability

B. be related to food consumption

C. correspond to levels of activity

D. vary among individuals within a species

Câu 856 :
Why are otters and mussel shells included in the discussion in paragraph 3?

A. To provide that certain species demonstrate greater ability in tool use than other species.

B. To provide an example of tool use among animals.

C. To show that animals are very good at using objects in their habitat.

D. To provide an example of the use of weapons among animals.

Câu 857 :
The word “rudimentary” is closest in meaning to ___________.

A. technical

B. basic

C. superior

D. original

Câu 858 :
Scientists concluded from the experiment with chimpanzees and chocolate chips that chimpanzees __________.

A. prefer to work in pairs or groups

B. have difficulty selecting when given choices

C. lack abilities that other primates have

D. exhibit behavior that indicates certain mathematical abilities

Câu 859 :
Read the following passage and mark the letter A, B, C, or D on your answer sheet to indicate the correct answer to each of the questions
In the very distant geological past, all animals were aquatic. The very first vertebrates or animals with backbones, of which we have any fossil record, lived in water. These vertebrates, the fish, were adapted to underwater living. Their streamlined bodies were covered with scales to reduce surface friction: they had muscular tails so that they could swim swiftly in such a dense medium as water; and they were endowed with gills for breathing underwater. Descendants of fishtype ancestors crossed the seashore barrier and accommodated themselves to life on land. As amphibians, they possessed limbs instead of fins and lungs instead of gills. But they never became completely free of the bonds that tied them to the water; even today many amphibians return to the water to lay their eggs. Millions of years after the first clumsy amphibians crawled over the land, newer types of land dwellers appeared, these animals give rise to the present-day reptiles and mammals. They were completely converted for land dwelling, with bodies and biological activities far different from those of fish. With these special adaptations, mammals have been able to colonize the woods and meadows, the deserts and high mountains, often far removed from the sea. 
Of the animals with backbones, the first to appear were ______

A. fish

B. amphibians

C. birds

D. mammals

Câu 860 :
Fish are suited to underwater life because of their ______

A. Gills

B. All of the answers

C. Streamlines shapes

D. Scales

Câu 862 :
The word “descendants” is closest in meaning to _____.

A. Grandchildren

B. Ancestors

C. Descenders

D. Off spring

Câu 863 :
An example of an amphibian’s incomplete adaptation to land life is…….

A. need to keep its skin wet

B. return to water to lay eggs

C. inability to breathe air

D. Scales-covered skin

Câu 864 :
Animals found desert living possible only _____

A. when they were fully adapted to land

B. after they could walk on two feet

C. when they became amphibious

D. if they migrated to sea periodically

Câu 865 :
The seashore was a barrier for descendants of fish-type because………

A. crossing it required bodily changes.

B. every attempt to cross it ended in death.

C. the land once rose much higher above the sea.

D. once they crossed, there was no return.

Câu 866 :
The adaptation process described in the article was completed……

A. By the receding of the sea

B. Through biological changes

C. Over millions of years and Through biological changes

D. Over millions of years

Câu 867 :
Read the following passage and mark the letter A, B, C, or D on your answer sheet to indicate the correct answer to each of the questions
Telecommuting is some form of computer communication between employees’ homes and offices. For employees whose job involve sitting at a terminal or word processor entering data or typing reports, the location of the computer is of no consequence. If the machine can communicate over telephone lines, when the work is completed, employees can dial the office computer and transmit the material to their employers. A recent survey in USA Today estimates that there are approximately 8,7 million telecommuters. But although the numbers are rising annually, the trend does not appear to be as significant as predicted when Business Week published “The Portable Executive” as its cover story a few years ago. Why hasn’t telecommuting become more popular? Clearly, change simply takes time. But in addition, there has been active resistance on the part of many managers. These executives claim that supervising the telecommuters in a large work force scattered across the country would be too difficult, or, at least, systems for managing them are not yet developed, thereby complicating the manager’s responsibilities. It is also true that employees who are given the option of telecommuting are reluctant to accept the opportunity. Most people feel that they need regular interaction with a group, and many are concerned that they will not have the same consideration for advancement if they are not more visible in the office setting. Some people feel that even when a space in their homes is set aside as a work area, they never really get away from the office. 
With which of the following topics is the passage primarily concerned?

A. An overview of telecommuting

B. The failure of telecommuting

C. The advantages of telecommuting

D. A definition of telecommuting

Câu 868 :
How many Americans are involved in telecommuting?

A. More than predicted in Business Week

B. More than 8 million

C. Fewer than last year

D. Fewer than estimated in USA Today

Câu 869 :
The phrase “of no consequence” means ________.

A. of no use

B. irrelevant

C. of no good

D. unimportant

Câu 870 :
The author mentions all of the following as concerns of telecommuting EXCEPT ________.

A. the opportunities for advancement

B. the different system of supervision

C. the lack of interaction with a group

D. the work place is in the home

Câu 871 :
The word “them” in the second paragraph refers to ________.

A. telecommuters

B. systems

C. executives

D. responsibilities

Câu 872 :
The reason why telecommuting has not become popular is that the employees ________

A. need regular interaction with their families.

B. are worried about the promotion if they are not seen at the office.

C. feel that a work area in their home is away from the office.

D. are ignorant of telecommuting.

Câu 873 :
The word “reluctant” in line 13 can best be replaced by ________.

A. opposite

B. willing

C. hesitant

D. typical

Câu 874 :
Read the following passage and mark the letter A, B, C, or D on your answer sheet to indicate the correct answer to each of the questions
In my experience , freshmen today are different from those I knew when I started as a counselor and professor 25 years ago . College has always been demanding both academically and socially . But students now are less mature and often not ready for the responsibility of being in college .
. It is really too easy to point the finger at parents who protect their children from life’s obstacle. Parents, who handle every difficulty and every other responsibility for their children writing admission essays to picking college courses, certainly may contribute to their children’s lack of coping strategies. But we can look even more broadly to the social trends of today. How many people do you know who are on medication to prevent anxiety or depression? The number of students who arrive at college already medicated for unwanted emotions has increased dramatically in the past 10 years. We, as a society, don’t want to “feel” anything unpleasant and we certainly don’t want our children to “suffer”. The resulting problem is that by not experiencing negative emotions, one does not learn the necessary skills to tolerate and negotiate adversity. As a psychologist, I am well aware of the fact that some individuals suffer from depression and anxiety and can benefit from treatment, but I question the growing number of medicated adolescents today. Our world is more stressful in general because of the current economic and political realities, but I don’t believe that the college experience itself is more intense today than that of the past 10 years. What I do think is that many students are often not prepared to be young “adults” with all the responsibilities of life. What does this mean for college faculty and staff? We are required to assist in the basic parenting of these students – the student who complains that the professor didn’t remind her of the due date for an assignment that was clearly listed on the syllabus and the student who cheats on an assignment in spite of careful instructions about plagiarism. As college professors, we have to explain what it means to be an independent college student before we can even begin to teach. As parents and teachers we should expect young people to meet challenges. To encourage them in this direction, we have to step back and let them fail and pick themselves up and move forward. This approach needs to begin at an early age so that college can actually be a passage to independent adulthood. 
According to the writer, students today are different from those she knew in that they are ____________.

A. too ready for college 

B. not as mature 

C. not so academic 

D. responsible for their work

Câu 875 :
The word “handle” in parapgraph 2 mostly means ____________.

A. deal with

B. gain benefits from

C. lend a hand to

D. point at

Câu 876 :
According to the writer, students’ difficulties to cope with college life are partly due to ____________.

A. the absence of parents’ protection 

B. the lack of parental support 

C. the over-parenting from parents 

D. the lack of financial support

Câu 877 :
Which of the following is NOT TRUE according to the passage?

A. College faculty and staff are required to help in the parenting of problematic students. 

B. The college experience itself is more intense today than that of the past 10 years. 

C. Our world is more stressful because of the current economic and political situation. 

D. Our society certainly doesn’t want our children to experience unpleasant things.

Câu 878 :
Students who are not well – prepared to be young “adults” with all the responsibilities of life will need ______.

A. to be assigned more housework from adults

B. to be given more social responsibilities

C. to be encouraged to meet challenges

D. daily coaching from their teachers

Câu 879 :
According to the writer, failure in life and less support from parents will ______.

A. help students to learn to stand on their own feet 

B. allow students to learn the first lesson in their lives 

C. defeat students from the very beginning 

D. discourage students and let them down forever

Câu 880 :
What is probably the writer’s attitude in the passage?

A. Praising

B. Indifferent

C. Humorous

D. Critial

Câu 881 :
Read the following passage and mark the letter A, B, C, or D on your answer sheet to indicate the correct answer to each of the questions
We get great pleasure from reading. The more advanced a man is, the greater delight he will find in reading. The ordinary man may think that subjects like philosophy or science are very difficult and that if philosophers and scientists read these subjects, it is not for pleasure. But this is not true. The mathematician finds the same pleasure in his mathematics as the school boy in an adventure story. For both, it is a play of the imagination, a mental recreation and exercise. The pleasure derived from this activity is common to all kinds of reading. But different types of books give us different types of pleasure. First in order of popularity is novel-reading. Novels contain pictures of imaginary people in imaginary situations, and give us an opportunity of escaping into a new world very much like our world and yet different from it. Here we seem to live a new life, and the experience of this new life gives us a thrill of pleasure. Next in order of popularity are travel books, biographies and memoirs. These tell us tales of places we have not seen and of great men in whom we are interested. Some of these books are as wonderful as novels, and they have an added value that they are true. Such books give us knowledge, and we also find immense pleasure in knowing details of lands we have not seen and of great men we have only heard of. Reading is one of the greatest enjoyments of life. To book-lovers, nothing is more fascinating than a favorite book. And, the ordinary educated man who is interested and absorbed in his daily occupation wants to occasionally escape from his drudgery into the wonderland of books for recreation and refreshment. 
What does the passage mainly discuss?

A. Different types of books

B. Different kinds of reading

C. Reading as an exercise for the brain

D. Reading as a pleasurable activity

Câu 882 :
According to paragraph 1, which of the following is NOT true?

A. Ordinary people may think that philosophy and science are difficult.

B. Reading about mathematics is mentally entertaining for a mathematician.

C. Philosophers and scientists do not read for pleasure.

D. A more advanced person takes greater pleasure in reading.

Câu 883 :
The word “derived” in paragraph 2 is closest in meaning to ______.

A. differed

B. established

C. bought

D. obtained

Câu 884 :
The word “it” in paragraph 2 refers to ______.

A. a new life

B. our world

C. an opportunity

D. a thrill of pleasure

Câu 885 :
The word “immense” in paragraph 3 is closest in meaning to ______.

A. great

B. limited

C. personal

D. controlled

Câu 886 :
According to the passage, travel books, biographies and memoirs______.

A. are wonderful novels

B. tell stories of well-known places

C. are less popular than novels

D. are more valuable than novels

Câu 887 :
According to paragraph 4, which of the following is the most fascinating to booklovers?

A. A daily occupation

B. An ordinary educated man

C. The wonderland

D. The wonderland

Câu 888 :
Read the following passage and mark the letter A, B, C, or D on your answer sheet to indicate the correct answer to each of the questions
Since water is the basis of life, composing the greater part of the tissues of all living things, the crucial problem of desert animals is to survive in a world where sources of flowing water are rare. And since man’s inexorable necessity is to absorb large quantities of water at frequent intervals, he can scarcely comprehend that many creatures of the desert pass their entire lives without a single drop. Uncompromising as it is, the desert has not eliminated life but only those forms unable to withstand its desiccating effects. No moistskinned, water-loving animals can exist there. Few large animals are found. The giants of the North American desert are the deer, the coyote, and the bobcat. Since desert country is open, it holds more swift-footed running and leaping creatures than the tangled forest. Its population is largely nocturnal, silent, filled with reticence, and ruled by stealth. Yet they are not emaciated. Having adapted to their austere environment, they are as healthy as animals anywhere else in the word. The secret of their adjustment lies in the combination of behavior and physiology. None could survive if, like mad dogs and Englishmen, they went out in the midday sun; many would die in a matter of minutes. So most of them pass the burning hours asleep in cool, humid burrows underneath the ground, emerging to hunt only by night. The surface of the sun-baked desert averages around 150 degrees, but 18 inches down the temperature is only 60 degrees. 
The author mentions all the following as examples of the behavior of desert animals EXCEPT ..........

A. they are watchful and quiet

B. they sleep during the day

C. they dig home underground

D. they are noisy and aggressive

Câu 889 :
We can infer from the passage that .......... 

A. healthy animals live longer lives 

B. living things adjust to their environment 

C. desert life is colorful and diverse 

D. water is the basis of desert life

Câu 890 :
According to the passage, creatures in the desert ...........

A. are smaller and fleeter than forest animals

B. are more active during the day than those in the tangled forest

C. live in an accommodating environment

D. are not as healthy as those anywhere else in the world

Câu 891 :
The phrase “those forms” in the passage refers to all of the following EXCEPT ..........

A. moist-skinned animals

B. many large animals

C. water-loving animals

D. the coyote and the bobcat

Câu 892 :
The word “them” means ....... 

A. animals
B. minutes C. people D. water

B. minutes 

C. people

D. water

Câu 893 :
The word “emaciated” in the passage mostly means .......

A. wild

B. unmanageable

C. cunning

D. unhealthy

Câu 894 :
Man can hardly understand why many animals live their whole life in the desert, as .........

A. water is an essential part of his existence

B. very few large animals are found in the desert

C. sources of flowing water are rare in a desert

D. water composes the greater part of the tissues of living things

Câu 895 :
The title for this passage could be ............. 

A. “Man’s Life in a Desert Environment” 

B. “Desert Plants” 

C. “Animal Life in a Desert Environment”

D. “Life Underground”

Câu 896 :
Read the following passage and mark the letter A, B, C, or D on your answer sheet to indicate the correct answer to each of the questions
Learning means acquiring knowledge or developing the ability to perform new behaviors . It is common to think of learning as something that takes place in school , but much of human learning occurs outside the classroom , and people continue to learn throughout their lives .
  Even before they enter school, young children learn to walk, to talk, and to use their hands to manipulate toys, food, and other objects. They use all of their senses to learn about the sights, sounds, tastes, and smells in their environments. They learn how to interact with their parents, siblings, friends, and other people important to their world. When they enter school, children learn basic academic subjects such as reading, writing, and mathematics.They also continue to learn a great deal outside the classroom . They learn which behaviors are likely to be rewarded and which are likely to be punished . They learn social skills for interacting with other children. After they finish school, people must learn to adapt to the many major changes that affect their lives, such as getting married, raising children, and finding and keeping a job. Because learning continues throughout our lives and affects almost everything we do, the study of learning is important in many different fields. Teachers need to understand the best ways to educate children. Psychologists, social workers, criminologists, and other human-service workers need to understand how certain experiences change people’s behaviors. Employers, politicians, and advertisers make use of the principles of learning to influence the behavior of workers, voters, and consumers.
  Learning is closely related to memory, which is the storage of information in the brain. Psychologists who study memory are interested in how the brain stores knowledge, where this storage takes place, and how the brain later retrieves knowledge when we need it. In contrast, psychologists who study learning are more interested in behavior and how behavior changes as a result of a person’s experiences.
  There are many forms of learning, ranging from simple to complex. Simple forms of learning involve a single stimulus. A stimulus is anything perceptible to the senses, such as a sight, sound, smell, touch, or taste. In a form of learning known as classical conditioning, people learn to associate two stimuli that occur in sequence, such as lightning followed by thunder. In operant conditioning, people learn by forming an association between a behavior and its consequences (reward or punishment). People and animals can also learn by observation - that is, by watching others perform behaviors. More complex forms of learning include learning languages, concepts, and motor skills. 
Getting married, raising children, and finding and keeping a job are mentioned in paragraph 2 as examples of ______.

A. the situations in which people cannot teach themselves 

B. the ways people’s lives are influenced by education

C. the changes to which people have to orient themselves 

D. the areas of learning which affect people’s lives

Câu 897 :
According to the passage, which of the following is learning in broad view comprised of?

A. Acquisition of social and behavioural skills 

B. Knowledge acquisition outside the classroom 

C. Acquisition of academic knowledge 

D. Knowledge acquisition and ability development

Câu 898 :
According to the passage, what are children NOT usually taught outside the classroom?

A. literacy and calculation

B. right from wrong

C. life skills

D. interpersonal communication

Câu 899 :
The passage mainly discusses ______. 

A. practical examples of learning inside the classroom 

B. simple forms of learning 

C. application of learning principles to formal education 

D. general principles of learning

Câu 900 :
It can be inferred from the passage that social workers, employers, and politicians concern themselves with the study of learning because they need to ______.

A. make the objects of their interest more aware of the importance of learning 

B. understand how a stimulus relates to the senses of the objects of their interest 

C. thoroughly understand the behaviours of the objects of their interest 

D. change the behaviours of the objects of their interest towards learning

Câu 901 :
Read the following passage and mark the letter A, B, C, or D on your answer sheet to indicate the correct answer to each of the questions
Learning means acquiring knowledge or developing the ability to perform new behaviors . It is common to think of learning as something that takes place in school , but much of human learning occurs outside the classroom , and people continue to learn throughout their lives .
  Even before they enter school, young children learn to walk, to talk, and to use their hands to manipulate toys, food, and other objects. They use all of their senses to learn about the sights, sounds, tastes, and smells in their environments. They learn how to interact with their parents, siblings, friends, and other people important to their world. When they enter school, children learn basic academic subjects such as reading, writing, and mathematics.They also continue to learn a great deal outside the classroom . They learn which behaviors are likely to be rewarded and which are likely to be punished . They learn social skills for interacting with other children. After they finish school, people must learn to adapt to the many major changes that affect their lives, such as getting married, raising children, and finding and keeping a job. Because learning continues throughout our lives and affects almost everything we do, the study of learning is important in many different fields. Teachers need to understand the best ways to educate children. Psychologists, social workers, criminologists, and other human-service workers need to understand how certain experiences change people’s behaviors. Employers, politicians, and advertisers make use of the principles of learning to influence the behavior of workers, voters, and consumers.
  Learning is closely related to memory, which is the storage of information in the brain. Psychologists who study memory are interested in how the brain stores knowledge, where this storage takes place, and how the brain later retrieves knowledge when we need it. In contrast, psychologists who study learning are more interested in behavior and how behavior changes as a result of a person’s experiences.
  There are many forms of learning, ranging from simple to complex. Simple forms of learning involve a single stimulus. A stimulus is anything perceptible to the senses, such as a sight, sound, smell, touch, or taste. In a form of learning known as classical conditioning, people learn to associate two stimuli that occur in sequence, such as lightning followed by thunder. In operant conditioning, people learn by forming an association between a behavior and its consequences (reward or punishment). People and animals can also learn by observation - that is, by watching others perform behaviors. More complex forms of learning include learning languages, concepts, and motor skills. 
According to the passage, the study of learning is important in many fields due to ______.

A. the need for certain experiences in various areas 

B. the exploration of the best teaching methods 

C. the influence of various behaviours in the learning process 

D. the great influence of the on-going learning process

Câu 902 :
The word “retrieves” in paragraph 4 is closest in meaning to _______.

A. recovers

B. creates

C. generates

D. gains

Câu 903 :
Read the following passage and mark the letter A, B, C, or D on your answer sheet to indicate the correct answer to each of the questions
Animals have an intuitive awareness of quantities. They know without analysis the difference between a number of objects and a smaller number. In his book “ The natural History of Selboure ” (1786 ) , the naturalist Gilbert White tells how he surreptitiously removed one egg a day from a plover’s nest , and how the mother laid another egg each day to make up for the missing one . He noted that other species of birds ignore the absence of a single egg but abandon their nests if more than one egg has been removed. It has also been noted by naturalists that a certain type of wasp always provides five – never four, never six - caterpillars for each of their eggs so that their young have something to eat when the eggs hatch . Research has also shown that both mice and pigeons can be taught to distinguish between odd and even numbers of food pieces. These and similar accounts have led some people to infer that creatures other than humans can actually count. They also point to dogs that have been taught to respond to numerical questions with the correct number of barks, or to horses that seem to solve arithmetic problems by stomping their hooves the proper number of times. Animals respond to quantities only when they are connected to survival as a species – as in the case of the eggs – or survival as individuals - as in the case of food. There is no transfer to other situations or from concrete reality to the abstract notion of numbers. Animals can “count” only when the objects are present and only when the numbers involved are small – not more than seven or eight. In lab experiments, animals trained to “count” one kind of object were unable to count any other type. The objects, not the numbers, are what interest them. Animals admittedly remarkable achievements simply do not amount to evidence of counting, nor do they reveal more than innate instincts, refined by the genes of successive generations, or the results of clever, careful conditioning by trainers. 
What is the main idea of this passage?

A. Although animals may be aware of quantities, they cannot actually count.

B. Of all animals, dogs and horses can count best.

C. Careful training is required to teach animals to perform tricks involving numbers.

D. Animals cannot “count” more than one kind of object.

Câu 904 :
Why does the author refer to Gilbert White’s book in line 2?

A. To indicate that more research is needed in this field.

B. To show how attitudes have changed since1786.

C. To provide evidence that some birds are aware of quantities.

D. To contradict the idea that animals can count.

Câu 905 :
The word “surreptitiously” is closest in meaning to ...........

A. quickly

B. occasionally

C. stubbornly

D. secretly

Câu 906 :
The word “odd” refers to which of the following?

A. numbers such as 1, 3, 5 and so on

B. lucky numbers

C. numbers such as 2, 4, 6 and so on

D. unusual numbers

Câu 908 :
The word “accounts” is closest in meaning to ...........

A. reasons

B. reports

C. deceptions

D. invoices

Câu 910 :
Read the following passage and mark the letter A, B, C, or D on your answer sheet to indicate the correct answer to each of the questions
Until recently, most American entrepreneurs were men. Discrimination against women in business, the demands of caring for families, and lack of business training had kept the number of women entrepreneurs small. Now, however, businesses owned by women account for more than $40 billion in annual revenues, and this figure is likely to continue rising throughout the 1990s. As Carolyn Doppelt Gray, an official of the Small Business Administration, has noted, "The 1970s was the decade of women entering management, and the 1980s turned out to be the decade of the woman entrepreneur". What are some of the factors behind this trend? For one thing, as more women earn advanced degrees in business and enter the corporate world, they are finding obstacles. Women are still excluded from most executive suites. Charlotte Taylor, a management consultant, had noted, "In the 1970s women believed if they got an MBA and worked hard they could become chairman of the board. Now they've found out that isn't going to happen, so they go out on their own". In the past, most women entrepreneurs worked in "women's" fields: cosmetics and clothing, for example. But this is changing. Consider ASK Computer Systems, a $22-million-a-year computer software business. It was founded in 1973 by Sandra Kurtzig, who was then a housewife with degrees in math and engineering. When Kurtzig founded the business, her first product was software that let weekly newspapers keep tabs on their newspaper carriers-and her office was a bedroom at home, with a shoebox under the bed to hold the company's cash. After she succeeded with the newspaper software system, she hired several bright computer-science graduates to develop additional programs. When these were marketed and sold, ASK began to grow. It now has 200 employees, and Sandra Kurtzig owns $66.9 million of stock. Of course, many women who start their own businesses fail, just as men often do. They still face hurdles in the business world, especially problems in raising money; the banking and finance world is still dominated by men, and old attitudes die hard. Most businesses owned by women are still quite small. But the situation is changing; there are likely to be many more Sandra Kurtzigs in the years ahead.
What is the main idea of this passage?

A. The computer is especially lucrative for women today.

B. Women today are better educated than in the past, making them more attractive to the business world.

C. Women are better at small business than men are.

D. Women today are opening more business of their own.

Câu 911 :
The word “excluded” is closest meaning to ___________ .

A. often invited to

B. decorators of

C. not permitted in

D. charged admission to

Câu 912 :
All of the following were mentioned in the passage as detriments to women in the business world EXCEPT ___________.

A. Women were required to stay at home with their families.

B. Women faced discrimination in business.

C. Women lacked ability to work in business.

D. Women were not trained in business.

Câu 913 :
According to the passage, Charlotte Taylor believes that women in 1970s ___________

A. were unrealistic about their opportunities in business management.

B. had fewer obstacles in business than they do today.

C. were unable to work hard enough to success in business.

D. were still more interested in education than business opportunities.

Câu 914 :
The author mentions the “ shoesbox under the bed” in order to ___________.

A. show the resourcefulness of Sandra Kurtzig

B. show the frugality of women in business

C. point out that initially the financial resources of Sandra Kurtzig’s business were limited

D. suggest that the company needed to expand

Câu 915 :
The expression “ keep tabs on” is closest meaning to ___________.

A. pay the salaries of

B. keep records of

C. provide transportation for

D. recognize the appearance of

Câu 916 :
The word “hurdles” can be best replaced by ___________.

A. fences

B. obstacles

C. questions

D. small groups

Câu 917 :
It can be inferred from the passage that the author believes that business operated by women are small because ___________.

A. Many women fail at large businesses. 

B. Women are not able to borrow money easily. 

C. Women prefer a small intimate setting. 

D. Women can’t deal with money

Câu 918 :
Read the following passage and mark the letter A, B, C, or D on your answer sheet to indicate the correct answer to each of the questions
Who talks more - men or women? Most people believe that women talk more. However, linguist Deborah Tannen, who has the studied the communication style of men and women, says that this is a stereotype. According to Tannen, women are more verbal - talk more - in private situations, where they use conversation as the “glue’ to hold relationships together. But, she says, men talk more in public situations, where they use conversation to exchange information and gain status. Tannen points out that we can see these difference even in children. Little girls often play with one ‘best friend’ and their play includes a lot of conversation. Little boys often play games in groups, their play usually involves more doing than talking. In school, girls are often better at verbal skills, while boys are often better at mathematics. A recent study at Emory University helps to shed light on the roots of this difference. Researchers studied conversation between children aged 3-6 and their parents. They found evidence that parents talk very differently to their sons than they do to their daughters. The startling conclusion was that parents use more language with their girls. Specifically, when parents talk with their daughters, they use more descriptive language and more details. There is also far more talk about emotions, especially with daughters than with sons. 
Which sentence best expresses the main idea of the first paragraph?

A. Women talk more than men on the whole

B. Women’s talking is a stereotype

C. Women talk more in private, and men talk more in public

D. Little boys and little girls have different ways of playing

Câu 919 :
Which word is similar in meaning to the word “glue”?

A. games

B. sticky substance

C. rope

D. means

Câu 920 :
Which of the following phrases best explains the meaning of the word “verbal”?

A. Deriving from verbs

B. Connected with use of spoken language

C. Using very loud noise

D. Being very talkative

Câu 921 :
The word “they” refers to___________.

A. Situations 

B. Men 

C. Men and women 

D. Women

Câu 922 :
Which sentence best expresses the main idea of the second paragraph?

A. Researchers have studied the conversations of children and their parents.

B. Parents do not talk much about sadness with their sons.

C. Study at Emory University can help to explain the differences between communication styles of boys and girls.

D. An Emory University study found than parents talk more with their daughters than with their sons.

Câu 923 :
Which of the following statement is TRUE about the passage?

A. Parents give more love to their daughters than to their sons.

B. Boys don’t like to be with their parents as much as girls do.

C. Parents use more language to talk with their daughters.

D. Boys don’t like showing emotions.

Câu 924 :
Read the following passage and mark the letter A, B, C, or D on your answer sheet to indicate the correct answer to each of the questions
A number of factors related to the voice reveal the personality of the speaker. The first is the broad area of communication, which includes imparting information by use of language, communicating with a group or an individual and specialized communication through performance. A person conveys thoughts and ideas through choice of words, by a tone of voice that is pleasant or unpleasant, gentle or harsh, by the rhythm that is inherent within the language itself, and by speech rhythms that are flowing and regular or uneven and hesitant, and finally, by the pitch and melody of the utterance. When speaking before a group, a person's tone may indicate uncertainty or fright, confidence or calm. At interpersonal levels, the tone may reflect ideas and feelings over and above the words chosen, or may believe them. Here, the participant's tone can consciously or unconsciously reflect intuitive sympathy or antipathy, lack of concern or interest, fatigue, anxiety, enthusiasm or excitement, all of which are usually discernible by the acute listener. Public performance is a manner of communication that is highly specialized with its own techniques for obtaining effects by voice and /or gesture. The motivation derived from the text, and in the case of singing, the music, in combination with the performers skills, personality, and ability to create empathy will determine the success of artistic, political, or pedagogic communication. Second, the voice gives psychological clues to a person's self-image, perception of others, and emotional health. Self-image can be indicated by a tone of voice that is confident, pretentious, shy, aggressive, outgoing, or exuberant, to name only a few personality traits. Also the sound may give a clue to the facade or mask of that person, for example, a shy person hiding behind an overconfident front. How a speaker perceives the listener’s receptiveness, interest, or sympathy in any given conversation can drastically alter the tone of presentation, by encouraging or discouraging the speaker. Emotional health is evidenced in the voice by free and melodic sounds of the happy, by constricted and harsh sound of the angry, and by dull and lethargic qualities of the depressed. 
The word “Here” in the passage refers to__________.

A. interpersonal interaction

B. the tone

C. ideas and feelings

D. words chosen

Câu 925 :
Why does the author mention “artistic, political, or pedagogic communication” in the passage?

A. To introduce the idea of self-image

B. As examples of public performance

C. As examples of basic styles of communication

D. To contrast them to singing

Câu 926 :
The word "derived" in line 13 is closest in meaning to ____________.

A. discussed

B. prepared

C. registered

D. obtained

Câu 927 :
According to the passage, an exuberant tone of voice may be an indication of a person's _______.

A. general physical health

B. personality

C. ability to communicate

D. vocal quality

Câu 928 :
According to the passage, an overconfident front may hide _______.

A. hostility

B. shyness

C. friendliness

D. strength

Câu 929 :
According to the passage, what does a constricted and harsh voice indicate?

A. Lethargy

B. Depression

C. Boredom

D. Anger

Câu 930 :
According to the passage, an exuberant tone of voice may be an indication of a person’s ____________.

A. general physical health

B. personality

C. ability to communicate

D. vocal quality

Câu 931 :
Read the following passage and mark the letter A, B, C, or D on your answer sheet to indicate the correct answer to each of the questions
Researchers in the field of psychology have found that one of the best way to make an important decision, such as choosing a university to attend or business to invest in, involves the utilization of a decision worksheet. Psychologists who study optimization compare the actual decisions made by people to theretical ideal decisions to see how similar they are. Proponents of the worksheet procedure believe that it will yield optimal, that is, the best decisions. Although there are several variations on the exact format that worksheets cam take, they are all similar in their essential aspects. Worksheets require defining the problem in a clear and concise way and then listing all possible solutions to the problem. Next, the pertinent considerations that all possible solutions to the problem. Next, the pertinent considerations that will be affected by each decision are listed, and the relative importance of each consideration or consequence is determined. Each consideration is assigned a numerical value to reflect its relative importance. A decision is mathematically calculated by adding these values together. The alternative with the highest number of points emerges as the best decision. Since most important problems are multifaceted, there are several alternatives to choose from, each with unique advantages and disadvantages. One of the benefits of a pencil and paper decision- making procedure is that it permits people to deal with more variables than their minds can generally comprehend and remember. On the average, people can keep about seven ideas in their minds at once. A worksheet can be especially useful when the decision involves a large number of variables with complex relationships. A realistic example for many college students is the question "What will I do after graduation?" A graduate might seek a position that offers specialized training, pursue an advanced degree, or travel abroad for a year. A decision-making worksheet begins with a succinct statement of the problem that will also help to narrow it. It is important to be clear about the distinction between long-range and immediate goals because long-range goals often involve a different decision than shortrange ones. Focusing on long-range goals, a graduating student might revise the question above to "What will I do after graduation that will lead to a successful career?" 
According to decision-worksheet theory, an optimal decision is defined as one that ............

A. has the fewest variables to consider

B. uses the most decision worksheet

C. has the most points assigned to it

D. is agreed to by the greatest number of people

Câu 932 :
Which of the following terms is defined in the passage?

A. Proponents (paragraph 1)

B. Optimal (paragraph 1)

C. Variables (paragraph 3)

D. Long-range goals (paragraph 4)

Câu 933 :
The word "essential" in paragraph 1 is closest in meaning to .......

A. introductory

B. fundamental

C. changeable

D. beneficial

Câu 934 :
The author states that "On the average, people can keep about seven ideas in their minds at once" (paragraph 3) to explain that .....

A. people can learn to keep more than seven ideas in their minds with practice.

B. most decisions involve seven steps.

C. human mental capacity has limitations.

D. some people have difficulty making minor as well as major decisions.

Câu 935 :
The word "it" in paragraph 4 refers to .........

A. worksheet

B. problem

C. distinction

D. decision

Câu 936 :
What does the passage mainly discuss?

A. A comparison of actual decisions and ideal decisions.

B. Research on how people make decisions.

C. A tool to assist in making complex decisions.

D. Differences between long-range and short-range decision making.

Câu 937 :
Of the following steps, which occurs BEFORE the others in making a decision worksheet?

A. Listing the consequences of each solution.

B. Writing down all possible solutions.

C. Deciding which consequences are most important.

D. Calculating a numerical summary of each solution.

Câu 938 :
Read the following passage and mark the letter A, B, C, or D on your answer sheet to indicate the correct answer to each of the questions
" When a snow leopard stalks its prey among the mountain walls, it moves softly, slowly, " explains Indian biologist Raghunandan Singh Chundawat, who has studied the animal for years. " If it knocks a stone loose, it will reach out a foot to stop it from falling and making noise." One might be moving right now, perfectly silent, maybe close by. But where? And how many are left to see? Best known for its spotted coat and long distinctive tail, the snow leopard is one of the world’s most secretive animals. These elusive cats can only be found high in the remote, mountainous regions of central Asia. For this reason, and because they hunt primarily at night, they are very rarely seen. Snow leopards have been officially protected since 1975, but enforcing this law has proven difficult. Many continue to be killed for their fur and body parts, which are worth a fortune on the black market. In recent years, though, conflict with local herders has also led to a number of snow leopard deaths. This is because the big cats kill the herders’ animals, and drag the bodies away to eat high up in the mountains. As a result of these pressures, the current snow leopard population is estimated at only 4,000 to 7,000, and some fear that the actual number may already have dropped below 3,500. The only way to reverse this trend and bring these cats back from near extinction, say conservationists, is to make them more valuable alive than dead. Because farming is difficult in Central Asia’s cold, dry landscape, traditional cultures depend mostly on livestock (mainly sheep and go a t s) t o s u r viv e in t h e s e m o u n t ain o u s r e gio n s. A t nig h t, w h e n s n o w leopards hunt, herders’ animals are in danger of snow leopard attacks. Losing only a few animals can push a family into desperate poverty. “The wolf comes and kills, eats, and goes somewhere else,” said one herder, “but snow leopards are always around. They have killed one or two animals many time. Everybody wanted to finish this leopard.” To address this problem, local religious leaders have called for an end to snow leopard killings, saying that these wild animals have the right to exist peacefully. They’ve also tried to convince people that the leopards are quite rare and thus it is important to protect them. Financial incentives are also helping to slow snow leopard killings. The organization Snow Leopard Conservancy–India has established Himalayan Homestays, a program that sends visitors to the region to herders’ houses. For a clean room and bed, meals with the family, and an introduction to their culture, visitors pay about ten U.S. dollars a night. Having guests once every two weeks through the tourist season provides the herders with enough income to replace the animals lost to snow leopards. In addition, Homestays helps herders build protective fences that keep out snow leopards. The organization also conducts environmental classes at village schools and trains Homestays members as nature guides, available for hire. In exchange, the herders agree not to kill snow leopards. In Mongolia, a project called Snow Leopard Enterprises (SLE) helps herder communities earn extra money in exchange for their promise to protect the endangered cat. Women in Mongolian herder communities make a variety of products—yarn for making clothes, decorative floor rugs, and toys—using the wool from their herds. SLE buys these items from herding families and sells them abroad. Herders must agree to protect the snow leopards and to encourage neighbors to do the same. The arrangement increases herders’ incomes by 10 to 15 percent and “elevates” the status of the women. If no one in the community kills the protected animals over the course of a year, the program members are rewarded with a 20 percent bonus in addition to the money they’ve already made. An independent review in 2006 found no snow leopard killings in areas where SLE operates. Today the organization continues to add more communities. Projects like the Homestays program in India and SLE’s business in Mongolia are doing well, though they cover only a small part of the snow leopard’s homeland, they make the leopards more valuable to more people each year. If these programs continue to do well, the snow leopard may just have a figting chance. 
Which of these is NOT true about the Himalayan Homestays program?

A. The organization helps herders to build fences. 

B. Herders provide accommodation to guests.

C. Some herders work as nature guides.

D. Visitors pay $10 a week to stay at a herder’s house.

Câu 939 :
The purpose of the projects mentioned is .................

A. to persuade herders to live on eco-tourism not raising animals 

B. to elevate the status of the women in the communities

C. to increase the population of snow leopards in wildlife reserves

D. to make local people have a sustainable life to protect snow leopards

Câu 940 :
The phrase “this trend” in paragraph 4 refers to ............

A. the fall in the snow leopard population 

B. the pressures caused by the black market 

C. increasing conflict with the herders

D. the opinions of conservationists

Câu 941 :
What is the main idea of the third paragraph?

A. Local herders are uncooperative in attempts to save snow leopards. 

B. The snow leopard’s endangerment is due to in part to the black market. 

C. Snow leopards are killed for their fur and body parts.

D. It is difficult to enforce the laws made to protect the snow leopard

Câu 942 :
The word “address” in in paragraph 6 can be replaced by ......

A. solve 

B. locate 

C. discuss

D. change

Câu 943 :
Why is the Mongolian women's status in the community “elevated”?

A. They can encourage their neighbours. 

B. They are saving money for the snow leopards. 

C. They are earning money for the community.

D. They are living higher up in the mountain.

Câu 944 :
According to conservationists, what is the best way to save the snow leopard?

A. Create a nature park where they can be free

B. Move herders away from where the snow leopard lives 

C. Pass laws to punish people who kill snow leopards for their fur

D. Make people recognize the value of living snow leopards.

Câu 945 :
The word “bodies” in paragraph 3 refers to ...........

A. the big cats 

B. snow leopards 

C. local herders

D. the herders’ animals

Câu 946 :
Read the following passage and mark the letter A, B, C, or D on your answer sheet to indicate the correct answer to each of the questions
Before the 1500’s, the western plains of North America were dominated by farmers. One group, the Mandans, lived in the upper Missouri River country, primarily in present – day North Dakota.They had large villages of houses built close together. The tight arrangement enabled the Mandans to protect themselves more easily from the attacks of others who might seek to obtain some of the food these highly capable farmers stored from one year to the next. The women had primary responsibility for the fields. They had to exercise considerable skill to produce the desired results, for their northern location meant fleeting growing seasons. Winter often lingered; autumn could be ushered in by severe frost. For good measure, during the spring and summer, drought, heat, hail, grasshoppers, and other frustrations might await the wary grower. Under such conditions, Mandan women had to grow maize capable of weathering adversity. They began as early as it appeared feasible to do so in the spring, clearing the land, using fire to clear stubble from the fields and then planting. From this point until the first green corn could be harvested, the crop required labor and vigilance. Harvesting proceeded in two stages. In August the Mandans picked a smaller amount of the crop before it had matured fully. This green corn was boiled, dried and shelled, with some of the maize slated for immediate consumption and the rest stored in animal – skin bags. Later in the fall, the people picked the rest of the corn. They saved the best of the harvest for seeds or for trade, with the remainder eaten right away or stored for alter use in underground reserves. With appropriate banking of the extra food, the Mandans protected themselves against the disaster of crop failure and accompany hunger. The woman planted another staple, squash, about the first of June, and harvested it near the time of the green corn harvest. After they picked it, they sliced it, dried it, and strung the slices before they stored them. Once again, they saved the seeds from the best of the year’s crop. The Mandans also grew sunflowers and tobacco; the latter was the particular task of the older men. 
What is the main topic of the passage?

A. The problems encountered by farmers specializing in growing once crop

B. The agricultural activities of a North American Society

C. Various ways corn can be used.

D. Weather conditions on the western plains.

Câu 947 :
The Mandans built their houses close together in order to _____.

A. guard their supplies of food

B. protect themselves against the weather

C. share farming implements

D. allow more room for growing corn

Câu 948 :
The word “enabled” in the first paragraph is closest in meaning to _____.

A. covered

B. helped

C. reminded

D. isolated

Câu 949 :
Why does the author believe that the Mandans were skilled farmers?

A. They developed effective fertilizers.

B. They could grow crops in most types of soil.

C. They could grow crops despite adverse weather.

D. They developed new varieties of corn.

Câu 950 :
The word “consumption” in the paragraph is closest in meaning to _____.

A. decay

B. conversation

C. eating

D. planting

Câu 951 :
Before the 1500’s, the western plains of North America were dominated by farmers. One group, the Mandans, lived in the upper Missouri River country, primarily in present – day North Dakota.They had large villages of houses built close together. The tight arrangement enabled the Mandans to protect themselves more easily from the attacks of others who might seek to obtain some of the food these highly capable farmers stored from one year to the next. The women had primary responsibility for the fields. They had to exercise considerable skill to produce the desired results, for their northern location meant fleeting growing seasons. Winter often lingered; autumn could be ushered in by severe frost. For good measure, during the spring and summer, drought, heat, hail, grasshoppers, and other frustrations might await the wary grower. Under such conditions, Mandan women had to grow maize capable of weathering adversity. They began as early as it appeared feasible to do so in the spring, clearing the land, using fire to clear stubble from the fields and then planting. From this point until the first green corn could be harvested, the crop required labor and vigilance. Harvesting proceeded in two stages. In August the Mandans picked a smaller amount of the crop before it had matured fully. This green corn was boiled, dried and shelled, with some of the maize slated for immediate consumption and the rest stored in animal – skin bags. Later in the fall, the people picked the rest of the corn. They saved the best of the harvest for seeds or for trade, with the remainder eaten right away or stored for alter use in underground reserves. With appropriate banking of the extra food, the Mandans protected themselves against the disaster of crop failure and accompany hunger. The woman planted another staple, squash, about the first of June, and harvested it near the time of the green corn harvest. After they picked it, they sliced it, dried it, and strung the slices before they stored them. Once again, they saved the seeds from the best of the year’s crop. The Mandans also grew sunflowers and tobacco; the latter was the particular task of the older men.
Which of the following processes does the author imply was done by both men and women?

A. clearing fields

B. Harvesting squash

C. Harvesting corn

D. Planting corn

Câu 952 :
The word “disaster” in paragraph 4 is closest in meaning to _____.

A. control

B. history

C. catastrophe

D. avoidance

Câu 953 :
The word “them” in the last paragraph refers to _____.

A. women

B. seeds

C. slices

D. the Mandans

Câu 954 :
Which of the following crops was cultivated primarily by men?

A. Sunflower

B. Corn

C. Squash

D. Tobacco

Câu 955 :
Throughout the passage, the author implies that the Mandans _____

A. valued individuality

B. were very adventurous

C. were open to strangers

D. planned for the future

Câu 956 :
The topic of the passage is ________.

A. the development of thunderstorms and squall lines 

B. the devastating effects of tornadoes 

C. cumulus and cumulonimbus clouds 

D. the power of tornadoes

Câu 957 :
The word “mechanisms” in paragraph 1 is most likely ________.

A. machines

B. motions

C. methods

D. materials

Câu 958 :
The word “benign” in paragraph 3 is closest in meaning to ________.

A. harmless

B. beneficial

C. ferocious

D. spectacular

Câu 959 :
It can be inferred from the passage that, in summer, ________.

A. there is not a great temperature differential between higher and lower altitudes

B. the greater temperature differential between higher and lower altitudes makes thunderstorms more likely to occur

C. there is not much cold air higher up in the atmosphere

D. the temperature of rising air drops more slowly than it does in winter

Câu 960 :
The expression “in concert” in paragraph 3 could best be replaced by ________.

A. as a chorus

B. with other musicians

C. as a cluster

D. in a performance

Câu 961 :
The word “itself” in paragraph 3 refers to ________.

A. a large-scale collision

B. a squall line

C. an advancing cold front

D. a layer of warm and moist air

Câu 962 :
All of the following are mentioned in the passage about supercells EXCEPT that they ________.

A. are of short duration

B. have circling winds

C. have extraordinary power

D. can give birth to tornadoes

Câu 963 :
This reading would most probably be assigned in which of the following courses?

A. Geography

B. Meteorology

C. Marine Biology

D. Chemistry

Câu 964 :
Read the following passage and mark the letter A, B, C, or D on your answer sheet to indicate the correct answer to each of the questions
A considerable body of research has demonstrated a correlation between birth order and aspects such as temperament and behavior , and some psychologists believe that birth order significantly affects the development of personality. Psychologist Alfred Adler was a pioneer in the study of the relationship between birth order and personality. A key point in his research and in the hypothesis that he developed based on it was that it was not the actual numerical birth position that affected personality; instead, it was the similar response s in large numbers of families to children in specific birth order positions that had an effect. For example, first-borns, who have their parents to themselves initially and do not have to deal with siblings in the first part of their lives, tend to have their first socialization experiences with adults and therefore tend to find the process of peer socialization more difficult. In contrast, later-born children have to deal with siblings from the first moment of their lives and therefore tend to have stronger socialization skills.
 Numerous studies since Adler’s have been conducted on the effect of birth order and personality. These studies have tended to classify birth order types into four different categories: first-born, second-born and/or middle, last, and only child.
 Studies have consistently shown that first-born children tend to exhibit similar, positive and negative personality traits. First-borns have consistently been linked with academic achievement in various studies; in one study, the number of National Merit scholarship winners who are first-borns was found to be equal to the number of second-and third-borns combined. First-borns have been found to be more responsible and assertive than those born in other birth-order positions and tend to rise to positions of leadership more often than others; more first-borns have served in the U.S. Congress and as U.S. presidents than have those born in other birth-orderpositions. However, studies have shown that first-borns tend to be more subject to stress and were considered problem children more often than later- borns.
 Second-born and/or middle children demonstrate markedly different tendencies from first-borns. They tend to feel inferior to the older child or children because it is difficult for them to comprehend that their lower level of achievement is a function of age rather than ability, and they often try to succeed in areas other than those in which their older sibling or siblings excel. They tend to be more trusting, accepting, and focused on others than the more self-centered first-borns, and they tend to have a comparatively higher level of success in team sports than do first-borns or only children, who more often excel in individual sports.
 The last-born child is the one who tends to be the eternal baby of the family and thus often exhibits a strong sense of security. Last- borns collectively achieve the highest degree of social success and demonstrate the highest levels of self-esteem of all the birth-order positions.They often exhibit less competitiveness than older brothers and likely to take part in less competitive group games or in social organizations such as sororities and fraternities.
 Only children tend to exhibit some of the main characteristics of first - borns and some of the characteristics of last - borns. Only children tend to exhibit the strong sense of security and self - esteem exhibited by lastborns while, like first - borns, they are more achievement oriented and more likely than middle - or last - borns to achieve academic success. However, only children tend to have the most problems establishing close relationships and exhibit a lower need for affiliation than other children. 
The word body " in paragraph 1 could best be replaced by______

A. amout

B. organization

C. corpse

D. skelrton

Câu 965 :
The word “key” in paragraph 1 could best be replaced by________.

A. secret

B. studied

C. significant

D. locked

Câu 966 :
The word “it” in paragraph 1 refers to________.

A. component 

B. research 

C. hypothesis 

D. personality

Câu 967 :
What is stated in paragraph 1 about Adler?

A. He had found that the responses by family members had little to do with personality. 

B. He believed that it was the actual birth order that affected personality.

C. He was one of the first to study the effect of birth order on personality. 

D. He was the only one to study birth order.

Câu 968 :
Which of the sentences below expresses the essential information in the italic sentence in paragraph 3 ? Incorrect choices change the meaning in important ways or leave out essential information. 

A. Because first - borns tend to be very assertive , they are uncomfortable serving in government positions 

B. Several examples support the idea that first - borns have characteristics that make them leaders 

C. An interesting fact that is difficult to explain is that many first - borns have served in high government positions

D. In spite of certain characteristics that first - borns possess , many of them become leaders 

Câu 969 :
The word “accepting” in paragraph 4 is closest in meaning to________

A. respectable

B. affectionate

C. admissible

D. torelant

Câu 970 :
Which of the following is NOT true ?

A. Middle children tend to have a preference for team sports

B. First - borns tend to do well in individual sports 

C. Only children tend to prefer individual over team sports

D. Last - borns tend to prefer games with fierce competition 

Câu 971 :
The phrase " more achievement oriented " in the paragraph 6 is closest in meaning to______

A. more skilled as leaders 

B. more aware of surroundings 

C. more directly involved 

D. more focused on accomplishments

Câu 972 :
Read the following passage and mark the letter A, B, C, or D on your answer sheet to indicate the correct answer to each of the questions
Certainly no creature in the sea is odder than the common sea cucumber. All living creature, especially human beings, have their peculiarities, but everything about the little sea cucumber seems unusual. What else can be said about a bizarre animal that, among other eccentricities, eats mud, feeds almost continuously day and night but can live without eating for long periods, and can be poisonous but is considered supremely edible by gourmets? For some fifty million years, despite all its eccentricities, the sea cucumber has subsisted on its diet of mud. It is adaptable enough to live attached to rocks by its tube feet, under rocks in shallow water, or on the surface of mud flats. Common in cool water on both Atlantic and Pacific shores, it has the ability to such up mud or sand and digest whatever nutrients are present. Sea cucumbers come in a variety of colors, ranging from black to reddish-brown to sand-color and nearly white. One form even has vivid purple tentacle. Usually the creatures are cucumber-shapedhence their name-and because they are typically rock inhabitants, this shape, combine with flexibility, enables them to squeeze into crevices where they are safe from predators and ocean currents. Although they have voracious appetites, eating day and night, sea cucumbers have the capacity to become quiescent and live at a low metabolic rate-feeding sparingly or not at all for long periods, so that the marine organisms that provide their food have a chance to multiply. If it were not for this faculty, they would devour all the food available in a short time and would probably starve themselves out of existence. But the most spectacular thing about the sea cucumber is the way it defends itself. Its major enemies are fish and crabs, when attacked, it squirts all its internal organs into the water. It also casts off attached structures such as tentacles. The sea cucumber will eviscerate and regenerate itself if it is attached or even touched; it will do the same if the surrounding water temperature is too high or if the water becomes too polluted. 
What does the passage mainly discuss?

A. Places where the sea cucumber can be found

B. The reason for the sea cucumber’s name

C. How to identify the sea cucumber

D. What makes the sea cucumber unusual

Câu 973 :
The word “bizarre” is closest meaning to ..........

A. rare

B. simple

C. marine

D. odd

Câu 974 :
The fourth paragraph of the passage primarily discusses ....

A. the eating habits of sea cucumbers

B. the food sources of sea cucumbers

C. the reproduction of sea cucumbers

D. threats to sea cucumbers’ existence

Câu 975 :
According to the passage, why is the shape of sea cucumbers important?

A. It makes them attractive to fish.

B. It helps them to protect themselves from danger.

C. It helps them to digest their food.

D. It makes it easier for them to move through the mud.

Câu 976 :
The phrase “casts off” is closest in meaning to ..........

A. gets rid of

B. grows again

C. grabs

D. uses as a weapon

Câu 977 :
The words “this faculty” refers to the sea cucumber’s ability to ....

A. live at a low metabolic rate

B. squeeze into crevices

C. devour all available food in a short time

D. suck up mud or sand

Câu 978 :
What can be inferred about the defense mechanisms of the sea cucumber?

A. They are very sensitive to surrounding stimuli.

B. They are almost useless.

C. They are similar to those of most sea creatures.

D. They require group cooperation.

Câu 979 :
Read the following passage and mark the letter A, B, C, or D on your answer sheet to indicate the correct answer to each of the questions
Scientists do not yet thoroughly understand just how the body of an individual becomes sensitive to a substance that is harmless or even wholesome for the average person. Milk, wheat, and egg, for example, rank among the most healthful and widely used foods. Yet these foods can cause persons sensitive to them to suffer greatly. At first, the body of the individual is not harmed by coming into contact with the substance. After a varying interval of time, usually longer than a few weeks, the body becomes sensitive to it, and an allergy has begun to develop. Sometimes it's hard to figure out if you have a food allergy, since it can show up so many different ways. Your symptoms could be caused by many other problems. You may have rashes, hives, joint pains mimicking arthritis, headaches, irritability, or depression. The most common food allergies are to milk, eggs, seafood, wheat, nuts, seeds, chocolate, oranges, and tomatoes. Many of these allergies will not develop if these foods are not fed to an infant until her or his intestines mature at around seven months. Breast milk also tends to be protective. Migraines can be set off by foods containing tyramine, phenathylamine, monosodium glutamate, or sodium nitrate. Common foods which contain these are chocolate, aged cheeses, sour cream, red wine, pickled herring, chicken livers, avocados, ripe bananas, cured meats, many Oriental and prepared foods (read the labels!). Some people have been successful in treating their migraines with supplements of B-vitamins, particularly B6 and niacin. Children who are hyperactive may benefit from eliminating food additives, especially colorings, and foods high in salicylates from their diets. 
The topic of this passage is______________.

A. reactions to foods

B. food and nutrition

C. infants and allergies

D. a good diet

Câu 980 :
According to the passage, the difficulty in diagnosing allergies to foods is due to ______________.

A. the vast number of different foods we eat

B. lack of a proper treatment plan

C. the similarity of symptoms of the allergy to other problems

D. the use of prepared formula to feed babies

Câu 981 :
The word "symptoms" is closest in meaning to______________.

A. indications

B. diet

C. diagnosis

D. prescriptions

Câu 982 :
The phrase "set off" is closest in meaning to______________.

A. relieved

B. identified

C. avoided

D. triggered

Câu 983 :
What can be inferred about babies from this passage?

A. They can eat almost anything.

B. They should have a carefully restricted diet as infants.

C. They gain little benefit from being breast fed.

D. They may become hyperactive if fed solid food too early.

Câu 984 :
The word "hyperactive" is closest in meaning to _____.

A. overly active

B. unusually low activity

C. excited

D. inquisitive

Câu 985 :
The author states that the reason that infants need to avoid certain foods related to allergies has to do with the infant's ________.

A. lack of teeth

B. poor metabolism

C. underdeveloped intestinal tract

D. inability to swallow solid foods

Câu 986 :
Read the following passage and mark the letters A, B, C or D on your answer sheet to indicate the correct answer to each of the questions
The rapid transcontinentat settlement and these new urban industrial circumstances of the last half of the 19th century were accompanied by the development of national literature of great abundance and variety. New themes, new forms, new subjects, new regions, new authors, new audiences all emerged in the literature of this half century. As a result, at the onset of World War I, the spirit and substance of American literature had evolved remarkably, just as its center of production had shifted from Boston to New York in the late 1880s and the sources of its energy to Chicago and the Midwest. No longer was it produced, at least in its popular forms, in the main by solemn, typically moralistic men from New England and the Old South; no longer were polite, well-dressed, grammatically correct, middle-class young people the only central characters in its narratives; no longer were these narratives to be set in exotic places and remote times; no longer, indeed, were fiction, poetry, drama, and formal history the chief acceptable forms of literary expression; no longer, finally, was literature read primarily by young, middle class women. In sum, American literature in these years fulfilled in considerable measure the condition Walt Whitman called for in 1867 in describing Leaves of Grass: it treats, he said of his own major work, each state and region as peers "and expands from them, and includes the world ... connecting an American citizen with the citizens of all nations." 
The main idea of this passage is________.

A. that the new American literature was less provincial than the old

B. that World War I caused a dramatic change in America

C. that centers of culture shifted from East to West

D. that most people were wary of the new literature

Câu 987 :
It can be inferred from the passage that the previous passage probably discussed ________.

A. the importance of tradition to writers

B. new developments in industrialization and population shifts

C. the fashions and values of 19th century America

D. the limitations of American literature to this time

Câu 988 :
The word “evolved” in paragraph two is closest in meaning to ________.

A. became famous

B. turned back

C. diminished

D. changed

Câu 989 :
The word “exotic" in paragraph two is closest in meaning to ________.

A. urban

B. unusual

C. well-known

D. old-fashioned

Câu 990 :
The word “it” in the second paragraph refers to ________.

A. the population

B. the energy

C. American literature

D. the manufacturing

Câu 991 :
The author uses the word “indeed” in the second paragraph for what purpose?

A. To emphasize the contrast he is making.

B. For variety in a lengthy paragraph.

C. To wind down his argument.

D. To show a favorable attitude to these forms of literature.

Câu 992 :
The phrase “these years” in the third paragraph refers to ________.

A. 1850-1900

B. the 1900s

C. the early 1800s

D. the present

Câu 993 :
It can be inferred from the passage that Walt Whitman________.

A. disliked urban life

B. was disapproving of the new literature

C. wrote Leaves of Grass

D. was an international diplomat

Câu 994 :
Read the following passage and mark the letter A, B, C, or D on your answer sheet to indicate the correct answer to each of the questions
There are a number of natural disasters that can strike across the globe. Two that are frequently linked to one another are earthquakes and tsunamis. Both of them can cause a great amount of devastation when they hit. However, tsunamis are the direct result of earthquakes and cannot happen without them. The Earth has three main parts. They are the crust, the mantle, and the core. The crust is the outer layer of the Earth. It is not a single piece of land. Instead, it is comprised of a number of plates. There are a few enormous plates and many smaller ones. These plates essentially rest upon the mantle, which is fluid. As a result, the plates are in constant - yet slow - motion. The plates may move away from or towards other plates. In some cases, they collide violently with the plates adjoining them. The movement of the plates causes tension in the rock. Over a long time, this tension may build up. When it is released, an earthquake happens. Tens of thousands of earthquakes happen every year. The vast majority are so small that only scientific instruments can perceive them. Others are powerful enough that people can feel them, yet they cause little harm or damage. More powerful earthquakes, however, can cause buildings, bridges, and other structures to collapse. They may additionally injure and skill thousands of people and might even cause the land to change it appearance. Since most of the Earth's surface is water, numerous earthquakes happen beneath the planet's oceans. Underwater earthquakes cause the seafloor to move. This results in the displacement of water in the ocean. When this occurs, a tsunami may form. This is a wave that forms on the surface and moves in all directions from the place where the earthquake happened. A tsunami moves extremely quickly and can travel thousands of kilometres. As it approaches land, the water near the coast gets sucked out to sea. This causes the tsunamis to increase in height. Minutes later, the tsunami arrives. A large tsunami - one more than ten meters in height - can travel far inland. As it does that, it can flood the land, destroy human settlements, and kill large numbers of people. 
Which of the following statements does paragraph 1 support?

A. Earthquakes cause more destruction than tsunamis.

B. A tsunami happens in tandem with an earthquake.

C. The most severe type of natural disaster is an earthquake.

D. Earthquakes frequently take place after tsunamis do.

Câu 995 :
The word "it" in bold in paragraph 2 refers to ............

A. the Earth

B. the core

C. the crust

D. the mantle

Câu 996 :
What is the passage mainly about?

A. How earthquakes and tsunamis occur.

B. What kind of damage natural disasters can cause.

C. Why tsunamis are deadlier than earthquakes.

D. When earthquakes are the most likely to happen.

Câu 997 :
The word "adjoining" in bold in paragraph 2 is closest in meaning to ...........

A. bordering

B. residing

C. approaching

D. appearing

Câu 998 :
The word "perceive" in bold in paragraph 3 is closest in meaning to ..............

A. comprehend

B. detect

C. prevent

D. locate

Câu 999 :
Which of the following is true regarding the crust?

A. There many separate pieces that make it up.

B. It is the smallest of the Earth's three layers.

C. It is thicker on land than it is under the water.

D. The mantle beneath it keeps it from moving too much.

Câu 1000 :
Based on the passage, what is probably true about tsunamis?

A. They kill more people each year than earthquakes.

B. They are able to move as fast as the speed of sound.

C. They cannot damage ships sailing on the ocean.

D. They can be deadly to people standing near shore.

Câu 1001 :
There are a number of natural disasters that can strike across the globe. Two that are frequently linked to one another are earthquakes and tsunamis. Both of them can cause a great amount of devastation when they hit. However, tsunamis are the direct result of earthquakes and cannot happen without them. The Earth has three main parts. They are the crust, the mantle, and the core. The crust is the outer layer of the Earth. It is not a single piece of land. Instead, it is comprised of a number of plates. There are a few enormous plates and many smaller ones. These plates essentially rest upon the mantle, which is fluid. As a result, the plates are in constant - yet slow - motion. The plates may move away from or towards other plates. In some cases, they collide violently with the plates adjoining them. The movement of the plates causes tension in the rock. Over a long time, this tension may build up. When it is released, an earthquake happens. Tens of thousands of earthquakes happen every year. The vast majority are so small that only scientific instruments can perceive them. Others are powerful enough that people can feel them, yet they cause little harm or damage. More powerful earthquakes, however, can cause buildings, bridges, and other structures to collapse. They may additionally injure and skill thousands of people and might even cause the land to change it appearance. Since most of the Earth's surface is water, numerous earthquakes happen beneath the planet's oceans. Underwater earthquakes cause the seafloor to move. This results in the displacement of water in the ocean. When this occurs, a tsunami may form. This is a wave that forms on the surface and moves in all directions from the place where the earthquake happened. A tsunami moves extremely quickly and can travel thousands of kilometres. As it approaches land, the water near the coast gets sucked out to sea. This causes the tsunamis to increase in height. Minutes later, the tsunami arrives. A large tsunami - one more than ten meters in height - can travel far inland. As it does that, it can flood the land, destroy human settlements, and kill large numbers of people.
Which of the following is NOT mentioned in paragraph 3 about earthquakes?

A. How many people they typically kill

B. How often powerful ones take place

C. What kind of damage they can cause

D. How severe the majority of them are

Câu 1002 :
Read the following passage and mark the letter A, B, C or D on your answer sheet to indicate the correct answer to each of the questions
In the 1960s, The Beatles were probably the most famous pop group in the whole world. Since then, there have been a great many groups that have achieved enormous fame, so it is perhaps difficult now to imagine how sensational The Beatles were at that time. They were four boys from the north of England and none of them had any training in music. They started by performing and recording songs by black Americans and they had some success with these songs. Then they started writing their own songs and that was when they became really popular. The Beatles changed pop music. They were the first pop group to achieve great success from songs they had written themselves. After that it became common for groups and singers to write their own songs. The Beatles did not have a long career. Their first hit record was in 1963 and they split up in 1970. They stopped doing live performances in 1966 because it had become too dangerous for them – their fans were so excited that they surrounded them and tried to take their clothes as souvenirs! However, today some of their songs remain as famous as they were when they first came out. Throughout the world many people can sing part of a Beatles song if you ask them. 
The passage is mainly about ............

A. Why the Beatles split up after 7 years

B. The Beatles’ fame and success

C. How the Beatles became more successful than other groups

D. Many people’s ability to sing a Beatles song

Câu 1003 :
The four boys of the Beatles ............... 

A. Were at the same age 

B. Came from a town in the north of England
C. Came from the same family D. Received good training in music

C. Came from the same family 

D. Received good training in music

Câu 1004 :
The word “sensational” is closest in meaning to ...........

A. shocking

B. bad

C. notorious

D. popular

Câu 1005 :
The first songs of the Beatles were ................ 

A. paid a lot of money 

B. broadcast on the radio 

C. written by themselves 

D. written by black Americans

Câu 1006 :
What is not true about the Beatles? 

A. They became famous when they wrote their own songs 

B. They had a long stable career 

C. The members had no training in music 

D. They were afraid of being hurt by fans

Câu 1007 :
The Beatles stopped their live performances because ........

A. They spent more time writing their own songs

B. They did not want to work with each other

C. They had earned enough money

D. They were afraid of being hurt by fans

Câu 1008 :
The tone of the passage is that of ................. 

A. neutral

B. criticism 

C. admiration 

D. sarcasm

Câu 1009 :
Read the following passage and mark the letter A, B, C or D on your answer sheet to indicate the correct answer to each of the questions
Many ants forage across the countryside in large numbers and undertake mass migrations; these activities proceed because one ant lays a trail on the ground for the others to follow. As a worker ant returns home after finding a source of food, it marks the route by intermittently touching its stinger to the ground and depositing a tiny amount of trail (5) pheromone – a mixture of chemicals that delivers diverse messages as the context changes. These trails incorporate no directional information and may be followed by other ants in either direction. Unlike some other messages, such as the one arising from a dead ant, a food trail has to be kept secret from members of other species. It is not surprising then that ant species use (10) a wide variety of compounds as trail pheromones. Ants can be extremely sensitive to these signals. Investigators working with the trail pheromone of the leafcutter ant Atta texana calculated that one milligram of this substance would suffice to lead a column of ants three times around Earth. The vapor of the evaporating pheromone over the trail guides an ant along the way, (15) and the ant detects this signal with receptors in its antennae. A trail pheromone will evaporate to furnish the highest concentration of vapor right over the trail, in what is called a vapor space. In following the trail, the ant moves to the right and left, oscillating from side to side across the line of the trail itself, bringing first one and then the other antenna into the vapor space. As the ant moves to the right, its left antenna arrives in the vapor space. The signal it receives causes it to swing to the left, and the ant then pursues this new course until its right antenna reaches the vapor space. It then swings back to the right, and so weaves back and forth down the trail. 
What does the passage mainly discuss?

A. The mass migration of ants

B. How ants mark and follow a chemical trail

C. Different species of ants around the world

D. The information contained in pheromones

Câu 1010 :
The word “intermittently” in line 4 is closest in meaning to ........

A. periodically

B. ncorrectly

C. rapidly

D. roughly

Câu 1011 :
The phrase “the one” in line 8 refers to a single ................

A. message

B. dead ant

C. food trail

D. species

Câu 1012 :
According to the passage, why do ants use different compounds as trail pheromones?

A. To reduce their sensitivity to some chemicals

B. To attract different types of ants

C. To protect their trail from other species

D. To indicate how far away the food is

Câu 1013 :
The author mentions the trail pheromone of the leafcutter ant in line 11 to point out ........

A. how little pheromone is needed to mark a trail

B. the different types of pheromones ants can produce

C. a type of ant that is common in many parts of the world

D. that certain ants can produce up to one milligram of pheromone

Câu 1014 :
According to the passage, how are ants guided by trail pheromones?

A. They concentrate on the smell of food.

B. They follow an ant who is familiar with the trail

C. They avoid the vapor spaces by moving in a straight line

D. They sense the vapor through their antennae

Câu 1015 :
he word “oscillating“ in line 17 is closest in meaning to ........

A. alling

B. depositing

C. swinging

D. starting

Câu 1016 :
According to the passage, the highest amount of pheromone vapor is found ........

A. in the receptors of the ants

B. just above the trail

C. in the source of food

D. under the soil along the trail

Câu 1017 :
Read the following passage and mark the letter A, B, C or D on your answer sheet to indicate the correct answer to each of the questions
Martin Luther King, Jf., is well- known for his work in civil rights and for his many famous speeches, among which is his moving “ I have a dream” speech. But fewer people know much about King’s childhood. M.L., as he was called, was born in 1929 in Atlanta, Georgia, at the home of his maternal grandfather. M.L.’s grandfather purchased their home on Auburn Avenue in 1909, twenty years before M.L was born. His grandfather allowed the house to be used as a meeting place for a number of organizations dedicated to the education and social advancement of blacks. M.L. grew up in the atmosphere, with his home being used as a community gathering place, and was no doubt influenced by it. M.L.’s childhood was not especially eventfully. His father was a minister and his mother was a musician. He was the second of three children, and he attended all black schools in a black neighborhood. The neighborhood was not poor, however. Auburn Avenue was an area of banks, insurance companies, builders, jewelers, tailors, doctors, lawyers, and other businesses and services. Even in the face of Atlanta’s segregation, the district thrived. Dr. King never forgot the community spirit he had known as a child, nor did he forget the racial prejudice that was a huge barrier keeping black Atlantans from mingling with whites. 
What is the passage mainly about?

A. The prejudice that existed in Atlanta.

B. M.L.’s grandfather

C. Martin Luther King’s childhood

D. The neighborhood King grew up in

Câu 1018 :
When was M.L. born? 

A. in 1909 

B. in 1929 

C. in 1949

D. 20 years after

Câu 1019 :
What is Martin Luthur King well- known for?

A. His publications

B. His neighborhood

C. His childhood.

D. His work in civil rights.

Câu 1020 :
According to the author, M.L. ............... 

A. had a difficult childhood. 

B. was a good musician as a child 

C. loved to listen to his grandfather speak.

D. grew up in a relatively rich area of Atlanta.

Câu 1021 :
Which of the following statements is NOT true?

A. Auburn was a commercial areas.

B. M.L.’s grandfather built their home on Auburn Avenue in 1909.

C. M. L. grew up in a rich, black neighborhood.

D. M.L.’s childhood was uneventful.

Câu 1022 :
From the passage we can infer that: ......... 

A. M.L.’s father was a church member. 

B. people gathered at M.L.’s to perform religious rituals. 

C. M.L.’s father purchased their home on Auburn Avenue. 

D. M.L. had a bitter childhood.

Câu 1023 :
M.L. was ................. by the atmosphere in which he grew up.

A. not affected at all

B. doubted

C. certainly influenced

D. prejudiced

Câu 1024 :
In line 12, the word “They” refers to ………………... 

A. Walls 

B. Animals 

C. Materials

D. Artists

Câu 1025 :
Based on the passage, what is probably true about the south of France?

A. It is home to rare animals.

B. It is known for horse-racing events.

C. It has attracted many famous artists.

D. It has a large number of caves

Câu 1026 :
Which title best summarizes the main idea of the passage?

A. Hidden Prehistoric Paintings

B. Determining the Age of French Caves

C. Wild Animals in Art

D. Exploring Caves Respectfully

Câu 1027 :
Why was painting inside the Lascaux complex a difficult task?

A. It was completely dark inside.

B. The caves were full of wild animals.

C. Many painting spaces were difficult to reach.

D. Painting materials were hard to find.

Câu 1029 :
What does the passage say happened at the Lascaux caves in 1963?

A. Another part was discovered.

B. Visitors were prohibited from entering.

C. A new entrance was created.

D. A new lighting system was installed.

Câu 1031 :
According to the passage, which animals appear most often on the cave walls?

A. Horses

B. Bison

C. Birds

D. Wild cats

Câu 1032 :
Read the following passage and mark the letter A, B, C or D on your answer sheet to indicate the correct answer to each of the questions
Because writing has become so important in our culture, we sometimes think of it as more real than speech. A little thought, however, will show why speech is primary and writing secondary to language. Human beings have been writing at least 5,000 years, but they have been talking for much longer, doubtless ever since there have been human beings. When writing developed, it was derived from and represented speech, although imperfectly. Even today, there are spoken languages that have no written form. Furthermore, we all learn to talk well before we learn to write; any child who is not severely handicapped physically or mentally will learn to talk: a normal man cannot be prevented from doing so. On the other hand, it takes a special effort to learn to write; in the past, many intelligent and useful members of society did not acquire the skill, and even today many who speak languages with writing systems never learn to read or write while some who learn the rudiments of those skills do so imperfectly. To affirm the primacy of speech over writing is not to disparage the later. One advantage writing has over speech is that it is more permanent and makes possible the records that any civilization must have. Thus, if speaking makes us human, writing makes us civilized. 
According to paragraph 1, the author of the passage argues that ................

A. A writing has become too important in today's society.

B. speech is more basic to language than writing.

C. everyone who learns to speak must learn to writing.

D. all languages should have a written form.

Câu 1033 :
The word “doubtless” in the passage mostly means ...........

A. “almost uncertainly”

B. “almost certainly”

C. “almost impossibly”

D. “almost doubtly”

Câu 1034 :
According to the passage, writing .............. 

A. is imperfect, but less than speech 

B. represents speech, but not perfectly 

C. developed from imperfect speech 

D. is represented perfectly by speech

Câu 1035 :
In order to show that learning to write requires effort, the author gives the example of ................

A. people who learn the rudiments of speech

B. people who speak many languages

C. intelligent people who couldn't write

D. severely handicapped children

Câu 1036 :
The word “acquire” in the passage mostly means .............

A. “help somebody learn something by giving information about it”

B. “gain something by our own efforts or ability”

C. “become aware of something by hearing about it”

D. “develop a natural ability or quality so that it improves”

Câu 1037 :
The word “disparage” in the passage mostly means ........

A. “think that something is more important”

B. “make something seem more important”

C. “think about something carefully”

D. “suggest that something is not important or valuable"

Câu 1038 :
In the author's judgment ............ 

A. writing has more advantages than speech 

B. speech is essential but writing has important benefits 

C. speech conveys ideas less accurately than writing does

D. writing is more real than speech.

Câu 1039 :
According to the author, one mark of a civilized society is that if ..............

A. affirms the primacy of speech over writing

B. affirms the primacy of writing over speech

C. teaches its children to speak perfectly

D. keeps written records

Câu 1040 :
Read the following passage and mark the letter A, B, C or D on your answer sheet to indicate the correct answer to each of the questions
After two decades of growing student enrollments and economic prosperity, business schools in the United States have started to face harder times. Only Harvard's MBA School has shown a substantial increase in enrollment in recent years. Both Princeton and Stanford have seen decreases in their enrollments. Since 1990, the number of people receiving Masters in Business Administration (MBA) degrees, has dropped about 3 percent to 75,000, and the trend of lower enrollment rates is expected to continue. There are two factors causing this decrease in students seeking an MBA degree. The first one is that many graduates of four-year colleges are finding that an MBA degree does not guarantee a plush job on Wall Street, or in other financial districts of major American cities. Many of the entry-level management jobs are going to students graduating with Master of Arts degrees in English and the humanities as well as those holding MBA degrees. Students have asked the question, "Is an MBA degree really what I need to be best prepared for getting a good job?" The second major factor has been the cutting of American payrolls and the lower number of entry-level jobs being offered. Business needs are changing, and MBA schools are struggling to meet the new demands. 
What is the main focus of this passage?

A. jobs on Wall Street

B. types of graduate degrees

C. changes in enrollment for MBA school

D. how schools are changing to reflect the economy

Câu 1041 :
The phrase "two decades" in line 1 refers to a period of ...........

A. 10 years

B. 20 years

C. 50 years

D. 100 years

Câu 1042 :
The word "prosperity" in line 1 could be best replaced by which of the following?

A. success

B. surplus

C. nurturing

D. education

Câu 1043 :
Which of the following business schools has NOT shown a decrease in enrollment?

A. Princeton

B. Harvard

C. Stanford

D. Yale

Câu 1045 :
According to the passage, what are two causes of declining business school enrollments?

A. lack of necessity for an MBA and an economic recession

B. low salary and foreign competition

C. fewer MBA schools and fewer entry-level jobs

D. declining population and economic prosperity

Câu 1046 :
Which of the following might be the topic of the next paragraph?

A. MBA schools' efforts to change

B. future economic predictions

C. a history of the recent economic changes

D. descriptions of non-MBA graduate programs

Câu 1047 :
Read the following passage and mark the letter A, B, C or D on your answer sheet to indicate the correct answer to each of the questions
The biological community changes again as one moves from the city to the suburbs. Around all cities is a biome called the "suburban forest". The trees of this forest are species that are favored by man, and most of them have been deliberately planted. Mammals such as rabbits, skunks, and opossums have moved in from the surrounding countryside. Raccoons have become experts at opening garbage cans, and in some places even deer wander suburban thoroughfares. Several species of squirrel get along nicely in suburbia, but usually only one species is predominant in any given suburb -fox squirrels in one place, red squirrels in another, gray squirrels in a third - for reasons that are little understood. The diversity of birds in the suburbs is great, and in the South, lizards thrive in gardens and even houses. Of course, insects are always present. There is an odd biological sameness in these suburban communities. True, the palms of Los Angeles are missing from the suburbs of Boston, and there are species of insects in Miami not found in Seattle. But over wide stretches of the United States, ecological conditions in suburban biomes vary much less than do those of natural biome. And unlike the natural biomes, the urban and suburban communities exist in spite of, not because of, the climate 
If there was a preceding paragraph to this passage it would most likely be concerned with which of the following topics?

A. The migration from cities to suburbs.

B. The biological community in urban areas.

C. The mammals of the American countryside.

D. The history of American suburbs.

Câu 1048 :
The author implies that the mammals of the "suburban forest" differ from most species of trees there in which of the following ways?

A. They were not deliberately introduced.

B. They are considered undesirable by humans.

C. They are represented by a greater number of species.

D. They have not fully adapted to suburban conditions.

Câu 1049 :
The word “thoroughfares” is closet in meaning to ..........

A. neighborhoods

B. lawns

C. open spaces

D. streets

Câu 1050 :
Which of the following conclusions about squirrels is supported by information in the passage?

A. The competition among the three species is intense.

B. Fox squirrels are more common than grey or red squirrels.

C. Two species of squirrels seldom inhabit the same suburb.

D. The reasons why squirrels do well in the suburbs are unknown.

Lời giải có ở chi tiết câu hỏi nhé! (click chuột vào câu hỏi).

Copyright © 2021 HOCTAP247